Add Maths Form5 Reference Book Answer
Add Maths Form5 Reference Book Answer
Add Maths Form5 Reference Book Answer
1 Circular Measure
Sukatan Membulat
1.1 Radian/ Radian
Smart Tip
For a sector with centre O, when the arc length is equal to its radius,
Conversion of radian and degree
the angle at the centre is 1 radian or 1 rad.
Penukaran radian dan darjah
Bagi suatu sektor berpusat O, apabila panjang lengkok adalah sama dengan jejarinya,
sudut pusat ialah 1 radian atau 1 rad. 180°
1 rad =
2π rad = 360° π
π rad = 180° π
P 1° = rad
180°
2 cm PQ = Arc length/Panjang lengkok
O 1 rad 2 cm OP = Radius/Jejari 180° π
a rad = a × a° = a × rad
π 180°
Q
1 rad ≈ 57.29°
Exercise 1 Convert each of the following angles into degrees. [Use π = 3.142]
Tukarkan setiap sudut yang berikut kepada darjah. [Guna π = 3.142]
PL 2 Demonstrate the understanding of circular measure.
Example 1
1 0.46 rad 2 0.738 rad
(a) 1.267 rad
7 0.46 rad 0.738 rad
(b) π rad 180° 180°
8 = 0.46 × = 0.738 ×
π π
Solution = 26.35° = 42.28°
180°
(b) 1.267 rad = 1.267 ×
π
= 72.58° 3 1.5 rad 4 2.47 rad
5 4.726 rad π 3π
6 rad 7 rad
3 4
4.726 rad π 3π
180° rad rad
= 4.726 × 3 4
π π 180° 3π 180°
= × = ×
= 270.74° 3 π 4 π
= 60° = 135°
4π 5π 1
8 rad 9 rad 10 2 π rad
5 6 6
4π 5π 1
rad rad 2 π rad
5 6 6
4π 180° 5π 180° 13π 180°
= × = × = ×
5 π 6 π 6 π
= 144° = 150° = 390°
Exercise 2 Convert each of the following angles into radian. [Use π = 3.142]
Tukarkan setiap sudut yang berikut kepada radian. [Guna π = 3.142]
PL 2 Demonstrate the understanding of circular measure.
Example 2
1 26° 2 37.5°
67.3° π π
26° = 26° × 37.5° = 37.5° ×
180° 180°
Solution = 0.4538 rad = 0.6546 rad
π
67.3° = 67.3° ×
180°
= 1.175 rad
Exercise 3 By using the formula s = rθ, find the arc length, s for each of the following sectors.
[Use π = 3.142]
Dengan menggunakan s = rθ, cari panjang lengkok, s bagi setiap sektor yang berikut. [Guna π = 3.142]
PL 2 Demonstrate the understanding of circular measure.
Example 3
78° P
m O
O 9c 7 cm r
121° θ rad s
O
Solution Solution Q
π
78° = 78° × 360° – 121° s = rθ
180°
= 239°
= 1.362 rad π where,
239° × rad dengan keadaan,
180°
s = rθ s = arc length
= 4.172 rad panjang lengkok
= 9 × 1.362
r = radius/ jejari
= 12.258 cm θ = angle in radian
s = rθ
sudut dalam radian
= 7 × 4.172
= 29.204 cm
s s
m
6 cm 4.968 rad
4c
O 0.83 rad O 1.36 rad O
s 8 cm
s = rθ s = rθ
s = rθ s = 8 × 4.968
s = 4 × 0.83 s = 6 × 1.36
= 8.16 cm = 39.744 cm
= 3.32 cm
4 5 6
12 m s
16 cm
48° 20 m 147°
152° s
O O O
π
48° = 48° × rad
180° π
= 0.838 rad 152° = 152° × rad 360° – 147° = 213°
180°
= 2.653 rad π
s = rθ 213° = 213° × rad
180°
= 12 × 0.838 s = rθ = 3.718 rad
= 10.056 m = 16 × 2.653
= 42.448 cm s = rθ
= 20 × 3.718
= 74.36 m
Exercise 4 By using the formula s = rθ, find the radius, r. [Use π = 3.142]
Dengan menggunakan rumus s = rθ, cari jejari, r. [Guna π = 3.142]
PL 2 Demonstrate the understanding of circular measure.
Example 4
1 2
P P 7 cm
P
Q
12 cm
15 cm r cm
0.986 rad 0.845 rad
O 0.8 rad
r cm O
Q O
r cm
Q
Solution rθ = s rθ = s
rθ = s r(0.8) = 12 r(0.845) = 7
r(0.986) = 15 12 7
r = 0.8 r = 0.845
15
r = 0.986 = 15 cm = 8.284 cm
= 15.213 cm
r cm 122.6° rm Q r cm Q
O
O Q
P P
π
122.6° = 122.6° × π
180° 351.75° = 351.75° ×
rθ = s 180°
= 2.14 rad
r(3.96) = 60 = 6.14 rad
rθ = s 60
r(2.14) = 16 r = 3.96 rθ = s
16 = 15.152 m r(6.14) = 75
r = 2.14 75
r = 6.14
= 7.477 cm
= 12.215 cm
Example 5
1 2
P P
O
6 cm
u 4 cm
Q 7.32 cm
u u
O Q O
P 5 cm
8.5 cm 6 cm
Q
Find the value of θ, in radian. Find the value of θ, in radian.
Cari nilai θ, dalam radian. Find the value of θ, in radian.
Cari nilai θ, dalam radian.
Cari nilai θ, dalam radian.
Solution
rθ = s
rθ = s rθ = s
(5) θ = 4
(6) θ = 8.5 4 (6) θ = 7.32
8.5 θ=5 7.32
θ= 6 θ= 6
= 0.8 rad
= 1.417 rad = 1.22 rad
3 4 5
P O Q
Q
7 cm 19 cm
x° P
8 cm 12 cm
Q x° O
12.8 cm O
P x°
Exercise 6 Calculate the perimeter of the shaded segment PRQ. [Use π = 3.142]
Hitung perimeter tembereng berlorek PRQ. [Guna π = 3.142]
PL 3 Apply the understanding of circular measure to perform simple tasks.
Example 6 1
P
P
Q
1.5 rad O 0.8 rad Q
O
9 cm R 15 cm
R
180°
1.5 rad = 1.5 × = 85.93° 180°
π 0.8 rad = 0.8 × = 45.83°
Perimeter of the shaded segment π
Perimeter tembereng berlorek Perimeter of the shaded segment
= 2r 2 – 2r 2 cos θ/kos θ + rθ = 2(15)2 – 2(15)2 cos 45.83° + (15)(0.8)
= 2(9)2 – 2(9)2 cos 85.93°/kos 85.93° + (9)(1.5) = 11.681 + 12
= 12.268 + 13.5 = 23.681 cm
= 25.768 cm
2 3
P P
Q
Q 1.74 rad
O 1.66 rad O R
12 cm
7 cm
R
180° 180°
1.66 rad = 1.66 × = 95.10° 1.74 rad = 1.74 × = 99.68°
π π
O 1.66 rad Q
Perimeter of the shaded segment Perimeter of the shaded segment
= 2(7)2 – 2(7)2 cos
9 cm95.10° + (7)(1.66) = 2(12)2 – 2(12)2 cos 99.68° + (12)(1.74)
= 10.33 + 11.62 R = 18.342 + 20.88
= 21.95 cm = 39.222 cm
180° 180°
2.56 rad = 2.56 × = 146.66° 1.86 rad = 1.86 × = 106.56°
π π
Perimeter of the shaded segment Perimeter of the shaded segment
= 2(16)2 – 2(16)2 cos 146.66° + (16)(2.56) = 2(15)2 – 2(15)2 cos 106.56° + (15)(2π – 1.86)
= 30.655 + 40.96 = 24.047 + 66.36
= 71.615 cm = 90.407 cm
F O
G
The garden has to be fenced. Calculate the length, in
1.35 rad O
m, of the fence needed.
16 m Taman itu akan dipasang pagar. Hitung panjang, dalam m,
H pagar yang diperlukan.
Perimeter
= FH + Arc length FGH/ Panjang lengkok FGH
= 2(16)2 – 2(16)2 cos 77.34° + (16)(2π − 1.35)
= 19.995 + 78.944
= 98.939 m
B
0.874 rad O
4m A 1.65 km
O
The curvatue part of the stage will be decorated with It is given that the perimeter of the resort is 7.425 km.
ribbons. Calculate the length, in m, of the ribbons Find /AOB, in radians.
needed. Diberi bahawa perimeter resort itu ialah 7.425 km. Cari /AOB,
Sekeliling panjang lengkok pentas itu akan dihiasi dengan reben. dalam radian.
Hitung panjang, dalam m, reben yang diperlukan.
OA + OB + Arc length AB = 7.425
Length of ribbons 1.65 + 1.65 + (1.65)(/AOB) = 7.425
= (4)(2π − 0.874) (1.65)(/AOB) = 4.125
= (4)(5.41) /AOB = 2.5 radians
= 21.64 m
4 The shaded region in the diagram below shows a 5 In the diagram below, the shaded region shows a
lawn. OPR is a sector with centre O. stage in the shape of a segment with centre O.
Kawasan berlorek dalam rajah di bawah menunjukkan kawasan Dalam rajah di bawah, kawasan berlorek menunjukkan sebuah
halaman rumput. OPR ialah sektor berpusat O. pentas berbentuk tembereng berpusat O.
P Q
R
6m
P
2.15 rad
θ 9m
O R
4m Q O
Calculate the perimeter, in m, of the lawn. The stage is to be surrounded by ribbons. Calculate
Hitung perimeter, dalam m, bagi kawasan halaman rumput. the length, in m, of the ribbons requirement.
Q
4 Sekeliling pentas itu akan diikat dengan reben. Hitung panjang,
cos θ = 6 dalam m, reben yang diperlukan.P R
θ = 48.19°
π 180° 9 m 2.15 rad
= 48.19° × 2.15 rad = 2.15 ×
180° π
= 0.8412 rad = 123.17°
O
PQ = 6 × sin 48.19° Perimeter
= 4.472 m = 2(9)2 – 2(9)2 cos 123.17° + (9)(2.15)
Perimeter = 15.831 + 19.35
= PQ + QR + Arc length PR = 35.181 m
= 4.472 + 2 + (6)(0.8412) The length of the ribbons required is 35.18 m.
= 11.519 m
Smart Tip
Area of minor sector
Luas sektor minor
1 2 θ in radian
= rθ θ dalam radian
r
2
Minor sector Major sector
Sektor minor θ Area of major sector
2π - θ Sektor major
r Luas sektor major
1 2
= r (2π − θ)
2
Example 8 1 2
P
P
Common Error
1
(a) 2 r2 θ
1
= 2 × 92 × 83°
= 3 361.5 cm2
θ must be converted to 3 4
radian. P
θ mesti ditukarkan kepada
5π
radian. 3 O 14 cm
P
98 cm
1 O 160˚
(b) 2 r2 θ
1 Q
= 2 × 92 × 1.4 Q
= 56.7 cm2 Area of sector OPQ ∠POQ = 360° – 160° = 200°
The value of θ must be in 1 π
= 2 r2 θ 200° = 200° × 180°
at least 3 decimal places.
Nilai θ mesti sekurang- 1 5 = 3.491 rad
kurangnya dalam 3 tempat = 2 × 92 × 3 π
perpuluhan. Area of sector OPQ
= 212.085 cm2
1
= 2 r2 θ
1
= 2 × 142 × 3.491
= 342.12 cm2
Area/Luas = 36 cm2
P
P 1.486 rad
O O
r cm
0.684 rad r cm
1.2 rad Q
O r Q
P Q
1 2 1 2
2 r θ = Area of sector OPQ 2 r θ = Area of sector OPQ
Solution 1 2 1 2
1 2 2 × r × (0.684) = 7.84 2 × r × (1.486) = 116
7.84 × 2 116 × 2
2 r θ = Area of sector OPQ r2 = 0.684 r2 = 1.486
Luas sektor OPQ
= 22.924 = 156.124
1 2
r = 12.495 cm
2 × r × (1.2) = 36 r = 4.788 cm
36 × 2
r2 = 1.2
= 60
r = 60
= 7.746 cm
P 1.034 rad
r cm P
1.034 rad 2.095 rad
O Q
Q P
O r cm O
47˚
r cm
Q
1 2 1 2
2 r θ = Area of sector OPQ 2 r θ = Area of sector OPQ π
1 2 47° × 180° = 0.8204 rad
1 2
2 × r × (2π − 1.034) = 378 2 × r × (2π − 2.095) = 471.28 1 2
1 2 1 2 2 r θ = Area of sector OPQ
2 × r × 5.25 = 378 2 × r × 4.189 = 471.28 1 2
378 × 2
r2 = 5.25 471.28 × 2
r2 = 4.189 2 × r × (2π − 0.8204) = 535.43
1 2
= 144 = 225 2 × r × 5.4636 = 535.43
r = 12 cm r = 15 cm 535.43 × 2
r2 = 5.4636
= 196
r = 14 cm
Solution
1 2 1 2
1 2
2 r θ = Area of sector OPQ 2 r θ = Area of sector OPQ
2 r θ = Area of sector OPQ
Luas sektor OPQ 1 1
2 × 4 × θ = 6.982
2
2 × 12 × θ = 102.8
2
1 6.982 × 2
θ= 102.8 × 2
2 × 8 × θ = 39.872
2
42 θ=
39.872 × 2 122
θ= θ = 0.8728 rad θ = 1.428 rad
82
= 1.246 rad
14 cm P
P 16 cm
θ
7 cm O θ O
θ
O
Q
Q
1 2 1 2 1 2
2 r θ = Area of sector OPQ 2 r (2π − θ) = Area of sector OPQ 2 r (2π − θ) = Area of sector OPQ
1 1 1
2 × 7 × θ = 128.30
2
2 × 14 × (2π − θ) = 240 2 × 16 × (2π − θ) = 693.76
2 2
128.30 × 2 2π − θ = 2.449 2π − θ = 5.42
θ=
72 θ = 2π − 2.449 θ = 2π − 5.42
θ = 5.237 rad
= 3.835 rad = 0.864 rad
10
(
θ
θ r 1 2 1 2 1
= r (θ – sin θ) r θ – r2 sin θ
2 Q P 2 2
Q P 1 2
r sin θ
T 2 In radian In degree
Dalam radian ( rad) Dalam darjah (°)
Exercise 11 Find the area of shaded segment of each of the following. [Use π = 3.142]
Cari luas bagi tembereng berlorek bagi setiap yang berikut. [Guna π = 3.142]
PL 3 Apply the understanding of circular measure to perform simple tasks.
Example 11
Solution
180° Alternative Method
0.85 rad = 0.85 × = 48.7°
π
Area of shaded segment Area of shaded segment
Luas tembereng berlorek Luas tembereng berlorek
O 1
1 1 = 2 r2 (θ − sin θ)
8 cm 0.85
1.45 rad = 2 × r2 × θ − 2 × r2 × sin θ
1
1 1 = 2 × (8)2 (0.85 − sin 48.7°)
Q P = 2 × (8)2 × 0.85 − 2 × (8)2 × sin 48.7°
= 32(0.85 – 0.7513)
= 27.2 − 24.040 = 3.158 cm2
= 3.160 cm2
1 2
O
O
12 cm
P 3.1 rad
12 cm 1.45 rad
Q
Q P
180° 180°
1.45 rad = 1.45 × = 83.07° 3.1 rad = 3.1 × = 177.59°
π π
Area of shaded segment Area of shaded segment
1 1 1 1
= × r2 × θ − × r2 × sin θ = × r2 × θ × − × r2 × sin θ
2 2 2 2
1 1 1 1
= 2 × (12)2 × 1.45 − 2 × (12)2 × sin 83.07° = 2 × 122 × 3.1 − 2 × 122 × sin 177.59°
= 104.4 − 71.47 = 223.2 – 3.028
= 32.93 cm2 = 220.17 cm2
11
2.43 rad
Q O 68˚
O 14 cm
16 cm
Q
180° π
2.43 rad = 2.43 × = 139.21° 68° = 68° × = 1.187 rad
π 180°
Area of shaded segment Area of shaded segment
1 1 1 1
= × r2 × θ − × r2 × sin θ = × r2 × θ − × r2 × sin θ
2 2 2 2
1 1 1 1
= 2 × (14)2 × 2.43 − 2 × (14)2 × sin 139.21° = 2 × (16)2 × 1.187 − 2 × (16)2 × sin 68°
= 238.14 − 64.022 = 151.936 − 118.680
= 174.12 cm2 = 33.26 cm2
5 Q 6
P
15 cm
1.4 rad
136° O Q
O 12 cm
π 180°
136° = 136° × = 2.374 rad 1.4 rad = 1.4 × = 80.2°
180° π
Area of shaded segment
Area of shaded segment 1 1
1 1 2
= πr2 − r2θ − r2 sin θ
2
= × r2 × θ − × r2 × sin θ
2 2 1 1
1 1 2
= 3.142 × 12 − × (12)2 × 1.4 − × (12)2 × sin 80.2
2
2
= 2 × (15)2 × 2.374 − 2 × (15)2 × sin 136° = 452.448 – (100.8 – 70.95)
= 267.075 − 78.149 = 422.60 cm2
= 188.93 cm2
A 0.68 rad C
B
16 cm 1.782 rad
Find the area, in cm2, of the shaded region.
Cari luas, dalam cm2, bagi rantau berlorek.
0.68 rad 1.36 rad
[Use/Guna π = 3.142] A B
O 8 cm
Area of the shaded region
= area of ∆AOC + area of sector OCB
1 1
= 2 × 82 ×(sin 102.09o) + 2 × 82 × 1.36
= 31.29 + 43.52
= 74.81 cm2
12
π
3
rad
π 180o
5π
( ) π
= (67.18 − 18) + (31.18) 6 + (67.18) 3 ()
rad = 3 × π = 60
o
3 = 49.18 + 81.64 + 70.36
AB = 201.18 cm
AO = cos 60
o
3 The diagram shows the plan of a garden. HQG is a semicircle with centre O and has a radius of 10 m. FPG is
a sector with centre P and has a radius of 18 m. Sector OQG is a lawn. The shaded region is a flower bed. The
whole flower bed has to be fenced. It is given that PQ is 10 m and ∠QOG = 1.795 rad.
F
Rajah menunjukkan pelan bagi sebuah taman. HQG ialah sebuah semibulatan berpusat O dan
mempunyai jejari 10 m. FPG ialah sebuah sektor berpusat P dan mempunyai jejari 18 m. Sektor Q
OQG ialah kawasan halaman rumput. Rantau berlorek ialah kawasan tanaman bunga. Seluruh
kawasan tanaman bunga perlu dipagarkan. Diberi bahawa PQ ialah 10 m dan ∠QOG =
1.795 rad.
Calculate/ Hitung Evaluating
H G
P O
(a) the area, in m2, of the lawn,/ luas, dalam m2, bagi kawasan halaman rumput,
(b) the length, in m, of the fence required for fencing the flower bed,
panjang, dalam m, pagar yang diperlukan untuk memagar kawasan tanaman bunga,
(c) the area, in m2, of the flower bed./ luas, dalam m2, bagi kawasan tanaman bunga.
[Use/Guna π = 3.142]
F
8m (c) Area of sector FPG – area of ∆QPO –
∠FPO = ∠QOP Q
area of sector OQG
= π – 1.795 rad
= 1.347 rad 10 m 10 m ( 1
) (
1
= 2 × 182 × 1.347 − 2 × 10 × 8 sin 77.17o − )
180 o
= 1.347 ×
= 77.17 o
π H
P
)1.347 rad )1.795 rad
8m O 10 m
G ( 1
2 × 102 × 1.795 )
= 218.21 − 39.001 − 89.75
1
(a) 2 × 102 × 1.795 = 89.75 m2 = 89.46 m2
13
Given diameter = 24 m, [ radius = 12 m Area to be planted with rose trees is 149.11 m2.
Given 2 m2 can plant 5 rose trees. Thus, 1 m2
Arc length KL = OL + arc length LM 5
can plant rose trees.
12 × /KOL = 12 + 12(p − /KOL) 2
/KOL = 1 + p − /KOL
Total number of rose trees to be planted
2/KOL = 1 + p 5
= 149.11 ×
2
1
/KOL = 2 1 + p ( ) = 372.78
373 rose trees
= 2.071 rad
1
Area sector KOL = 2 × 2.071 × 122
= 149.11 m2
2 The diagram shows the plan of a park. EOF and GOH are two sectors
with common centre O. Given GO = 20 m, GE = EO and the arc length
GH = 8p m.
Rajah menunjukkan pelan bagi sebuah taman. EOF dan GOH ialah dua buah sektor
G H
dengan pusat sepunya O. Diberi GO = 20 m, GE = EO dan panjang lengkok GH =
8p m. E F
O
The shaded region will be covered with tiles. If the cost to cover an area
of 1 m2 is RM150, calculate the total cost.
Kawasan berlorek akan ditutupi dengan jubin. Jika kos untuk menutup kawasan seluas
1 m2 ialah RM150, hitung jumlah kos.
Area of the shaded region = Area of sector OGH – Area of sector OEF
1 2p 1 2p
= 2 × 202 × 5 − 2 × 102 × 5
= 251.36 – 62.84
= 188.52 m2
14
15
8 cm
3.5 cm
2.69 rad
G E K O F
Diagram 1 / Rajah 1
Given HK = 3.5 cm and /HOF = 2.69 radians.
Diberi HK = 3.5 cm dan /HOF = 2.69 radian.
[Use/ Guna = 3.142]
Calculate/ Hitung
(a) the perimeter, in cm, of the whole diagram,
perimeter, dalam cm, bagi seluruh rajah,
[4 marks/markah]
(b) the area, in cm2, of the shaded region.
luas, dalam cm2, bagi kawasan berlorek.
[3 marks/markah]
Q θ O S
Diagram 2 / Rajah 2
1
Given the arc length PQR is 31.5 cm and the area of the shaded region is 118 8 cm2.
1
Diberi panjang lengkok PQR ialah 31.5 cm dan luas kawasan berlorek ialah 118 cm2.
8
[Use/ Guna = 3.142]
Calculate/ Hitung
(a) the radius, in cm, of the circle,
jejari, dalam cm, bagi bulatan,
[3 marks/markah]
(b) θ, in radians,
θ, dalam radian,
[2 marks/markah]
(c) the area, in cm2, of the segment PRS.
luas dalam cm2, bagi tembereng PRS.
[3 marks/markah]
3 Diagram 3 shows a semicircle with centre O and a diameter of 26 cm. BA is a tangent to the semicircle at point A.
Rajah 3 menunjukkan sebuah semibulatan berpusat O dan berdiameter 26 cm. BA ialah tangen kepada semibulatan di titik A.
B
A D
O
Diagram 3 / Rajah 3
16
Section B
4 Diagram 4 shows a circle with centre O and radius 18 cm inscribed in a sector EFG with centre E. The straight
lines EF and EG are tangents to the circle at point H and point K respectively.
Rajah 4 menunjukkan sebuah bulatan berpusat O dan berjejari 18 cm terterap di dalam sektor EFG berpusat E. Garis lurus EF dan EG
ialah tangen kepada bulatan masing-masing pada titik H dan titik K.
P
F G
18 cm O
H K
60˚
E
Diagram 4/ Rajah 4
Calculate/Hitung
(a) the length, in cm, of the arc FG,
panjang, dalam cm, bagi lengkok FG,
[5 marks/markah]
(b) the area, in cm2, of the shaded region.
luas, dalam cm2, bagi kawasan berlorek.
[5 marks/markah]
H O TS Zo n e
1 Diagram 1 shows two circles with centres K and L respectively and touch at point P. The larger circle has a radius
of 14 cm and the smaller circle has a radius of 10 cm. The straight line FG is a common tangent to the circles at
point F and point G.
Rajah 1 menunjukkan dua buah bulatan masing-masing berpusat K dan L dan bersentuh pada titik P. Bulatan yang lebih besar mempunyai
jejari 14 cm dan bulatan yang lebih kecil mempunyai jejari 10 cm. Garis lurus FG ialah tangen sepunya kepada kedua-dua bulatan itu pada
titik F dan titik G.
P
L
K
θ
10 cm
10 cm 14 cm
F G
Diagram 1/ Rajah 1
Given ∠KLG = θ radians,
Diberi ∠KLG = θ radian,
(a) show that θ = 1.404 (round off to three decimal places), Applying
17
2 Differentiation
Pembezaan
2.1 Limit and its Relation to Differentiation / Had dan Hubungannya dengan Pembezaan
Smart Tip
Limit/Had Limit idea/ First principles:
lim 𝑓(𝑥)= C can be read as “the limit of f of x, as x Idea had/ Prinsip pertama:
x→a
approaches a, is C”. dy δy
= lim , where 𝛿x and 𝛿y are small changes of x and y
had f(x) = C boleh dibaca sebagai “had f bagi x, apabila x menghampiri a, dx δx → 0 δx
x→a
ialah C”. respectively;
dy had δy
dx = δx → 0 δx , dengan keadaan 𝛿x dan 𝛿y masing-masing adalah
perubahan kecil dalam x dan y;
or/atau
dy / ℎ𝑎𝑑 f(x + δx)
x − f(x)
= f ′(x) = δxlim
dx → 0 δx → 0
δx
Example 1
1 lim / ℎ𝑎𝑑 (13 − 2x)
x→2 x→2
lim / ℎ𝑎𝑑 x − 9
2
= 13 − 2(2)
x→3 x→3 x − 3
=9
Solution
lim / ℎ𝑎𝑑 x − 9 = lim / ℎ𝑎𝑑 (x + 3)(x − 3)
2
18
Example 2
1 lim / ℎ𝑎𝑑 (0.8)n
n→∞ n→∞
2 3n − 2
2
→ ∞ 5 + 8n2
3−
4 = nlim
= nlim n
1+
3 = nlim
→∞ →∞ 3 2 2
n +2 3− 3−
n n2 ∞2
= 4 2 = nlim =
3 3− →∞ 5 5
1+ ∞ +8 +8
∞ = n2 ∞2
4 3 3−0 3
+2
= 1+0 ∞ = 0+8 = 8
3−0 3
=4 = 0+2 = 2
Example 3
lim / ℎ𝑎𝑑 m − 3 m− 6
1 m→3 m→3
m2 − 9 2 lim / ℎ𝑎𝑑
m→6 m→6 m 2
− 36
lim / ℎ𝑎𝑑 m – 4
2
m→2 m→2
m–2
m− 3 m− 6
= mlim
→ 3 (m + 3)(m − 3)
= mlim
→ 6 (m + 6)(m − 6)
Solution
lim / ℎ𝑎𝑑 m – 4
2
1 1
m→2 m→2
m–2 Common Error
= mlim
→3 m + 3
= mlim
→6 m + 6
19
Example 4 1 y = 4x + 5
y = 7x2
dy f(x + δx) – f(x)
= lim δx
Solution dx δx → 0
[4(x + δx) + 5] – (4x + 5)
dy lim had f(x + δx) – f(x) = δlim
= /
dx δx → 0 δx → 0 δx x→0 δx
7(x + δx)2 – 7x2 4x + 4δx + 5 – 4x – 5
= δlim / had = δlim
x→0 δx
x → 0 δx → 0 δx
7[x2 + 2xδx + (δx)2] – 7x2 4δx
= δlim / had = δlim
x → 0 δx
x → 0 δx → 0 δx
7x2 + 14xδx + 7(δx)2 – 7x2 = lim 4
= δlim / had δx
δx → 0
x → 0 δx → 0 =4
14xδx + 7(δx)2
= δlim / had
x → 0 δx → 0 δx
lim had
= δx → 0 / δx → 0 (14x + 7δx)
= 14x + 7(0)
= 14x
2 y = 2x2 3 y = x2 + x
20
y = 3x2 y = 5x 1
y + δy = 5(x + δx)
Solution
y + δy = 5x + 5δx 2
y = 3x2 1
y + δy = 3(x + δx)2 2 − 1,
y + δy = 3[x2 + 2xδx + (δx)2]
δy = 5δx
y + δy = 3x2 + 6xδx + 3(δx)2 2
δy
2 − 1, δx = 5
δy = 6xδx + 3(δx)2 lim
δy
δx → 0 δ x
=5
δy
δx = 6x + 3δx dy
lim / ℎ𝑎𝑑 δy dx = 5
δx → 0 δx → 0 δx = 6x + 3(0)
dy
dx = 6x
Smart Tip
lim / ℎ𝑎𝑑 δy = dy
δx
δx → 0 δx → 0
dx
2 y = 4x2 3
3 y= x
y = 4x2 1
3
y + δy = 4(x + δx)2 y = x 1
y + δy = 4[x2 + 2xδx + (δx)2] 3
y + δy = 4x2 + 8xδx + 4(δx)2 2 y + δy = x + δx 2
2 − 1, 2 – 1,
δy = 8xδx + 4(δx)2 3 3
δy = x + δx − x
δy
δx = 8x + 4δx 3x − 3(x + δx)
δy = x(x + δx)
lim
δy
δx → 0 δ x
= 8x + 4(0)
–3δx
dy δy = x(x + δx)
dx = 8x
δy –3
δx = x(x + δx)
lim
δy –3
δx → 0 δ x
=
x[x + (0)]
dy 3
=– 2
dx x
21
Example 6
1 y = 3x6
1 dy
(a) y = 4x3 (b) f(x) = 3x5 (c) y = x2
dx = 6(3x )
6–1
Solution = 18x5
(a) y = 4x3 (b) f(x) = 3x5 1
, (c) x2 = x–2
dy f (x) = 5(3x5−1)
dx = 3(4x2 )
3−1
= 15x4 dy
dx = –2x –3
–2 – 1
= 12x
= –2x
2
= – x3
2 y = –6x4
Smart Tip dy
dx = 4(–6x )
4–1
dy
1 If y = axn, then = naxn – 1, where a and n are constants. = –24x3
dx
dy
Jika y = axn, maka dx = naxn – 1, dengan keadaan a dan n ialah pemalar.
dy
2 If y = a, then = 0, where a is a constant.
dx
dy
Jika y = a, maka dx = 0, dengan keadaan a ialah pemalar.
dy 6
3 Notation of f ’(x) is equivalent to
’(x) when y = f(x).
f x).
f( 3 y= 7
dx
dy
Tatatanda f ’(x) adalah setara dengan
dx
f x).
f(
apabila y = f(x). dy
dy dx = 0
4 The derivative of y with respect to x is written as dx .
dy
Terbitan y terhadap x ditulis sebagai dx .
2 x6 4
4 y = x3 5 f(x) = 2 6 f(x) = – x2
5x3 1 18x–2
7 y= x 8 y = 3x2 9 f(x) = x
= – 23
3x
22
= 8x3
Solution
x3 x = –2,
(a) y = 6
(b) f(x) = 3x4 dy
dy x3–1 f ′(x) = 4(3x4−1) = 8(–2)3
dx
dx = 3 6 = 12x3 = –64
x2 f ′(−2) = 12(−2)3
= 2 = −96
dy (4)2
x = 4, dx = 2
=8 2 y = 2x7, x = 1
Calculator Corner dy
dx = 7(2x )
7–1
Determine the solution of Example 7 by using scientific calculator Casio
fx-570EX. = 14x6
Menentukan penyelesaian bagi Contoh 7 menggunakan kalkulator saintifik Casio fx-570EX.
Step 1/ Langkah 1: x = 1,
Press ‘MENU’ and find 1 : Calculate (normal display), press ‘ = ’. dy
Tekan ‘MENU’ dan cari 1 : Calculate (paparan normal), tekan ‘ = ’. = 14(1)6
dx
Step 2/ Langkah 2:
= 14
SHIFT ∫ 3 x x 4 (–) 2 =
3 f(x) = 3x2 , x = 5 x6 3
4 f(x) = – 3 , x = 2 5 f(x) = x2 , x = –1
f ′(x) = 2(3x2–1)
x6–1 f(x) = 3x–2
= 6x
f ′(x) = –6 3
f ′(x) = –2(3x–2–1)
f ′(5) = 6(5) = –2x5 = –6x–3
= 30 6
f ′(2) = –2(2)5 =– 3
x
= –64
6
f ′(–1) = –
(–1)3
6
= – (–1)
=6
6 f(x) = 9x2 , x = –4 6 1
7 y = x3 , x = 3 8 y = 5x7 , x = –2
f ′(x) = 2(9x2–1)
y = 6x–3 x–7
= 18x y= 5
dy
f ′(–4) = 18(–4) = –3(6x–3–1) dy –7(x–7–1)
dx
= –72 = –18x–4 dx = 5
–18 –7x–8
= 4 =
x 5
–7
dy –18 = 8
= 5x
dx (3)4 dy –7
2 =
=– dx 5(–2)8
9
7
=–
1 280
23
Example 8
1 2x7 − 8x + 9 3 1
2 4x2 +
3x + 5x − 2
4
d 4x − 5x + 9
dx (2x − 8x + 9)
7
PAK-21
d 3 1
Solution = 7(2x7 – 1) – 1(8x1 – 1) dx 4x + 4x − 5x + 9
2
VIDEO
d = 14x6 − 8
dx (3x + 5x − 2)
4
3x–1–1
= 2(4x2 – 1) + (–1)
4 −
5x1 – 1
= 4(3x4 – 1) + (1)(5x1 – 1 )
3x –2
= 12x3 + 5 = 8x − −5
4
3
= 8x − 4x2 − 5
2 x7 2x x4 − 3
3 x5 + 4 5
x 8 − 3 +9 x
d 5 d x7 2x d x4 − 3
dx (x + 2x )
–1
dx 8 − 3 + 9 dx x
= 5(x5 – 1) + (–1)(2x–1 – 1) x7– 1 2x1– 1 d 3
= 5x4 − 2x–2 =7
8 − 3
= dx (x − 3x–1)
2 7x6 2 = 3(x3 – 1) – (–1)(3x–1 – 1)
= 5x4 − 2 = −3
x 8 = 3x2 + 3x–2
3
= 3x2 + 2
x
Smart Tip
dy
is also called gradient function.
dx
dy juga dikenali sebagai fungsi kecerunan.
dx
24
= 9x2 − 30 + 25x– 2
dy
dx = 18x − 50x
–3
50
= 18x − x3
3 2 5x4 + 6 3x4 − 5x
6 y= x− x 7 y=
x2
8 y=
x3 + 2x − 1
3 2 5x4 + 6 3x4 − 5x
y= x− x y=
x2
y=
x3 + 2x − 1
3 3 = 5x + 6x–2
2
= 3x − 5x + 2x − 1
–2
= x − x x − x
dy = 5x − 5x–2 − 1
9 dx = 10x − 12x
–3
= x2 − 6 + x2 dy
dx = 5 + 10x
–3
12
= x2 − 6 + 9x–2 = 10x − x3 10
= 5 + x3
dy
dx = 2x − 18x
–3
18
= 2x − x3
Smart Tip
dy dy du
1 It is given that y = un, then = × , where u is a function of x (Chain rule).
dx du dx
dy dy du
Diberi bahawa y = un, maka dx = du × dx , dengan keadaan u ialah fungsi bagi x (Petua rantai).
dy dv du
2 It is given that y = uv, then = u + v , where u and v are functions of x (Product rule).
dx dx dx
dy dv du
Diberi bahawa y = uv, maka dx = u dx + v dx , dengan keadaan u dan v ialah fungsi bagi x (Petua hasil darab).
du dv
v— – u—
u dy dx dx
3 It is given that y = , then = , where u and v are functions of x (Quotient rule).
v dx v2
du dv
v— – u—
u dy dx dx
Diberi bahawa y = v , maka dx = , dengan keadaan u dan v ialah fungsi bagi x (Petua hasil bahagi).
v2
25
Example 10
1 y = (3x − 1)4
y = (x2 + 4x)3
y = (3x − 1)4
dy d 2
= 3(x2 + 4x)2 (x + 4x) dy d
dx dx = 4(3x − 1)3 (3x − 1)
dx dx
= 3(x2 + 4x)2(2x + 4)
= 3(x2 + 4x)2 • 2(x + 2) = 4(3x − 1)3(3)
= 6(x2 + 4x)2(x + 2) = 12(3x − 1)3
Alternative Method
du
Let/ Katakan u = x2 + 4x ∴ dx = 2x + 4
dy
y = u3 ∴ du = 3u2
dy dy du
dx = du × dx
= 3u2 × (2x + 4) = 3(x2 + 4x)2 • 2(x + 2)
= 6(x2 + 4x)2(x + 2)
3 6
4 y= 5 y=
2x + 1 (x − 5)4
y = 3(2x + 1)–1 y = 6(x − 5)–4
dy d dy d
= –1[3(2x + 1)–2] (2x + 1) = –4[6(x − 5)–5] (x − 5)
dx dx dx dx
dy
= –3(2x + 1)–2(2) = –24(x − 5)–5 ( 1)
dx
6 24
=– =–
(2x + 1)2 (x − 5)5
26
Example 11 1 y = 3x (4x2 – 1)
du
y = (2x + 1)(3x2 − 2) u = 3x, ∴ =3
dx
dv
Solution Smart Tip v = 4x2 − 1, ∴ = 8x
dx
du
u = 2x + 1, ∴ =2 Expand the equation
dx for checking.
dy dv du
=u +v
dv Kembangkan persamaan dx dx dx
v = 3x2 − 2, ∴ = 6x
dx untuk tujuan semakan. = (3x)(8x) + (4x2 − 1)(3)
dy dv du = 24x2 + 12x2 − 3
=u +v y = (2x + 1)(3x2 − 2)
dx dx dx = 36x2 − 3
= 6x3 − 4x + 3x2 − 2
= (2x + 1)(6x) + dy
dx = 18x − 4 + 6x
2
(3x2 − 2)(2)
= 12x2 + 6x + 6x2 − 4
= 18x2 + 6x − 4
dy dv du dy dv du
=u +v =u +v
dx dx dx dx dx dx
= (2x )(12x) + (6x2 + 7)(8x3)
4
= (2x + 1)(2x + 4) + (x2 + 4x − 3)(2)
= 24x5 + 48x5 + 56x3 = 4x2 + 8x + 2x + 4 + 2x2 + 8x− 6
= 72x5 + 56x3 = 6x2 + 18x − 2
dy dv du dy dv du
=u +v =u +v
dx dx dx dx dx dx
= (4x − 5)(6x) + (3x2 + 8)(4) = (x − 1)( 2x − 2) + (x2 − 2x + 5)(1)
= 24x2 − 30x + 12x2 + 32 = 2x2 − 2x − 2x + 2 + x2 − 2x + 5
= 36x2 − 30x + 32 = 3x2 − 6x + 7
27
Example 12
6x
1 y=
3x + 2
3x − 4
y=
2x2 − 1 du
u = 6x, ∴ =6
Solution dx
du dv
u = 3x − 4, ∴ =3 v = 3x + 2, ∴ =3
dx dx
dv du dv
v = 2x2 − 1, ∴
dx
= 4x
dy v— —
dx – u dx
=
du dv dx v2
dy v— —
dx − u dx (3x + 2)(6) – (6x)(3)
= =
dx v2 (3x + 2)2
(2x − 1)(3) − (3x − 4)(4x)
2 18x + 12 – 18x
= =
(2x2 – 1)2 (3x + 2)2
6x2 − 3 − 12x2 + 16x 12
= = (3x + 2)2
(2x2 − 1)2
16x − 6x2 − 3
= (2x2 − 1)2
7 3x − 1
2 y= 3 y=
2x − 1 4x − 3
du du
u = 7, ∴ =0 u = 3x − 1, ∴ =3
dx dx
dv dv
v = 2x − 1, ∴ =2 v = 4x − 3, ∴ =4
dx dx
du dv du dv
dy v— —
dx − u dx dy v— —
dx − u dx
= =
dx v2 dx v2
(2x − 1)(0) − (7)(2) (4x − 3)(3) − (3x − 1)(4)
= =
(2x − 1)2 (4x − 3)2
14 12x − 9 − 12x + 4
=– =
(2x − 1)2 (4x − 3)2
5
= – (4x − 3)2
5 − 2x x2 − x − 1
4 y= 5 y=
6x − 1 2x + 3
du du
u = 5 − 2x, ∴ = –2 u = x2 − x − 1, ∴ = 2x − 1
dx dx
dv dv
v = 6x − 1, ∴ =6 v = 2x + 3, ∴ =2
dx dx
du dv du dv
dy v— —
dx − u dx dy v— —
dx − u dx
= =
dx v2 dx v2
(6x − 1)(–2) − (5 − 2x)(6) (2x + 3)(2x − 1) − (x2 − x − 1)(2)
= =
(6x – 1)2 (2x + 3)2
–12x + 2 − 30 + 12x 4x − 2x + 6x − 3 − 2x2 + 2x + 2
2
= =
(6x − 1)2 (2x + 3)2
28 2x + 6x − 1
2
= – (6x – 1)2 = (2x + 3)2
28
Example 13 1 y = x2(4x − 8)
y = (x2 − 5)2
y = x4 – 3
2
3
x
y = (x2 − 5)2
= x4 − 10x2 + 25 y = x4 − 3
dy x
= 4x3 − 20x = x4 − 3x–1
dx
d2y dy
= 4x3 + 3x–2
dx2 = 12x − 20 dx
2
d2y
dx2 = 12x − 6x
2 –3
= 12x2 − 63
x
Example 14 1
1 f(x) = x2 +
Smart Tip x
4 d
f(x) = (3x − 5)2 f ″ (x) is equal to dx [𝑓′(𝑥)].
1
f(x) = x2 +
x
d 1
Solution f “(x) adalah sama dengan dx 𝑓′(𝑥). f ′(x) = 2x − 2
x
4 2
f(x) = (3x − 5)2 = 4(3x − 5)–2 f ″(x) = 2 + 3
x
f ′(x) = –2[4(3x − 5)–3](3)
= –24(3x − 5)–3
f ″(x) = –3[–24(3x – 5)–4](3)
216
= (3x − 5)4
29
Smart Tip
y
l = Tangent 1 l is a tangent to the curve y = f(x) at point P.
Tangen l ialah tangen kepada lengkung y = f(x) pada titik P.
y = f(x)
2 The gradient of the tangent can be determined by
dy
substituting the value of x1 into .
dx
Kecerunan tangen boleh ditentukan dengan menggantikan nilai x1 ke dalam
P(x1, y1) dy
.
dx
x
0
Exercise 15 Calculate the gradient of the tangent to the curve at the points given.
Hitung kecerunan
y tangen kepada lengkung pada titik-titik yang diberi.
PL 3 Apply the understanding of differentiation to perform Tangent
simple tasks.
Tangen
y = f(x)
Example 15
1
Given y = x2 + 5.
m
N
or
1
m Diberi y = x2 + 5.
m al
Given y = −x2 + 6. 2
30
x
0
Smart Tip
y 1 Normal line is a line at right angles (perpendicular) to the
Tangent tangent.
Tangen
y = f(x) Garis normal ialah garis pada sudut tegak (berserenjang) dengan tangen.
2 The normal line and the tangent line are perpendicular, hence
1
m
N
or m1m2 = –1.
m
m al
2 Garis normal dan garis tangen adalah berserenjang, maka m1m2 = –1.
P(x1, y1)
dy
3 is the gradient function of the tangent, m1, of the curve
x dx
0 f x) at point P. Therefore, the gradient function of the
f(
y = f(x)
–1
normal, m2, is dy .
Remember/ Ingat: dx
Equation of the tangent/Persamaan tangen
y − y1 = m1(x − x1) dy
ialah fungsi kecerunan tangen, m1, bagi lengkung y = f(x) pada titik P.
dx
Equation of the normal/Persamaan normal –1
y − y1 = m2(x − x1) Oleh itu, fungsi kecerunan normal, m2, ialah .
dy
dx
Exercise 16 Find the gradient of the tangent, m1, and the gradient of the normal, m2, at a given point.
Hence, find the equation of the tangent and normal of each of the following.
Cari kecerunan bagi tangen, m1, dan kecerunan bagi normal, m2, pada titik yang diberi. Seterusnya, cari
persamaan tangen dan normal bagi setiap yang berikut.
PL 3 Apply the understanding of differentiation to perform simple tasks.
Example 16
y = 2x2 − 5x + 6; (2, 4) m1 = 3 1
m2 = −
3
Solution y − y1 = m1(x − x1)
y − y1 = m2(x − x1)
y − 4 = 3(x − 2)
dy 1
= 4x − 5 y − 4 = 3x − 6 y − 4 = − (x − 2)
dx 3
= 4(2) − 5 y = 3x − 2
3(y − 4) = –(x − 2)
=3
1 3y − 12 = –x + 2
∴ m1 = 3, m2 = – 3y = –x + 14
3
1 y = 3x2 − 4x + 5; (1, 4) m1 = 2 1
m2 = –
dy 2
y − 4 = 2(x − 1)
dx = 6x − 4 y − 4 = 2x − 2 y−4=–
1
(x − 1)
= 6(1) − 4 2
y = 2x + 2 2(y − 4) = –(x − 1)
=2
1 2y − 8 = –x + 1
∴ m1 = 2, m2 = – 2y = –x + 9
2
31
2 y = x2 − 2x + 5; (–4, 3) m1 = –10 1
m2 =
dy 10
y − 3 = –10[x − (–4)]
dx = 2x − 2 y − 3 = –10x − 40 y−3=
1
[x − (–4)]
= 2(–4) − 2 10
y = –10x − 37 10(y − 3) = x + 4
= –10
1 10y − 30 = x + 4
∴ m1 = –10, m2 =
10 10y = x + 34
3 y = x3 + x − 4; (1, –2) m1 = 4 1
m2 = –
dy 4
y − (–2) = 4(x − 1)
dx = 3x + 1
2
1
y + 2 = 4x − 4 y – (–2) = – (x − 1)
= 3(1)2 + 1 4
y = 4x − 6 4(y + 2) = –(x − 1)
=4
1 4y + 8 = –x + 1
∴ m1 = 4, m2 = – 4y = –x − 7
4
12 m1 = –3 1
4 y= ; (2, 3) m2 =
x 3
dy 12 y − 3 = –3(x − 2) 1
dx = – x2 y − 3 = –3x + 6 y−3= (x − 2)
3
12 y = –3x + 9 3(y − 3) = (x − 2)
= – 2
(2) 3y − 9 = x − 2
= –3 3y = x + 7
1
∴ m1 = –3, m2 =
3
32
Example 17 3 2
1 Given the equation of a curve is y = x – 22. Find
2
Given the equation of a curve is y = 7x – x 2. 3 2
Diberi persamaan lengkung ialah y =x – 22.Cari
Find 2
dy dy
Diberi persamaan lengkung ialah y = 7x – x2. (a) at point P(4, 2),/ pada titik P(4, 2),
dx dx
Cari
dy (b) the equation of the tangent to the curve at
(a) at point P(2, 10),/ dy pada titik P(2, 10), point P,
dx dx
persamaan tangen kepada lengkung pada titik P,
(b) the equation of the tangent to the curve at (c) the equation of the normal to the curve at
point P, point P.
persamaan tangen kepada lengkung pada titik P, persamaan normal kepada lengkung pada titik P.
(c) the equation of the normal to the curve at
point P. 3
persamaan normal kepada lengkung pada titik P. (a) y = 2 x2 – 22
dy
= 3x
Solution dx
dy
(a) y = 7x – x 2 When x = 4, = 3(4)
dx
dy = 12
= 7 – 2x
dx
dy
When/Apabila x = 2, = 7 – 2(2) (b) Gradient of the tangent = 12
dx
=3 Equation of the tangent,
y – 2 = 12(x – 4)
(b) Gradient of the tangent/ Kecerunan tangen = 3
Equation of the tangent/ Persamaan tangen, y – 2 = 12x – 48
y – 10 = 3(x – 2) y = 12x – 46
y – 10 = 3x – 6
y = 3x + 4 1
(c) Gradient of the normal = –
1 12
(c) Gradient of the normal/ Kecerunan normal = – Equation of the normal,
3
Equation of the normal/ Persamaan normal, 1
y – 2 = – 12 (x – 4)
1
y – 10 = – (x – 2) 12y – 24 = –x + 4
3
3y – 30 = –x + 2 12y = –x + 28
3y = –x + 32
33
6
(a) y= x +8 (c) Gradient of the normal,
1 2
dy 6 =– =
dx
=– 2
x
dy 6
– ( )
3
2
3
When x = –2, =–
dx (–2)2
6 Equation of the normal,
=– 2
4 y – 5 = [x – (–2)]
3 3
=– 3(y – 5) = 2(x + 2)
2
3y – 15 = 2x + 4
3 3y = 2x + 19
(b) Gradient of the tangent = –
2
Equation of the tangent,
PAK-21
3
y – 5 = – [x – (–2)]
ACTIVITY
2
2(y – 5) = –3(x + 2)
2y – 10 = –3x – 6
2y = –3x + 4
Steps/ Langkah-langkah:
1 Teacher provides a set of questions involving tangent and normal to the curve on coloured cards.
Guru menyediakan beberapa set soalan melibatkan tangen dan normal kepada lengkung pada kad berwarna.
2 Students perform this activity in groups of three students. A coloured card is randomly selected for each group.
Murid melakukan aktiviti ini secara berkumpulan yang terdiri daripada tiga orang murid. Satu kad berwarna dipilih secara rawak
bagi setiap kumpulan.
3 Each group is required to answer all questions on the selected card. Write each answer on a mahjung paper.
Setiap kumpulan dikehendaki menjawab semua soalan yang terdapat pada kad yang dipilih. Tulis setiap jawapan pada kertas
mahjung.
4 The group work of each group is posted on the class’s notice board. Students are required to stand next to their
group work.
Hasil kerja setiap kumpulan ditampal pada papan kenyataan kelas. Murid-murid dikehendaki berdiri di sebelah hasil kerja
masing-masing.
5 A group is required to move to each group to evaluate the work of other groups. Once completed, other groups
need to do the same.
Satu kumpulan dikehendaki bergerak ke setiap kumpulan bagi menilai hasil kerja kumpulan yang lain. Setelah selesai, kumpulan
lain perlu melakukan langkah yang sama.
6 Teacher holds a discussion with students to enhance their understanding about solving problems involving tangent
and normal.
Guru mengadakan perbincangan dengan murid untuk menambahkan kefahaman mereka mengenai penyelesaian masalah
melibatkan tangen dan normal.
34
y
Remember/ Ingat:
dy
=0
dx The maximum and minimum points are the highest and the
lowest points at certain region of the curve respectively.
dy Titik maksimum dan titik minimum masing-masing ialah titik tertinggi
dy dy >0
>0 <0 dx dan terendah kawasan tertentu bagi lengkung itu.
dx dx
dy
=0
dx
x
0
d2y d2y
1 Second derivative method, /Kaedah terbitan kedua, dx2 .
dx2
d2y
(a) For maximum point, the value of 2 is negative.
dx
d2y
Bagi titik maksimum, nilai dx2 ialah negatif.
d2y
(b) For minimum point, the value of is positive.
dx2
d2y
Bagi titik minimum, nilai dx2 ialah positif.
Remember/ Ingat:
dy
Turning point/Titik pusingan: = 0.
dx
dy d2y
At maximum point/Pada titik maksimum: = 0, 2 < 0.
dx dx
dy d2y
At minimum point/Pada titik minimum: = 0, 2 > 0.
dx dx
35
Example 18
= 0, 6x2 − 12x − 18 = 0
dx x = –1,
PAK-21 ÷6, x2 − 2x − 3 = 0
(x + 1)(x – 3) = 0 x = –2 x=0
VIDEO
x = –1
x + 1 = 0 or/atau x − 3 = 0 (x < –1) (x > –1)
x = –1 x=3
x = –1, y = 2(–1)3 − 6(–1)2 − 18(–1) + 9 y = 6(–2)2 – 12(–2) – 18 y = 6(0)2 – 12(0) – 18
0
= 30 = –18
= 19
x = 3, y = 2(3)3 − 6(3)2 − 18(3) + 9
+ 0 –
= –45
Video
Curve/Lengkung:
Scan or visit https://youtu.be/cXxW6mHhVXY Minimum point/Titik minimum: (3, −45)
to watch a video on how to find maximum
and minimum turning points.
For educational purposes only
1 y = 6x − x2 + 7
y = 6x − x2 + 7 d2y
dx2 = –2 < 0
dy
= 6 − 2x y has a maximum value at x = 3
dx
dy
=0 y = 6(3) − (3)2 + 7
dx
= 16
6 − 2x = 0
–2x = –6 The turning point is a maximum point (3, 16)
x=3
36
1 8
y = 3 x3 − x2 − 15x + 6 y = 2x +
x
dy dy 8
= x2 − 2x − 15 =2− 2
dx dx x
At turning points, At turning points,
dy dy
=0 =0
dx dx
8
x2 − 2x − 15 = 0 2− 2 =0
x
(x + 3)(x − 5) = 0
2x2 − 8 = 0
x + 3 = 0 or x − 5 = 0
x2 − 4 = 0
x = –3 x=5
(x + 2)(x − 2) = 0
dy2
x = –2 or x = 2
dx2 = 2x − 2
d2y d2y 16
x = –3, dx2 = 2(–3) − 2 dx2 = x3
= –8 < 0 d2y 16
x = –2, dx2 =
y has a maximum value at x = –3 (–2)3
1 = –2 < 0
y = 3 (–3)3 − (–3)2 − 15(–3) + 6 y has a maximum value at x = –2
= 33 8
y = 2(–2) +
d2y (–2)
x = 5, dx2 = 2(5) − 2 = –8
= 8>0 d2y 16
x = 2, dx2 = 3
y has a minimum value at x = 5 (2)
1 = 2>0
y = 3 (5)3 − (5)2 − 15(5) + 6 y has a minimum value at x = 2
1
= –52 8
3 y = 2(2) +
(2)
Maximum point: (–3, 33) =8
1
Minimum point: (5, – 52 3 ) Maximum point: (–2, –8)
Minimum point: (2, 8)
1
4 y=5– − 4x
x
(2x − 1)(2x + 1) = 0
1 1
x = 2 or x = –
2
37
Example 19
1 A closed box has a square base of side x cm and
height y cm. Given the volume of the box is 216 cm3.
A closed cylinder with base radius r cm and height Show that its total surface area, A, is
of h cm has a volume of 27 cm3. Show that the total Sebuah kotak tertutup mempunyai tapak segi empat sama
surface area, A, is bersisi x cm dan tinggi y cm. Diberi isi padu kotak itu ialah
Sebuah silinder tertutup dengan jejari tapak r cm dan tinggi 216 cm3. Tunjukkan bahawa jumlah luas permukaan, A, ialah
h cm mempunyai isi padu 27 cm3. Tunjukkan bahawa jumlah
864
luas permukaan, A, ialah A = 2x2 +
x
54
A= 2πr2
r + Hence, find the value of x when A is minimum.
Seterusnya, cari nilai x apabila A adalah minimum.
Hence, find the value of r when A is minimum.
Seterusnya, cari nilai r apabila A adalah minimum. Volume = length (x) × length (x) × height (y)
216 = x2y
Solution
216
Volume/Isi padu = 27 cm3 y= 2
x
πr2h = 27
27 Total surface area:
h= 2 A = (x)(x) + (x)(x) + (x)(y) + (x)(y)
πr y
+ (x)(y) + (x)(y)
Total surface area:/Jumlah luas permukaan: = 2x2 + 4xy
A = 2πrh + πr2 + πr2 216 x
27 = 2x2 + 4x
x2 x
= (2πr)
πr2
+ 2πr2
864
A = 2x2 +
54πr x (shown)
= + 2πr2
πr2
dA 864
= 4x − 2
54 dx x
A= + 2πr2 (shown)
r 54 dA
r = 54r
–1
= 0,
dx
dA 54 dA 864
= – 2 + 4πr dr = (–1) 54r
–1–1
4x − 2 = 0
dr r x
= –54r –2
dA 4x3 − 864 = 0
= 0, 54
dr =– 2 864
r
–54 + 4πr = 0 x3 =
4
r2 = 216
–54 + 4πr3 = 0
27 x = 3 216
r3 = x =6
2π
dA
2
1 728
27 =4+
r = 3� dx2 x3
2π
1 728
3 =4+
r= (6)3
2π
3
= 12 (> 0)
d2A 108
= 3 + 4π A has a minimum value at x = 6.
dr2 r
108
= + 4π
3 3
3
2π
108
= + 4π
27
2π
= 12π (> 0)
3 .
A has a minimum value at r =
2π
3
3 .
A mempunyai nilai minimum pada r =
2π
3
38
r cm
h cm h cm
r cm
128
A= 2πr2
r +
(b) Hence, find the value of r when A is minimum. h cm C
Seterusnya, cari nilai r apabila A adalah minimum. 1 12 cm
2h
r cm
(a) Volume:
πr2h = 64
h =
64 12 h2
+ r2 = 122
πr2
h2 + r2 = 144
Total surface area: 4
A = 2πrh + πr2 + πr2 r2 = 144 − h
2
64 4
= (2πr) 2 + 2πr2
πr Vcylinder = πr2h
128πr
= πh 144 − h
2
=
πr2
+ 2πr2
4
128 h 3
A=
r
+ 2πr2 (shown)
= π 144h −
4
dV 3h 2
(b)
dA –128
= 2 + 4πr dh
= π 144 −
4
dr r dV
dA =0
= 0, dh
dr 3h2 = 0
–128
+ 4πr = 0
π 144 −
4
r2 3h2 = 144
–128 + 4πr3 = 0 4
32 h2 = 144 × 4
r3 = 3
π
h2 = 192
r = 3� 32 h = √192 = 8 3 cm
π
d A 256
2
d2V
dh2
= π 0 – 3h
2[ ]
= 3 + 4π
dr2 r 3h
=– π
256 2
= 3 + 4π
32
3 π
=–
3(8 3 )
2
π
256
= + 4π = 12π (. 0) = –12 3 π (, 0)
32
π Volume V is maximum when h = 8 3 cm.
A has a minimum value at r = 3 � 32
π
39
Example 20
1 Given y = x3 – 4x and the value of x changes at a rate
of 0.6 unit s–1. Find the rate of change in y when
The volume of a sphere decreases at a rate of x = 3.
5.4π cm3 s–1. Find the rate of change in the radius of Diberi y = x3 − 4x dan nilai bagi x berubah pada kadar
the sphere when the radius is 6 cm. 0.6 unit s−1. Cari kadar perubahan y apabila x = 3.
Isi padu sebuah sfera menyusut pada kadar 5.4π cm3 s−1. Cari
kadar perubahan jejari sfera itu apabila jejari ialah 6 cm.
dx
= 0.6
Solution dt
Common Error y = x3 − 4x
dV
= –5.4π WRONG CORRECT dy
dt SALAH BETUL = 3x2 − 4
4 dx
V = πr3 dV dV
3 = 5.4π, = –5.4π x = 3,
dt dt
dV dy
= 4πr2 The negative sign of −5.4π = 3(3)2 − 4 = 23
dr dx
dV means the decreasing of volume
r = 6, = 4π(6)2 of the sphere. dy dy dx
dr = ×
= 144π Tanda negatif pada −5.4π bermaksud dt dx dt
penyusutan isi padu sfera.
= (23) × (0.6)
dV dV dr
= × = 13.8 unit s–1
dt dr dt
dr Chain rule/Petua rantai
–5.4π = (144π) ×
dt dV dV dr
dr dt = dr × dt
= –0.0375 cm s–1
dt V = Volume/Isi padu
t = Time/Masa
r = Radius/Jejari
2 The radius of a circle increases at a rate of 0.3 cm s–1. 3 The diagram shows a square.
Find the rate of change of the area of the circle when Rajah menunjukkan sebuah segi empat sama.
the radius is 4 cm. Give the answer in terms of π.
Jejari sebuah bulatan menokok pada kadar 0.3 cm s−1. Cari kadar
perubahan luas bulatan apabila jejari ialah 4 cm. Beri jawapan
2x cm
dalam sebutan π.
dr
= 0.3 The area of the square decreases at a rate of
dt
A = πr2 3.2 cm2 s–1. Calculate the rate of change of the length
dA of the square when x = 5 cm.
= 2πr Luas segi empat sama itu menyusut pada kadar 3.2 cm2 s–1.
dr
Hitung kadar perubahan panjang sisi segi empat sama itu
r = 4, apabila x = 5 cm.
dA
= 2π(4) = 8π Area, A = 2x × 2x
dr
= 4x2
dA dA dr
= × dA
dt dr dt = 8x
dx
= (8π) × (0.3)
= 2.4π cm2 s–1 dA
x = 5, = 8(5) = 40
dx
dA
Given = 3.2
dt
dA dA dx
= ×
dt dx dt
dx
3.2 = 40 ×
dt
dx
= 0.08 cm s–1
dt
40
3 cm
L cm2
12 cm
h cm
h cm
(a) Express the area of water surface, L cm2,
in terms of h.
(a) Express the volume of water, V cm3, in terms Ungkapkan luas permukaan air, L cm2, dalam sebutan h.
of h. (b) It is given that the rate of change of the height
Ungkapkan isi padu air, V cm3, dalam sebutan h. of water is 0.5 cm s−1. Find the rate of
(b) It is given that the water flows out from the hole change of the area of water surface when
of the container at a constant rate of 1.8 cm3 s−1. h = 6 cm.
Find the rate of change of the height of the water Diberi bahawa kadar perubahan tinggi air ialah
level when h = 3.6 cm. 0.5 cm s−1. Cari kadar perubahan luas permukaan air
Diberi bahawa air mengalir keluar melalui lubang bekas apabila h = 6 cm.
itu pada kadar tetap 1.8 cm3 s−1. Cari kadar perubahan
ketinggian paras air apabila h = 3.6 cm. (a) Radius of hemisphere = 13 cm
Let r = radius of the water surface
Solution
26 cm
(b) Let r = radius of the water surface
Katakan r = jejari permukaan air
r 3
=
h 12 13 − h 13
h
r= r
4
1 h cm
V = × π × r2 × h
3
1 h 2 By using Pythagoras’ theorem,
= ×π×
3 4
×h( ) (13 – h)2 + r 2 = 132
1 3 132 – 26h + h2 + r 2 = 132
= πh
48 r 2 = 26h − h2
L = πr2
dV 1 2 = π(26h − h2)
(b) = πh
dh 16 = 26πh – πh2
When/Apabila h = 3.6, Smart Tip
dV 1 dL
= π(3.6)2 (b) = 26π – 2πh
dh 16 • If V decreases, then dh
dV dL
= 0.81 π is negative. When h = 6, = 26π – 2π(6) = 14π
dt dh
dV dh
Given/Diberi = –1.8 Jika V menyusut, maka Given = 0.5
dt dV dt
dV dV dh adalah negatif.
= × dt dL dL dh
dt dh dt = ×
• If V increases, then dt dh dt
dh dV dL
–1.8 = 0.81π × is positive. = (14π) × (0.5)
dt dt dt
dh –1.8 Jika V menokok, maka = 21.99 cm2 s–1
= dV
dt 0.81π adalah positif.
dt
= –0.7074 cm s–1
41
dV 1
(b) = 9 πh2
dh
dV 1
When h = 4, = π(4)2
dh 9
16
= 9π
dh
Given = 0.72
dt
dV dV dh
= ×
dt dh dt
dV 16
= 9 π × 0.72
dt
= 4.021 cm3 s–1
42
Example 22 dy
1 Given y = x3, find the value of when x = 5.
dy dy dx
Given y = x , find the value of when x = 81.
4
( )
1 1 = 125 + 3.75
= 4 27 = 128.75
1
= 108 (b) x = 5, x = 4.95 – 5 = –0.05
x3 = y
(a) 4
81.04 (x + x)3 = y + y
dy
x = 81, x = 81.04 – 81 = 0.4 [5 + (–0.05)]3 = 125 + × x
dx
x =y
4
4.95 = 125 + (75) × (–0.05)
3
x + x = y + y
4
= 125 – 3.75 = 121.25
dy
81 + 0.04 = (3) + × x
4
dx
dy
4
81.04 = (3) +
1
( )
108 × 0.04
2 Given y = x , find the value of
Diberi y =
dy
x , cari nilai
dx
when x = 16.
apabila x = 16.
= (3) + 0.00037 dx
Hence, find the approximate value of
= 3.00037
Seterusnya, cari nilai hampir bagi
(a) 16.2 , (b) 15.8 .
(b) 4
80.96
x = 81, x = 80.96 – 81 = –0.4
dy 1 – 1
1
4
x =y y = x2,
= x2
dx 1 2
x + x = y + y
4
When x = 16, y = 16 2 = 4
dy
4
81 + (–0.04) = (3) +
dx
× x dy 1 1 1 1
= (16)– 2 = 2 4 = 8
dx 2 ( )
1
4 1
( )
80.96 = (3) + 108 × (–0.04) (a) x = 16, x = 16.2 – 16 = 0.2
= (3) – 0.00037 x =y
= 2.9996 x + x = y + y
dy
16 + 0.2 = (4) + × x
dx
Smart Tip
δy dy
16.2 = (4) +
1
8 × 0.2()
δx dx = (4) + 0.025 = 4.025
δy dy × δx (b) x = 16, x = 15.8 – 16 = –0.2
dx x =y
x + x = y + y
dy
16 + (–0.2) = (4) + × x
dx
15.8 = (4) +
1
()
8 × (–0.2)
= (4) – 0.025 = 3.975
43
dy dx
Diberi y = 3 x , cari nilai apabila x = 64.
dx
Hence, find the approximate value of
Seterusnya, cari nilai hampir bagi
(a) 64.3 , (b) 63.7 .
3 3
dy 1 – 2
1
y = x3,
= x3 (b) x = 64, x = 63.7 – 64 = –0.3
dx 1 3
x =y
3
When x = 64, y = 64 3 = 4
( )
x + x = y + y
3
dy 1 2 1 1 1
= (64)– 3 = 3 16 = 48 dy
dx 3 3
64 + (–0.3) = (4) + × x
dx
(a) x = 64, x = 64.3 – 64 = 0.3
3
x =y
3
63.7 = (4) + ( )
1
48 × (–0.3)
3
x + x = y + y = (4) – 0.00625
dy = 3.994
64 + 0.3 = (4) +
3
× x
dx
3
64.3 = (4) +
1
48 × 0.3 ( )
= (4) + 0.00625
= 4.006
44
V + δV
2
= 3 π(6)3 + 1.44π
= 145.44π cm3
45
= 38.16π m3
4 A spherical air bubble is produced from the base 5 Jamal bought a roll of barbed wire of length 68 m.
of a lake. When the air bubble comes to the water He wants to use all the barbed wire to fence up a
surface, it expands. If the radius of the air bubble rectangular area as shown in the diagram below, to
increases from 4.5 cm to 4.7 cm, find the increase in plant sweet potatoes. Find the maximum area, in m2,
volume of the air bubble. of the sweet potatoes field. Applying
Suatu gelembung udara yang berbentuk sfera terhasil dari dasar Jamal membeli segulung dawai berduri yang panjangnya
sebuah tasik. Apabila gelembung udara naik ke permukaan 68 m. Dia ingin menggunakan kesemua dawai berduri itu
air, gelembung itu mengembang. Jika jejari gelembung itu untuk memagar suatu kawasan yang berbentuk segi empat tepat
bertambah daripada 4.5 cm kepada 4.7 cm, cari tokokan dalam seperti ditunjukkan dalam rajah di bawah, untuk menanam ubi
isi padu gelembung udara itu. keledek. Cari luas maksimum, dalam m2, bagi kawasan tanaman
ubi keledek itu.
Volume of air bubble, V
4 Old fence/Pagar lama
V = 3 πr3
dV Barbed wire ym
= 4πr2
dr Dawai berduri
When r = 4.5,
dV xm
= 4π(4.5)2 x + 2y = 68
dr
= 81π 1
y = 2 (68 − x)
dV
δV = × δr A = xy
dr
= (81π) × (4.7 − 4.5)
= (81π) × (0.2)
[1
= x 2 (68 − x) ]
= 16.2π cm3 1
= 34x − 2 x2
dA
= 34 − x
dx
dA
When = 0,
dx
34 − x = 0
x = 34
1
y = 2 [68 − (34)] = 17
A = (34)(17) = 578 m2
46
Paper 2
Section A
h
1 Given the curve y = x + , where h is a constant passes through point (−4, −5).
x
h
Diberi lengkung y = x + , dengan keadaan h ialah pemalar melalui titik (−4, −5).
x
(a) Find the value of h.
Cari nilai h.
[2 marks/markah]
(b) Find the coordinates of all the turning points of the curve.
Cari koordinat bagi semua titik pusingan bagi lengkung itu.
[4 marks/markah]
(c) Sketch the curve.
Lakarkan lengkung itu.
[2 marks/markah]
47
FF 2r metre/ meter
D C
Diagram 1/ Rajah 1
(a) Given the length of the fence around the lawn is 120 m. Show that the area, A m2, of the lawn is represents by
3
(
A = 120r – 2 + π r 2.
2 )
Diberi panjang pagar di sekeliling halaman rumput ialah 120 m. Tunjukkan bahawa luas, A m2, bagi halaman rumput diwakili oleh
3
(
A = 120r – 2 + π r 2.
2 )
[2 marks/markah]
(b) Find the value of r such that A is maximum.
Cari nilai r supaya A adalah maksimum.
[4 marks/markah]
(c) Find the maximum area, in m2, of the lawn.
Cari luas maksimum, dalam m2, bagi halaman rumput itu.
[Use/Guna π = 3.142]
[2 marks/markah]
21
3 Given the equation of a curve is y = 3 .
x
21
Diberi persamaan bagi suatu lengkung ialah y = .
x3
dy
(a) Find the value of when x = 2.
dx
Cari nilai dy apabila x = 2.
dx
[3 marks/markah]
21
(b) Hence, determine the approximate value of correct to two decimal places.
(1.98)3
Seterusnya, tentukan nilai hampir bagi 21 betul kepada dua tempat perpuluhan.
(1.98)3
[4 marks/markah]
5 It is given that a curve y = 2x(3 – x)4 passes through the point P(2, 5).
Diberi bahawa suatu lengkung y = 2x(3 – x)4 melalui titik P(2, 5).
Find/ Cari
(a) the gradient of the tangent to the curve at point P,
kecerunan tangen kepada lengkung pada titik P,
[4 marks/markah]
(b) the equation of the normal to the curve at point P.
persamaan normal kepada lengkung pada titik P.
[3 marks/markah]
48
r cm
15 cm
h cm
Diagram 2/ Rajah 2
(a) Express r in terms of h.
Ungkapkan r dalam sebutan h.
[2 marks/markah]
3
(b) Show that the volume, V cm3, of the water in the container is given by V = πh3.
100
3
Tunjukkan bahawa isi padu, V cm3, air di dalam bekas itu diberi oleh V = πh3.
100
[2 marks/markah]
(c) The water flows out from a small hole at the tip of the container at a constant rate of 1.8 cm3 s−1. Find the rate
of change of the height of water level when the height of water level is 6 cm.
Air mengalir keluar melalui lubang kecil di hujung bekas itu dengan kadar tetap 1.8 cm3 s−1. Cari kadar perubahan ketinggian paras
air di dalam bekas apabila ketinggian paras air ialah 6 cm.
[3 marks/markah]
Section B
7 A curve y = x3 – 6x2 + 9x – 6 passes through the point M(2, −4) and has two turning points, K(3, −6) and L. Find
Satu lengkung y = x3 – 6x2 + 9x – 6 melalui titik M(2, −4) dan mempunyai dua titik pusingan, K(3, −6) dan L. Cari
(a) the gradient of the tangent to the curve at point M,
kecerunan tangen kepada lengkung pada titik M,
[3 marks/markah]
(b) the equation of the normal to the curve at point M,
persamaan normal kepada lengkung pada titik M,
[3 marks/markah]
(c) the coordinates of L and determine whether L is a maximum point or a minimum point.
koordinat L dan tentukan sama ada L ialah titik maksimum atau titik minimum.
[4 marks/markah]
H O TS Zo n e
1
1 The daily profit, U, in RM, of a bakery is given by U = 42x − x2, where x is the number of cakes of same type
6
being sold. How many cakes must be sold to get maximum profit and what is the maximum profit of the bakery?
Applying
1
Keuntungan harian, U, dalam RM, bagi sebuah kedai kek diberi oleh U = 42x − x2, dengan keadaan x ialah bilangan kek yang sama jenis
6
dijual. Berapakah bilangan kek yang mesti dijual supaya mendapat keuntungan maksimum dan berapakah keuntungan maksimum kedai
kek tersebut?
49
3 Integration
Pengamiran
3.1 Integration as the Inverse of Differentiation / Pengamiran sebagai Songsangan Pembezaan
Smart Tip
d
Integration is a technique of finding a function, f(x) from the derivative, which is [f(x)]. This process is denoted by the sign
dx
∫ ... dx, as in ∫ f(x) dx. Integration is known as the reverse process of differentiation.
d
Pengamiran ialah suatu teknik untuk mencari suatu fungsi, f(x) daripada pembezaan, iaitu [f(x)]. Proses ini diwakilkan dengan tatatanda
dx
∫ ... dx, seperti ∫ f(x) dx. Pengamiran dikenali sebagai proses songsangan bagi pembezaan.
d
If [f(x)] = f (x), then ∫ f (x) dx = f(x)
dx
d
Jika [f(x)] = f (x), maka ∫ f (x) dx = f(x)
dx
Exercise 1 Determine the following integral based on the result of differentiation given.
Tentukan kamiran berikut berdasarkan hasil pembezaan yang diberikan.
PL 1 Demonstrate the basic knowledge of integration.
Example 1 d
1 If (7x) = 7, determine ∫ 7 dx.
d dx
(a) Given (6x) = 6, determine ∫ 6 dx. d
dx Jika (7x) = 7, tentukan ∫ 7 dx.
dx
d
Diberi (6x) = 6, tentukan ∫ 6 dx.
dx d
d (7x) = 7
(b) Given (4x2 – 3x) = 8x – 3, determine dx
dx [ ∫ 7 dx = 7x
∫ (8x – 3) dx.
d
Diberi (4x2 – 3x) = 8x – 3, tentukan ∫ (8x – 3) dx.
dx
Solution
d
(a) (6x) = 6 d
dx 2 If (4x2) = 8x, determine ∫ 8x dx.
[ ∫ 6 dx = 6x dx
d
Jika (4x2) = 8x, tentukan ∫ 8x dx.
dx
d
(b) (4x2 – 3x) = 8x – 3
dx d
(4x2) = 8x
[ ∫ (8x – 3) dx = 4x2 – 3x dx
[ ∫8x dx = 4x2
3 If
d 3
dx
(x + x2) = 3x2 + 2x, determine ∫ (3x2 + 2x) dx. 4 If ( )
d 6
dx x x
6
x
6
( )
= – 2 , determine ∫ – 2 dx.
dx ( x )
= – 6 , tentukan ∫ (– 6 ) dx.
d 3 d 6
Jika (x + x2) = 3x2 + 2x, tentukan ∫ (3x2 + 2x) dx. Jika
dx x 2
x2
d 3
dx
(x + x2) = 3x2 + 2x d 6
dx x ( ) 6
=– 2
x
[ ∫(3x2 + 2x) dx = x3 + x2
( )
6
[ ∫ – 2 dx =
x
6
x
50
Example 2
1 Given
d 1
dx 3x + 2 (
=–
3
)
(3x + 2)2
, determine
Given
d 1
dx 2x + 1(=–
2
(2x + 1)2 )
, determine
∫
12
(3x + 2)2
dx.
6
∫ (2x + 1)2 dx. Diberi d
1
dx 3x + 2(=–
3
)
(3x + 2)2
, tentukan ∫
12
(3x + 2)2
dx.
Diberi d
1
(
dx 2x + 1
=–
2
(2x + 1)2), tentukan ∫
6
(2x + 1)2
dx.
d 1
dx 3x + 2 ( =– )3
(3x + 2)2
Solution
3 1
d
( 1
dx 2x + 1
=– ) 2
(2x + 1)2
[ ∫–
(3x + 2)2
dx =
3x + 2
× (–4)
12 4
[ ∫–
2
dx =
1
× (–3) ∫ (3x + 2)2 dx = – 3x + 2
(2x + 1)2 2x + 12
6 3
∫ (2x + 1)2 dx = – 2x + 1
2 Given
d 2
(
dx 5x – 3
=–
10
(5x – 3)2 )
, determine 3 Given
d 3x
dx 4x + 3
2 (=
9 – 12x2
(4x2 + 3)2), determine
30 3 – 4x2
∫ (5x – 3)2 dx. ∫ (4x2 + 3)2
dx.
Diberi d
2
dx 5x – 3(=–
10
(5x – 3)2 )
, tentukan ∫
30
(5x – 3)2
dx. Diberi d
3x
(
dx 4x2 + 3
= )
9 – 12x2
(4x2 + 3)2
, tentukan ∫
3 – 4x2
(4x2 + 3)2
dx.
d 2
dx 5x – 3 ( =–
10
(5x – 3)2 ) d
(
3x
dx 4x + 3
2
= )9 – 12x2
(4x2 + 3)2
10 2
[ ∫– 9 – 12x 3x
( 13 )
2
dx = × (–3)
(5x – 3)2 5x – 3 [ ∫ (4x2 + 3)2
dx =
4x2 + 3
×
30 6
∫ (5x – 3)2 dx = – 5x – 3
∫ (4x2 + 3)2 dx = 3 × ( 4x2 + 3 )
3 – 4x2 1 3x
x
=
4x2 + 3
d d
4 Given (3x + 1)4 = 12(3x + 1)3, determine ∫6(3x + 1)3 dx. 5 Given (2x – 3)9 = 18(2x – 3)8, determine ∫(2x – 3)9 dx.
dx dx
d d
Diberi (3x + 1)4 = 12(3x + 1)3, tentukan ∫ 6(3x + 1)3 dx. Diberi (2x – 3)9 = 18(2x – 3)8, tentukan ∫ (2x – 3)9 dx.
dx dx
d d
(3x + 1)4 = 12(3x + 1)3 (2x – 3)9 = 18(2x – 3)8
dx dx
[ ∫ 12(3x + 1)3 dx = (3x + 1)4 × ( 12 ) [ ∫18(2x – 3)8 dx = (2x – 3)9 × ( 181 )
1
1 ∫(2x – 3)8 dx = (2x – 3)9
∫ 6(3x + 1)3 dx = 2 (3x + 1)4 18
51
Smart Tip
axn + 1
1 ∫ axn dx = + c, where a and c are constants, n is an integer and n ≠ –1.
n+1
axn + 1
∫ axn dx = + c, dengan keadaan a dan c ialah pemalar, n ialah integer dan n ≠ –1.
n+1
2 ∫ k dx = kx + c, where k and c are constants.
∫ k dx = kx + c, dengan keadaan k dan c ialah pemalar.
3 ∫ 9 dx x 5 ∫4x5 dx 6 ∫–x6 dx
4 ∫3 dx
∫ 9 dx = 9x + c x ∫4x5 dx ∫–x6 dx
∫ 3 dx1 + 1 4x5 + 1 x6 + 1
x = +c =– +c
= +c 5+1 6+1
3(1 + 1) 4x6 x7
x 2 = +c =– +c
= +c 6 6+1
6 2 x 7
= x6 + c =– +c
3 7
4 8 dx 3 10 ∫6 x dx
7 ∫ 5 x 3 dx 8 ∫ x3 9 ∫ 4x6 dx
4 8 3 ∫6 x dx
∫ 5 x 3 dx ∫ x3 dx ∫ 4x6 dx 1
3 1
( )
4 x3 + 1
= 5 3+1 +c
= ∫8x– 3 dx
8x–3 + 1
= 4 ∫ x6 dx
= ∫6x 2 dx
6x 2
1
+1
= –3 + 1 + c 3 –6
4 x4
( )
= 5 4 +c 8x–2
= 4 ∫x dx
=
1
2 +1
+c
1
= x4 + c
5
= –2 + c
4
(
3 x–6 + 1
)
= 4 –6 + 1 + c
3
6x 2
=– 2 +c = +c
3x–5 3
x =– +c
20 2
3
3 = 4x 2 + c
=– +c
20x5 = 4 x3 + c
52
Example 4
1 ∫ (3x2 – 3x + 5) dx
∫(x – 1)(x + 4) dx
∫ (3x2 – 3x + 5) dx
Solution 3x3 3x2
= – + 5x + c
∫(x – 1)(x + 4) dx 3 2
3x 2
= ∫ (x2 + 3x – 4) dx = x3 – + 5x + c
Smart Tip 2
x2 + 1 3x1 + 1
= + – 4x + c ∫[f(x) g(x)] dx
2+1 1+1
x 3
3x 2 = ∫f(x) dx ∫g(x) dx
= + – 4x + c
3 2
∫ (x + x )(x – x ) dx ∫ (x – x )
3 3 2 2
4 5 dx
∫ (x + x )(x – x ) dx ∫ (x – x )
3 3 2 2
dx
(
= ∫ x2 –
9
x2
dx ) ( 2x )(x – 2x ) dx
=∫ x–
= ∫ (x – 2 – 2 + ) dx
= ∫ (x2 – 9x–2) dx 2 4
x 2
x3 9x–1
= – +c = ∫ (x – 4 + 4x ) dx
2 –2
3 –1
x3 9 x3 4x–1
= + +c = – 4x + +c
3 x 3 –1
x 3
4
= – 4x – +c
3 x
∫ (x2 – ) dx
1 2 (x4 + 2) (x4 + 2)
6
x2
7 ∫ x4
dx
(x4 + 2) (x4 + 2)
∫( )
1
∫
2
x2 – dx dx
x2 x4
(x + 2x + 2x + 4)
( x1 )(x – x1 ) dx
8 4 4
= ∫ x2 – 2 =∫ dx
2 2 x4
= ∫ (x – 1 – 1 + ) dx
1 (x8 + 4x4 + 4)
4 =∫ dx
x 4 x4
= ∫ (x – 2 + x ) dx
4 –4 = ∫ (x4 + 4 + 4x–4) dx
x5 x–3 x5 4x–3
= – 2x + +c = + 4x + +c
5 –3 5 –3
x5 1 x 5
4
= – 2x – 3 + c = + 4x – 3 + c
5 3x 5 3x
53
Example 5
1 ∫ 3(5x + 4)3 dx
∫4(2x – 3)3 dx u = 5x + 4
du
=5
Solution dx du
Given/ Diberi ∫4(2x – 3)3 dx dx =
5
Let/ Katakan u = 2x – 3
du ∫ 3(5x + 4)3 dx Alternative Method
Thus/ Jadi, =2 Alternative Method
dx du ∫3(5x + 4)3 dx
du ∫ 4(2x – 3)3 dx = ∫ 3u3
dx = 5
= 3 × (5x + 4)
3+1
2 (2x – 3)3 + 1 3 +c
=4×
2(3 + 1)
+c = ∫ u du3 5(3 + 1)
du 5
= 3 (5x + 4)4 + c
∫ 4(2x – 3) dx = ∫ 4u
( )
3 3
2 (2x – 3)4 + c 3 u4 20
= = +c
4 3 2 5 4
= ∫ u du 3
2 = u4 + c
=2
u4
4 ( )
+c
20
3
= (5x + 4)4 + c
=2 [
(2x – 3)4
4
+c ] 20
(2x – 3)4
= +c
2
=–
u4
+c
= ( )
5 u7
4 7
+c
4 5
(4 – x)4 = (4x – 3)7 + c
=– +c 28
4
= (4 – x) = 5 × (4x – 3)
3+1 6+1
+c +c
–1(3 + 1) 4(6 + 1)
= – (4 – x) + c = 5 (4x – 3)7 + c
4
4 28
54
4∫
= u du 5
=
4 –1( )
3 u–1
+c =
3 u–2
5 –2 ( ) +c
3 3
=– +c = – u–2 + c
4u 10
3 3
=– +c =– +c
4(4x – 1) 10(5x – 1)2
4(–1) 5(–2)
= – 3 (4x – 1)–1 + c 3
= – (5x – 1)–2 + c
4 10
=– 3 +c 3
=– +c
4(4x – 1) 10(5x – 1)2
Exercise 6 Find the equation of the curve for each of the gradient function that passes through the
following points.
Cari persamaan lengkung bagi setiap fungsi kecerunan yang melalui titik yang berikut.
PL 3 Apply the understanding of integration to perform simple tasks.
Example 6 dy
1 = 8x – 1; (0, 5)
dx
dy
= (2x –1)3; (1, 0) y = ∫ (8x –1) dx
dx
8x2
Solution y= –x+c
2
y = ∫ (2x –1)3 dx y = 4x2 – x + c
(2x – 1) 3 + 1
y= +c
2(3 + 1)
When x = 0 and y = 5
(2x – 1) 4
y= +c 5 = 4(0)2 – 0 + c
8
c=5
When x = 1 and y = 0/ Apabila x = 1 dan y = 0 y = 4x2 – x + 5
[2(1) – 1]4
0= +c
8
1
0= +c
8
1
c=–
8
(2x – 1)4 1
y= –
8 8
55
dy 3 dy
4 = 2 – 1; (3, 0) 5 = (4x – 5)3; (1, 0)
dx x dx
y = ∫ (3x–2 – 1) dx y = ∫ (4x – 5)3 dx
3x –1
(4x – 5)4
y = –x+c y = +c
–1 4(4)
3 (4x – 5) 4
y = – – x + c y= +c
x 16
dy 5 dy 6
6 = ; (1, 3) 7 = ; (1, 2)
dx (4x – 3)2 dx (2x + 1)3
[ (4x4(–1)
– 3)
]+c [ (2x2(–2)
+ 1)
]+c
–1 –2
y=5 y=6
y = 5[ ]+c y = 6[ ]+c
(4x – 3) –1
(2x + 1) –2
–4 –4
5 3
y = – +c y = – +c
4(4x – 3) 2(2x + 1)2
56
Smart Tip
y
1 The definite integral of a function f(x) with respect to x for
the interval from x = a to x = b is written as:
Kamiran tentu bagi suatu fungsi f(x) terhadap x untuk batasan dari y = f(x)
x = a ke x = b ditulis sebagai:
b
a
f(x) dx = [g(x) + c] ba
= [g(b) + c] – [g(a) + c]
a x
= g(b) + c – g(a) – c O b
= g(b) – g(a)
y
2 Reversing the direction of the interval gives the negative
of the original definite integral. y = f(x)
Penukaran arah batasan akan menyebabkan negatif kepada
kamiran tentu yang asal.
b a
a
f(x) dx = – b
f(x) dx
x
O b a
a x
O b c
Example 7 4
1 2
7x dx
3
(a) –1
5x dx
2 4
(b) 1
(3x3 – x2 + 1) dx 7x dx
2
Solution =
7x2 4
2 2 [ ]
[ 5x2 ] [ ] [ ]
2 3
(a)
3
5x dx = 7(4)2 7(2)2
–1 = –
–1 2 2
=[
2 ] [ 2 ]
5(3) 5(–1)
2 2
= 56 – 14
–
= 42
= 20
2
(b) 1
(3x3 – x2 + 1) dx
[ ]
2
3x 4 x 3 2
2 2x2 dx
= – +x –1
4 3 1
= [
3(2)4 (2)3
4
–
3
+2 –
3(1)4 (1)3
4
–
3 ] [
+1 ] 2
2x2 dx
–1
=
34 17
3
–
12
=
2x 3 2
3 –1[ ]
=
119
12
=
2(2)3
3 [ ] [
–
2(–1)3
3 ]
=
16
3
– – ( )
2
3
=6
57
2
1
(3x + 2) dx 3
(6x2 – 2x + 4) dx
0
[
3 2
] = [2x3 – x2 + 4x] 30
2
= x + 2x
2 1 = [2(3)3 – (3)2 + 4(3)] – [2(0)3 – (0)2 + 4(0)]
=
2[
3 2
(2) + 2(2) –
3 2
2
(1) + 2(1) ] [ ] = 57 – 0
= 57
7
= 10 –
2
13
=
2
5
2
(2x – 1)(x2 + 2) dx
3 4x2 + 1
6 dx
0
2 x2
2
(2x – 1)(x + 2) dx 2
0 3 4x2 + 1
2
= 0 (2x3 + 4x – x 2 – 2) dx dx
2 x2
[
2x 4 4x2 x 3
]
2 3
= + – – 2x = 2 (4 + x–2) dx
4 2 3 0
[
x4 x
] [ x–1 ]
3 2 –1 3
= + 2x2 – – 2x = 4x +
2 3 0 2
[ ] [ ] = [4x – ]
(2) 4
(2) 3
(0)4 (0)3 1 3
= + 2(2)2 – – 2(2) – + 2(0)2 – – 2(0)
2 3 2 3 x 2
= [4(3) – ] – [4(2) – ]
28 1 1
= –0
3 3 2
28 25
= =
3 6
(2x – 3x ) dx 1
–1 2 1
7 8 dx
–2 –1 (2 – x)3
–1
(4x2 – 12 + 9x–2) dx 1 1
–2 dx
(2 – x)3
[ ]
–1
4x 3 9x –1 –1
= – 12x +
3 –1 –2 =
1
(2 – x)–3 dx
[ ] [ ]
–1
4(–1)3 9 4(–2)3 9
=
3
– 12(–1) –
(–1)
–
3
– 12(–2) –
(–2) =[ (2 – x)–2 1
–2(–1) –1 ]
59 107
=[ ]
= – 1 1
3 6 2(2 – x) –1
2
11 1 1
= = –
6 2(2 – 1)2 2[2 – (–1)]2
4
=
9
Calculator Corner
3
Find the value of 1
(2x4 – x2 + 3) dx./ Cari nilai bagi 31 (2x4 – x2 + 3) dx.
By using scientific calculator Casio fx-570EX./ Dengan menggunakan kalkulator saintifik Casio fx-570EX.
∫ 2 x x 4 – x x2 + 3 1 3 =
1 412
Answer/ Jawapan:
15
58
Example 8 5
1 Given 1
f(x) dx = 3, find the value of
Given
3
1
f(x) dx = 5, find the value of Diberi 5
1
f(x) dx = 3, cari nilai
1
Diberi 3
1
f(x) dx = 5, cari nilai (a) 5
[f(x) dx,
5
(a)
3
1
[f(x) + 2] dx, (b)
3
1
[4f(x) – x] dx (b) 1
[f(x) + 1] dx.
1
Solution (a) 5
f(x) dx
3
(a) 1 [f(x) + 2] dx =–
5
f(x) dx
1
3 3
= 1
f(x) dx + 1
2 dx = –3
= 5 + [2x ] 3
1
5
= 5 + [2(3) – 2(1)] (b) 1
[f(x) + 1] dx
=9 5
= 1 f(x) dx +
5
1 dx
1
3 = 3 + [x] 51
(b) 1
[4f(x) – x] dx = 3 + (5 – 1)
3 3
=4 f(x) dx – 1 x dx
1 =3+4
= 4(5) −
x2 3
2 1 [ ] =7
= 20 −
(3)2 (1)2
2
−
2 [ ]
= 20 – 4
= 16
2
2 Given
4
g(x) dx = 6, find the value of 3 Given –1
g(x) dx = 4, find the value of
1
4 2
Diberi g(x) dx = 6, cari nilai Diberi –1
g(x) dx = 4, cari nilai
1
3
[ g(x) + ] dx,
1
4 2
(a) g(x) dx, (a)
1 4 –1 6 2
[3
]
4
2
(b) g(x) + x dx. (b) [3g(x) – x2] dx.
1 2 –1
3
[ g(x) + ] dx
1
4 2
(a) g(x) dx (a)
1 4 –1 6 2
3 4 1 2 2 1
= g(x) dx = g(x) dx + dx
4 1 6 –1 –1 2
3
= (6)
4 =
1
6
(4) +
1 2
x
2 –1 [ ]
9
=
2 =
2
3
1
[1
+ (2) – (–1)
2 2 ]
2 3
= +
3
[ ] 3 2
4
(b) g(x) + x dx 13
1 2
=
3 4 4
6
= g(x) dx + 1 x dx
2 1
3
= (6) +
2
x2 4
2 1 [ ] (b)
2
–1
[3g(x) – x2] dx
=9+
(4)2 (1)2
2
–
2 [ ] =3
2
–1
g(x) dx –
2
–1
x2 dx
[ x3 ]
3
15 = 3(4) –
2
=9+
2 –1
=
33
2
= 12 –
(2)3 (–1)3
3
–
3 [ ]
= 12 – 3
=9
59
[ ]
2 x2 5 (b) –2
[f(x) + kx] dx = 16
= (9) + 4
3 2 1 f(x) dx +
1 1
kx dx = 16
–2 –2
= 6 + [2x2] 51
4+k x [ ]
2 1
= 6 + [2(5)2 – 2(1)2] = 16
2 -2
= 6 + 48
[ ]
k (1) – (–2) = 12
2 2
= 54
2 2
[ ]
k – 3 = 12
2
k = − 8
x2 3x2 + 2x d
6 Given
d
dx 3x + 1 (
= (3x + 1)2 . ) 7 Given
dx
(4x2 – 5)3 = 24x(4x2 – 5)2.
2
1 3x2 + 2x Evaluate 1 3x(4x2 – 5)2 dx.
Evaluate 2 dx.
0 (3x + 1)
Diberi d (4x2 – 5)3 = 24x(4x2 – 5)2.
Diberi d
x2
dx 3x + 1
= (
3x2 + 2x
(3x + 1)2
. )
dx
Nilaikan 21 3x(4x2 – 5)2 dx.
1 3x2 + 2x
Nilaikan 2 dx.
0 (3x + 1)
2
1
24x(4x2 – 5)2 dx = [(4x2 – 5)3] 21 (4 8)
1
0
3x2 + 2x x2 1
(3x + 1)2 dx = 3x + 1 0 [ ]
2
1
3x(4x2 – 5)2 dx
(1)2 (0)2 1
= – = [(4x2 – 5)3] 21
3(1) + 1 3(0) + 1 8
1 1 1
= = [(4(2)2 – 5)3] – [(4(1)2 – 5)3]
4 8 8
=
1 331
8
– –
1
8 ( )
1 331 1
= +
8 8
333
=
2
60
Exercise 9 Find the area of the shaded region between the curve and the x-axis.
Cari luas bagi rantau berlorek antara lengkung dengan paksi-x.
PL 3 Apply the understanding of integration to perform simple tasks.
Example 9
1 y
y y = x2 + 1
y = x2 − 3x
x x
O 3 O 2
[ ]
3 3
y dx = (x – 3x) dx 2 x3 2
0 0 = +x
3
[
x3 3x 2 3
]
0
= –
3 2 0 = [
(2)3
3
+ (2) –
(0)3
3 ] [
+ (0) ]
=
3 [
(3)3 3(3)2
−
2
–
(0)3 3(0)2
3
−
2 ] [ ] =
14
units2
3
1
= –4
2
= –4
1
2| 1
|
= 4 units2/ unit2
2
2 y 3 y
y = x2 − 1
y = (2 + x)(3 − x)
x
O 1 x
−2 O 3
[ x3 – x]
3 1 = (6 + x – x2) dx
–2
=
0
[
= 6x +
x2
–
x3 3
]
=[ – (1)] – [ ]
(1) 3
(0) 3
2 3 –2
– (0)
=–
3
2
3
[
= 6(3) +
32 33
2
–
3
– 6(–2) +
2
– ] [
(–2)2 (–2)3
3 ]
3 5
= 20 units2
Area = – | 23 | = 23 unit 2 6
61
y = (x 2)2
y = x3 − 4x2 + 3x
x
O 1 3
x
2 O 2 4
[ ]
–2
x4 4x3 3x2 3
[
x3 4x2
]
4
= – + 4x = – +
3 2 –2
4 3 2 1
[
(3) 4(3)3 3(3)2 (1)4 4(1)3 3(1)2
] [ ]
4
=
43
3 [ – 2(4)2 + 4(4) –
(–2)3
3 ] [
– 2(–2)2 + 4(–2) ] =
4
–
3
+
2
–
4
–
3
+
2
9 5
= 24 units2 =– –
4 12
8
=–
3
8
Area = – = units2
3
8
3 | |
Exercise 10 Find the area of the shaded region in each of the following diagrams.
Cari luas bagi rantau berlorek dalam setiap rajah yang berikut.
PL 3 Apply the understanding of integration to perform simple tasks.
Example 10
1 y
y
y = x3 − 4x
y = 9 − x2
x
−1 O 2
x
O 3 4
[ x4 – 2x ]
4 0
=
3
(9 – x2) dx = 2
0 –1
[
= 9x –
x3 3
3 0 ] =[
(0)
4
– 2(0) ] – [
4
(–1)
4
2
4
– 2(–1)2 ]
[ ] [ ] = 0 – (– ) =
(3)3 (0)3 7 7
= 9(3) – – 9(0) –
3 3 4 4
= 18
Area below the x-axis
Area below the x-axis/ Luas di bawah paksi-x
2
4
= 3 y dx = 0
y dx
2
=
4
(9 – x ) dx
3
2 = (x3 – 4x) dx
0
[ x3 4
] [
x4
]
2
= 9x – = – 2x2
3 3 4 0
[
= 9(4) –
(4)3
3
– 9(3) – ] [
(3)3
3 ] = [
(2)4
4
– 2(2)2 –
(0)4
4 ] [
– 2(0)2 ]
= –4 – 0
44 10
= – 18 = – = –4
3 3
7
Total area/ Jumlah luas = 18 + – | 103 | Total area = + |– 4|
4
23
64 = units2
= units2/ unit2 4
3
62
y = x (x + 3)(x − 1)
x
−3 O 1 x
O 1 2 3
= 0 – –
45
4 ( ) =6
45
= Area below the x-axis
4 3
= 2 y dx
Area below the x-axis
=
1
0
y dx
=
3 24
2 x
2 (
– 6 dx )
24
[ ]
1 3
= (x3 + 2x2 – 3x) dx
0 = – – 6x
x
[ ]
2
x4 2x3 3x2 1
=
4
+
3
–
2 0 = –
24
(3)[ – 6(3) – –
24
(2)
– 6(2) ] [ ]
=
4[
(1)4 2(1)3 3(1)2
+
3
–
2
–
(0)4
4
+
2(0)3
3
–
3(0)2
2 ] [ ] = –26 – (–24)
7 = –2
=–
12
Total area = 6 + |–2|
Total area =
45
4
+ –
7
12 | | = 8 units2
45 7
= +
4 12
71
= units2
6
4 y
Area above the x-axis
y = 8x2 − x3 − 15x 5
= 3
y dx
5
= 3
(8x2 – x3 – 15x) dx
x
[ 8x3 – x4 – 15x2 ]
O 3 5 3 4 2 5
=
3
=[
2 ] [ 3 ]
8(5) (5) 15(5)
3
8(3) (3)
4 2 3 4
15(3)2
– – – – –
Area below the x-axis 3 4 4 2
12 ( 4 )
3 125 63
= 0
y dx =– – –
3
= 0
(8x – x – 15x) dx
2 3
16
=
[ 8x3 – x4 – 15x2 ]
3 4 2 3 3
=
0
=[
8(3)
3
–
(3)
4
–
2 ] [ 3
15(3)
3
–
8(0) 4
–
(0)
4
2 3 4
–
15(0)2
2 ] Total area = – | 634 | + 163
63 16
63 = +
=– 4 3
4 1
= 21 units2
12
63
=[ + 2(0) ] – [ + 2(–4) ]
(0) 3
(–4) 2
3
2
Area above the x-axis 3 3
–4
= y dx 32
–5 =0–
–4 3
= (x + 4x) dx
2
–5
32
[ x3 + 2x ]
3 –4 =–
= 2 3
=[
(–4)
+ 2(–4) ] – [
3
(–5)
–5
+ 2(–5) ]2
3
2
Total area =
7
3
+ –
32
3 | |
3 3 7 32
32 25 = +
= – 3 3
3 3 = 13 units2
7
=
3
Smart Tip y
Area of the shaded region between a curve and the y-axis.
x = f(y)
Luas rantau berlorek antara lengkung dengan paksi-y. Area of the shaded region,
b
Area of the shaded region, Luas rantau berlorek,
a b
Luas rantau berlorek, A= x dy
a
b
A= a
x dy x
O
Exercise 11 Find the area of the shaded region between the curve and the y-axis.
Cari luas bagi rantau berlorek antara lengkung dengan paksi-y.
PL 3 Apply the understanding of integration to perform simple tasks.
Example 11 1 y
y
5
y2 = 25 − 5x
3 y2 = 9 x
1 x
x
O O
9
2
3
Given y2 = 25 – 5x
Solution 5x = 25 – y2
Given/ Diberi y2 = 9 – x 1
x = (25 – y2)
x = 9 – y2 5 2
y
x=5–
Area of the shaded region/ Luas rantau berlorek 5
3
= –2 x dy Area of the shaded region
5
=
3
(9 – y ) dy 2 = 1 x dy
–2
[ ]
y2
( )
5
y3 3 = 5− dy
= 9y – 1 5
3 –2
[ y 5
]
3
[
= 9(3) −
(3)3
3
– 9(–2) −
3 ] [
(–2)3
] = 5y −
15 1
= 18 – –
46
3 ( ) = 5(5) −[ (5)3
15
– 5(1) −
(1)3
15 ] [ ]
50 74
1 = –
= 33 units2/unit2 3 15
3
11
= 11 units2
15
64
4 y2 = x − 2
2
x
O x
O
−1
y2 = 4(x − 1)
[ y3 + 2y]
4 3 2
= 0 x dy =
4 y2
( )
–1
= + 1 dy
=[ + 2(2)] – [ ]
4 (2) 3
(–1) 3
0 + 2(–1)
[
y3
] 3 3
4
= +y
3 ( 3)
12 0 20 7
= – –
[
(4)
] [
(0)3
]
3
= + (4) – + (0)
12 12 = 9 units2
1
= 9 units2
3
Smart Tip
y
Area of the shaded region between a curve and a straight line.
y = f(x)
Luas rantau berlorek antara lengkung dengan satu garis lurus.
Area of the shaded region, y = g(x)
Luas rantau berlorek,
x
b b O
A= a
f(x) dx – a
g(x) dx a b
Area
Exercise 12 Find the area of the shaded region between the curve and the straight line. Luas ra
Cari luas bagi rantau berlorek antara lengkung dengan garis lurus.
Example 12
y
y = x + 2
4
y = x2 + 4
2 Alternative Method
x Area of the shaded region
2 O 2 Luas rantau berlorek
2 1
Solution = y dx –
(2)(2)
0 2
Area of the shaded region 2
= (–x2 + 4) dx – 2
0
Luas rantau berlorek
[
x3
]
2
=
2
[(–x2 + 4) – (–x + 2)] dx = – + 4x – 2
0 3 0
=
2
(–x2 + x + 2) dx
0 = – [
(2)3
3
+ 4(2) – 0 – 2 ]
[x3 x2
]
2
= – + + 2x 1
3 2 0 = 3 units2/ unit2
3
= – [
(2)3 (2)2
3
+
2
+ 2(2) – –
(0)3 (0)2
3
+
2 ] [
+ 2(0) ]
1
= 3 units2/ unit2
3
65
3
y= 3
y = x2 + 3
x
O 1
y = −x2 + x + 3
x
O
Area of the shaded region
1
= 0 [(–x2 + x + 3) – 3] dx y = 4x 1
1 y=x +3
2
2
= (–x2 + x) dx
0 Subsitute 1 into 2
= – [
x3 x2 1
3
+
2 0 ] 4x = x2 + 3
x – 4x + 3 = 0
2
= – [
(1)3 (1)2
3
+
2
– –
(0)3 (0)2
3
+
2 ] [ ] (x – 1)(x – 3) = 0
1 x = 1 or x = 3
= unit 2
6
Area of the shaded region
3
= 1 [(4x) – (x2 + 3)] dx
3
= (–x2 + 4x – 3) dx
1
[
x3
]
3
= – + 2x2 – 3x
3 1
= – [
(3)3
3
+ 2(3)2 – 3(3) – –
(1)3
3 ] [
+ 2(1)2 – 3(1) ]
=0– –
4
3 ( )
Alternative Method
4
Alternative Method = units2 When x = 1, y = 4(1) = 4
3
Area of the shaded region When x = 3, y 4(3) = 12
1
= 0
y dx – 3(1) Area of the shaded region
1
= 0
(–x2 + x + 3) dx – 3 = 1 (4 + 12) × 2 – 31 y dx
2
[ x3 x2
]
1
= – + + 3x – 3 = 16 – 31 (x2 + 3) dx
3 2
[ ]
0
= 16 – x + 3x
3 3
=[ ] [ ]
(1)3 (1)2 (0)3 (0)2
– + + 3(1) – – + + 3(0) – 3 3 1
3 2 3 2
[ ] [
= 16 – (3) + 3(3) – (1) + 3(1) ]
3 3
19 3 3
= –0–3
=
1
6
unit2
(
= 16 – 18 – 10
3
)
6 = 4 units2
3
3 y
y= x + 4
Area of the shaded region
4 Alternative Method
= 1 [(6x – x2) – (x + 4)] dx
4
When x = 1, y = (1) + 4 = 5
= (–x2 + 5x – 4) dx
1 When x = 4, y = (4) + 4 = 8
y = 6x −x2
[
x 3 5x 2
]
4
Area of the shaded region
= – + – 4x
x 3 2 1 4 1
= y dx – (5 + 8) × 3
O
= – [
(4)3 5(4)2
3
+
2 ]
– 4(4) – 4
1 2
= 1 (6x –x2) dx – 19.5
y = 6x – x
[ ]
2
1 (1) 3
5(1) 2
y=x+4 2
–
3
+
2
– 4(1)
[
= 3x2 –
x3 4
3 1 ]
– 19.5
Subsitute 1 into 2
6x – x2 = x + 4
8
= – –
3 ( )11
6 [
= 3(4)2 –
(4)
3
3
] [
– 3(1)2–
(1)3
3
– 19.5 ]
x – 5x + 4 = 0
(
80 8
)
2
= 4.5 units2 = – – 19.5
(x – 1)(x – 4) = 0 3 3
x = 1 or x = 4 = 4.5 units2
66
Given y2 = 5x
y2 Given 2x = (y – 2)2
x= 5 (y – 2)2
x= 2
5
2x = (y 2)2
Area = 10 6 4
Area = +
5
5 4 2
(2, 4)
5
= 5(10) – x dy
–5 1 4
5 y 2 = (2)(2) + 2 x dy
= 50 – dy 2
–5 5
4 (y – 2)2
= 50 – [ ] y2+ 1 5
5(2 + 1) –5
=2+
2
1 4 2
2
dy
= 50 – [ ]
y3 5 =2+ (y – 4y + 4) dy
2 2
15 –5
[
1 y3
]
4
= 50 – [ ]
(5)3 (–5)3 =2+ – 2y2 + 4y
– 2 3 2
15 15
= 50 –
50 =2+ [(
1 (4)3
2 3 ) (
– 2(4)2 + 4(4) –
(2)3
3
– 2(2)2 + 4(2) )]
3
=2+ ( – )
1 1 16 8
= 33 units2 2 3 3
3
=2+ ( )
1 8
2 3
4
=2+
3
1
=3 units2
3
Smart Tip
The volume of the solid generated when the region bounded by the curve y = f(x),
f x), the x-axis, the line x = a and the line
f(
x = b is revolved through 360˚ about the x-axis is
Isi padu bongkah janaan apabila suatu rantau yang dibatasi oleh lengkung y = f(x), paksi-x, garis x = a dan garis x = b dikisarkan melalui 360˚ pada
paksi x ialah
y y = f(x)
b
x Vx = π a y2 dx
O a b
67
Example 13 1
y
y
y = 4 − x2
4
y= x
x
−2 −1 O 2
x
O 1 3
2
Vx = π y2 dx
Solution –1
2
3
Vx = π 1 y2 dx =π –1
(4 – x2)(4 – x2) dx
()
2
3 4 2 =π (16 – 8x2 + x4) dx
=π dx –1
[ ]
1 x
= π 16x – 8x + x
3 5 2
=π
3 16
( ) dx 3 5 –1
[( ) ( )]
1 x2
= π 16(2) – 8(2) + (2) – 16(–1) – 8(–1) + (–1)
3 5 3 5
[ ]
= π – 16
3
3 5 3 5
[ ( )]
x 1 256 203
=π – –
[ ]
16 3
= –π 15 15
x 1 3
= 30 π units3
[
= –π 16 – 16
(3) (1) ] 5
[ ]
= –π – 32
3
32
= π units3/ unit3
3
2 3
y y
y = x2 − 5x 1
y= x − x
3 x
x O 1 5
−1 O 5
5
Vx = π
3
y2 dx Vx = π 1 y2 dx
–1
(x – 1x ) dx
3 5 2
=π –1
(x2 – 5x)2 dx =π
1
3
= π (x – 2 + 1 ) dx
=π –1
(x4 – 10x3 + 25x2) dx 5
2
[ ] x 2
= π x – 10x + 25x
5 4 3 3 1
= π[ – 2x – 1 ]
5 4 3 –1 x 3 5
3 x
[( ) (
1
= π (3) – 10(3) + 25(3) – (–1) – 10(–1) +
5 4 3 5 4
68
x = f(y)
b
Vy = π a
x2 dy
b
a
x
O
Exercise 14 Find the volume generated for each of the following diagrams when the shaded region is
revolved through 360° about the y-axis.
Cari isi padu janaan bagi setiap rajah yang berikut apabila rantau berlorek diputarkan melalui 360˚ pada paksi-y.
PL 3 Apply the understanding of integration to perform simple tasks.
Example 14
1 y
y2 = 8x
y
x = (y − 3)2
x
O
3
−2
x −4
O
Solution Given y2 = 8x
x= y
3 2
Vy = π 0 x2 dy
8
3
=π (y – 3)4 dy x2 = y
4
0
64
[ ]
= π (y – 3)
4+1 3
(1)(4 + 1) 0 –2
Vy = π x2 dy
[ ]
–4
= π (y – 3)
5 3
(64y ) dx
–2 4
5 0 =π
[ ]
–4
= π ((3) – 3) – ((0) – 3)
5 5
= π[ y ]
5 –2
5 5
5(64)
[ ( )]
–4
243
= π[ – (–4) ]
=π 0– – (–2) 5 5
5
243 320 320
= π[–
320 ( 320 )]
= π units3 32
5 – – 1 024
= 3.1π units3
69
x
O
−1
x
O
5 2
Vy = π 0 x2 dy Given y = x – 2
= π 0 (y2 – 5y)2 dy
5
2
x = y + 2
5
= π 0 (y4 – 10y3 + 25y2) dy 4
x2 = (y + 2)2
[
= π y – 10y + 25y ]
5 4 3 5
5 4 3 0 2
[ ]
Vy = π x2 dy
= π (5) – 10(5) + 25(5) – 0
5 4 3
–1
5 4 3 42
= π 2 dy
1 –1 (y + 2)
= 104 π units 3
6 [
4 2
= π – y + 2
–1
]
4 2
= –π y + 2 [–1
]
= –π 4
[ – 4
(2) + 2 (–1) + 2 ]
= –π (1 – 4)
= 3π units3
4 y 5
y
y= x + 1
2
6 y = 6 − x2
3
2
x
O
x
O
2 ) (
– 6(2) – (2) )]
2 2 (6) 2 2
= π 12 –
3
2 ( ) = π (18 – 10)
2
= 10.5π units3
= 8π units 3
70
Example 15 1
y
y x= 2
x = 4y − y2 12
y= x
y= 2
x x
O 3 O
Solution
x=3 1 x=2 1
12
x = 4y – y2 2 y= 2
x
Substitute 1 2
into / Gantikan 1 ke dalam 2 Substitute 1 into 2
3 = 4y – y2 12
y – 4y + 3 = 0
y= =6
2
2
(y – 1)(y – 3) = 0
2 units
y = 1 or/ atau y = 3 6
PAK-21
Volume = 4 units
3 units/ unit
ACTIVITY
3 2
Volume = 2 units/ unit
Isi padu 1
6
3
Vy = π 2 x2 dy – πr2h
Vy = π x2 dy – πr2h
( 12y ) dy – (π × 2 × 4)
1 6 2
=π
3
(4y – y ) dy – (π × 3 × 2)
2 2 2 =π 2
1 2
= π ( 144 ) dy – 16π
3 6
=π 1
(16y2 – 8y3 + y4) dy – 18π
2 y
2
[
= π 16y – 8y + y ]
3 4 5 3
[
= π 16y – 2y4 + y ]
3 5 3
=π 153
5( – 53
15
– 18π)
= 406 π – 18π
15
136
= π units3/ unit3
15
Steps/ Langkah-langkah:
1 Students are divided into groups of four.
Murid-murid dibahagikan kepada kumpulan yang terdiri daripada empat orang.
2 Each group is required to construct a question related to the volume of a region revolved at the x-axis or the
y-axis. Then, each group is required to trade the question with other groups.
Setiap kumpulan dikehendaki membina satu soalan mengenai isi padu suatu rantau yang dikisarkan pada paksi-x atau paksi-y.
Kemudiam, setiap kumpulan dikehendaki menukar soalan dengan kumpulan yang lain.
3 Teacher holds a discussion with students.
Guru mengadakan perbincangan bersama-sama murid.
71
5
1
x
O
2 x+y=8
x
O
x + y = 8 4 units
5
x = 8 – y
x2 = (8 – y)2 Volume = 4 units
x2 = 64 – 16y + y2 1
5
Vy = π 1 x2 dy – πr2h
1 unit 5
5 = π 1 (25 + y2) dy – (π × 42 × 4)
Volume =
[ ] – 64π
3 units
= π 25y + y
3 5
2 3 1
3 3
= π(
3 )
5
Vy = π 2 x2 dy – πr2h 500 76
– – 64π
5 3
= π 2 (64 – 16y + y2) dy – (π × 12 × 3)
= 424 π – 64π
[
= π 64y – 8y + y ] – 3π
3 5
2 3
3 2 232
= π units3
= π[(64(5) – 8(5) + (5) ) – (64(2) – 8(2) + (2) )] – 3π
3 3
2 2 3
3 3
= π(
3 )
485 296
– – 3π
3
= 63π – 3π
= 60π units3
4 y x=3
5 y
y = x2 + 1
y = x2 − 9 y = 2x + 1
y=6
Q(2, 5)
x P
O x
O
When x = 0, y = 2(0) + 1 = 1
3 units [ P = (0, 1)
6
Volume = 6 units 2 units
5
0
Volume = 4 units
6
Vy = π 0 x2 dy – πr2h 1
6
= π 0 (y + 9) dy – (π × 32 × 6)
Vy = π 1 x2 dy – 1 πr2h
5
( )
[
= π y + 9y – 54π ] 3
2 6
2 0 = π 1 (y – 1) dy – 1 π × 22 × 4
5
3 ( )
(6)
[(
+ 9(6) – (0) + 9(0) – 54π ) ( )]
2 2
[ ]
= π y 2 5
16
2 2 = π –y – π
= π(72 – 0) – 54π 2 1 3
[( ) (
= π (5) – (5) – (1) – (1) – 16 π )]
2 2
= 72π – 54π
= 18π units3 2 2 3
2[ ( )]
= π 15 – – 1 – 16 π
2 3
16
= 8π – π
3
= 8 π units3
3
72
1 Daud is painting a wall at a rate of (25 – t) m2 per 2 Dewi invests an amount of RM5 000. The expected
hour, where t is the number of hours after he started rate of increase in the investment, RMv, is given by
painting. Given the area of the wall is 132 m2, how dv
2 = 5y + 1 200, where y is the number of years of
long will it take for him to finish painting the wall in dy
a day? investment.
Daud mengecat sebuah dinding dengan kadar (25 – t) m2 per Dewi melabur sebanyak RM5 000. Kadar pertambahan
jam, dengan keadaan t ialah bilangan jam selepas dia mula dv
nilai pelaburan, RMv, diberi oleh 2 = 5y + 1 200, dengan
mengecat. Diberi luas dinding itu ialah 132 m2, berapa lamakah dy
masa yang diperlukan untuk dia habis mengecat dinding itu keadaan y ialah bilangan tahun melabur.
dalam sehari? 5y2
(a) Show that v = + 600y + 5 000.
4
dA 5y
2
Given = 25 – t such that A = area of the wall. Tunjukkan bahawa v = + 600 y + 5 000.
dt 4
(b) Find the minimum number of years required for
Time taken, j, to paint 132 m2 wall is
j the total amount of investment to be at least three
(25 – t) dt = 132
0 times the initial amount of investment.
[ ] [ ]
1 1 nilai pelaburan awal.
25j − j2 – 25(0) − (0)2 = 132
2 2
1 dv
25j − j2 = 132 (a) 2 = 5y + 1 200
2 dy
1 2 dv 5y
j – 25j + 132 = 0 = + 600
2 2 dy 2
j – 50j + 264 = 0
5y2
(j – 6)(j – 44) = 0 v = + 600y + c
4
j – 6 = 0 or j – 44 = 0 When y = 0, v = 5 000
j = 6 j = 44
5(0)2
5 000 = + 600(0) + c
Since j < 24, thus j = 6 hours. 4
c = 5 000
5y2
[ v= + 600y + 5 000
4
(b) Total amount of investment triple
= 5 000 × 3
= 15 000
5y2
+ 600y + 5 000 = 15 000
4
5y2
+ 600y – 10 000 = 0
4
5y + 2 400y – 40 000 = 0
2
–b b2 – 4ac
y =
2a
(2 400)2 – 4(5)(–40 000)
y = –(2 400)
2(5)
y = 16.125 or y = −496
73
P(5, 1)
x
O
x
O Diagram 3/ Rajah 3
Diagram 1/ Rajah 1 Find the volume generated, in terms of π, when
5
the shaded region is revolved through 360°
It is given that 0 g(x) dx = 9, find the area, in about the x-axis.
units2, of the shaded region. Cari isi padu janaan, dalam sebutan π, apabila rantau
5
Diberi bahawa 0 g(x) dx = 9, cari luas, dalam unit2, bagi berlorek itu diputarkan melalui 360° pada paksi-x.
rantau berlorek. [3 marks/markah]
[2 marks/markah]
(b) The graph of y = h(x) passes through the point 5 Diagram 4 shows a part of the curve y = x2 + 1 intersects
d with the straight line x + y = 7 at point F.
(2, −16). Given [h(x)] = 3x2 + x – 26, find h(x) in
dx Rajah 4 menunjukkan sebahagian daripada lengkung y = x2 + 1
terms of x. bersilang dengan garis lurus x + y = 7 pada titik F.
Graf y = h(x) melalui titik (2, −16). Diberi y
d
[h(x)] = 3x2 + x – 26, cari h(x) dalam sebutan x. y = x2 + 1
dx
[3 marks/markah]
F
3 Diagram 2 shows a curve y = f(x). x + y= 7
Rajah 2 menunjukkan lengkung y = f(x).
E
y x
O G
y = f(x) Diagram 4/ Rajah 4
Find/ Cari
(a) the coordinates of point F,
koordinat titik F,
x
4 O 1 6 [2 marks/markah]
(b) the area of the shaded region.
Diagram 2/ Rajah 2
luas bagi rantau berlorek.
The area of the shaded region is given by [3 marks/markah]
n 13
f(x) dx = .
m 4
n 13
Luas bagi rantau berlorek diberi oleh f(x) dx = .
m 4
74
y
y = h(x) y= 7
y = 8x − x2
y= 5 x
O
x
O Diagram 6/ Rajah 6
Diagram 5/ Rajah 5
Find the area, in units2, of the shaded region.
Given the gradient function of the curve is Cari luas, dalam unit2, bagi rantau berlorek.
dy [4 marks/markah]
= 2x – 4, find the equation of the curve.
dx
Diberi fungsi kecerunan bagi lengkung itu ialah
dy
= 2x – 4, cari persamaan bagi lengkung itu.
dx
[4 marks/markah]
1 Given
Paper 2
Section A
1 Diagram 1 shows a part of the curve y = 6x – x2 and the straight line 2x + y = 16.
Rajah 1 menunjukkan sebahagian daripada lengkung y = 6x – x2 dan garis lurus 2x + y = 16.
y 2x + y = 16
K
y = 6x − x2
x
O 2 4
Diagram 1/ Rajah 1
(a) Find the coordinates of point K and of point L,
Cari koordinat titik K dan titik L,
[3 marks/markah]
(b) Calculate the area, in units2, of the shaded region.
Hitung luas, dalam unit2, bagi rantau yang berlorek.
[4 marks/markah]
Section B
2 Diagram 2 shows the curve y = 16 − x2 and the straight line FH is a tangent to the curve at point G.
Rajah 2 menunjukkan lengkung y = 16 − x2 dan garis lurus FH ialah tangen kepada lengkung itu pada titik G.
y
G
y = 16 x2
p
x
O 9 H
Diagram 2/ Rajah 2
Given the gradient of the straight line FH is −2. Find
Diberi kecerunan garis lurus FH ialah −2. Cari
(a) the coordinates of point G,
koordinat titik G,
[2 marks/markah]
75
D
y= 7 − x
E(5, 2) x = y2 + 1
2
x
O F(m, 0) 5
Diagram 3/ Rajah 3
Find/ Cari
(a) the value of m,
nilai m,
[2 marks/markah]
(b) the area, in units2, of the shaded region,
luas, dalam unit2, bagi rantau yang berlorek,
[5 marks/markah]
(c) the volume generated, in terms of π, when the region bounded by the curve x = y2 + 1, the straight line x = 5 and
the x-axis is revolved through 360° about the x-axis.
isi padu janaan, dalam sebutan π, apabila rantau yang dibatasi oleh lengkung x = y2 + 1, garis lurus x = 5 dan paksi-x diputarkan
melalui 360° pada paksi-x.
[3 marks/markah]
H O TS Zo n e
1 Diagram 1 shows a circle with centre O and radius r units.
Rajah 1 menunjukkan sebuah bulatan berpusat O dan berjejari r unit.
y
x
O
Diagram 1/ Rajah 1
(a) Show that the equation of the circle is x + y2 = r2. 2
Cari isi padu janaan, dalam sebutan π dan r, apabila rantau berlorek diputarkan melalui 360° pada paksi-x.
76
P(0, h)
x
O Q(r, 0)
Diagram 2/ Rajah 2
(a) Express the equation of PQ in the form of x = my + c.
Ungkapkan persamaan PQ dalam bentuk x = my + c.
(b) Calculate the volume generated, in terms of π, r and h, when the shaded region is revolved through 360° about
the y-axis. Applying
Hitung isi padu janaan, dalam sebutan π, r dan h, apabila rantau berlorek diputarkan melalui 360° pada paksi-y.
3 Diagram 3 shows a shaded region bounded by the curve y = x2 + 2, the straight line y = k and the y-axis.
Rajah 3 menunjukkan rantau berlorek yang dibatasi oleh lengkung y = x2 + 2, garis lurus y = k dan paksi-y.
y
y = x2 + 2
y= k
x
O
Diagram 3/ Rajah 3
Given the volume generated when the shaded region is revolved through 360° about the y-axis is 18π units3.
Find the value of k. Applying
Diberi isi padu janaan apabila rantau berlorek diputarkan melalui 360° pada paksi-y ialah 18π unit3. Cari nilai k.
77
Smart Tip
Multiplication rule is defined as the number of ways for two or more events can occur. If a certain event can occur in m
ways and another event can occur in n ways, then the number of ways for both events can occur is m × n ways.
Petua pendaraban didefinisikan sebagai bilangan cara bagi dua atau lebih peristiwa boleh berlaku. Jika suatu peristiwa boleh berlaku dalam m cara dan
suatu peristiwa lain boleh berlaku dalam n cara, maka bilangan cara kedua-dua peristiwa boleh berlaku ialah m × n cara.
Example 1
1 In a chess competition, team X has 10 players and
An undergraduate from Jengka wishes to go back team Y has 6 players. Find the number of ways a
to his hometown at Kota Kinabalu. There are 5 bus single game can be held.
Dalam suatu pertandingan catur, pasukan X mempunyai
companies that provide transport from Jengka to
10 orang pemain dan pasukan Y mempunyai 6 orang pemain.
Kuala Lumpur and 3 airlines that provide flight
Cari bilangan cara perlawanan individu boleh diadakan.
from Kuala Lumpur to Kota Kinabalu. Find the
number of ways he can go home.
Seorang mahasiswa dari Jengka ingin pulang ke kampungnya di Number of ways = 10 × 6 = 60
Kota Kinabalu. Terdapat 5 buah syarikat bas yang menyediakan
perjalanan dari Jengka ke Kuala Lumpur dan 3 buah syarikat
penerbangan yang menyediakan penerbangan dari Kuala
Lumpur ke Kota Kinabalu. Cari bilangan cara dia boleh pulang
ke kampungnya.
Solution
Number of ways/ Bilangan cara
=5×3
= 15
2 If a man has 4 trousers and 3 shirts, how many ways 3 How many ways to form a pair of mixed double
can he wear the trousers and the shirts? player from 5 female players and 7 male players?
Jika seorang lelaki mempunyai 4 helai seluar panjang dan 3 helai Berapakah bilangan cara untuk membentuk sepasang pemain
baju, berapakah bilangan cara dia boleh memakai seluar panjang beregu campuran daripada 5 orang pemain perempuan dan 7
dan baju itu? orang pemain lelaki?
78
A B C D
Number of ways = 5 × 4 × 7 = 140
VIDEO
PAK-21
Smart Tip
ACTIVITY
1 The number of permutations of n different objects is given by n!, such that
Bilangan pilih atur bagi n objek yang berbeza diberi oleh n!, dengan keadaan
n! = n × (n – 1) × (n – 2) ... × 3 × 2 × 1
2 n! is read as n factorial.
n! dibaca sebagai n faktorial.
3 0! = 1.
Find the value of 8!./ Cari nilai bagi 8!. Steps/ Langkah-langkah:
1 Students are divided into groups of four.
By using scientific calculator Casio fx-570EX. Murid-murid dibahagikan kepada kumpulan yang terdiri daripada
Dengan menggunakan kalkulator saintifik Casio
empat orang.
fx-570EX.
2 Teacher gives a word to each group.
Step 1: Press ‘MENU’ and find 1: Calculate Guru memberikan satu perkaatan kepada setiap kumpulan.
(normal display), press ‘ = ’. Example/ Contoh : “CUTE”
Langkah 1: Tekan ‘MENU’ dan cari 1: Calculate
3 Each group is required to arrange all the letters from the
(paparan normal), tekan ‘ = ’.
word without repetition.
Step 2: Key in the values and press ‘ = ’ to get
Setiap kumpulan dikehendaki menyusun semua huruf dalam
the answer.
perkataan itu tanpa ulangan huruf.
Langkah 2: Masukkan nilai dan tekan ‘ = ’ untuk
mendapatkan jawapan. 4 In the group, the first student write the first arrangement of
–1 the letters in a paper. Then, the first student passes the
8 SHIFT x =
paper to the other students in the group in clockwise rotation
Answer/ Jawapan: 40 320 so that each member of the group can write the answers.
Dalam kumpulan, murid pertama menulis susunan huruf di atas
kertas. Kemudian, murid pertama mengedarkan kertas kepada
murid seterusnya dalam kumpulan mengikut susunan jam supaya
setiap ahli kumpulan dapat menulis jawapan.
79
Example 2 1 2! 4! 2 5! 0!
8!
(a) 3! (b) 6!
2! 4! 5! 0!
Solution = (2 × 1) × (4 × 3 × 2 × 1) = (5 × 4 × 3 × 2 × 1) × 1
8! 8 × 7 × 6! = 2 × 24 = 120
(a) 3! = 3 × 2 × 1 = 6 (b) 6! = 6! = 48
=8×7
= 56
6! 8! 7! 2! 9! 5!
3 4 5 6
3! 6! 2! 5! 7! 4!
6! 6 × 5 × 4 × 3! 8! 8 × 7 × 6!
3! = 3! 6! 2! = 6! × 2 × 1 7! 2! 9! 5!
=6×5×4 56 5! 7! 4!
= 120 = 2 7 × 6 × 5! × 2 × 1 9 × 8 × 7! × 5 × 4!
= =
= 28 5! 7! × 4!
=7×6×2×1 =9×8×5
= 84 = 360
3 Find the number ways to form 4-digit numbers 4 Find the number of different ways to arrange all the
using the digits 1, 3, 4 and 5 if no digit can be letters in the word ‘FLOWER’.
repeated. Cari bilangan cara berlainan menyusun semua huruf daripada
Cari bilangan cara membentuk nombor 4 digit menggunakan perkataan ‘FLOWER’.
digit 1, 3, 4 dan 5 jika digit-digit itu tidak berulang.
n=6
n=4 Number of ways = 6!
Number of ways = 4! = 720
= 24
80
Calculator Corner
7 7
Find the value of P4./ Cari nilai P4.
VIDEO
For educational purposes only
7 SHIFT × 4 =
81
6 6! 5 5!
P3 = P2 =
(6 – 3)! (5 – 2)!
= 120 = 20
Smart Tip
The number of permutation of n objects involving identical objects is given by
Bilangan pilih atur bagi n objek yang melibatkan objek secaman diberi oleh
n!
P=
a! b! c! ...
where a, b and c, … is the number of objects for each identical object.
dengan keadaan a, b dan c, … ialah bilangan objek bagi setiap objek secaman.
Exercise 5 Find the number of ways to arrange all the letters from the following words.
Cari bilangan cara menyusun semua huruf daripada perkataan yang berikut.
PL 2 Demonstrate the understanding of permutation and combination.
82
Example 6
1 Find the number of ways 5-digit numbers can be
Find the number of ways all the letters from the formed from the digits below if the digits 5 and 7
word ‘FORMULA’ can be arranged with the are always separated and no digits can be repeated.
Cari bilangan cara nombor 5-digit yang boleh dibentuk
condition that the arrangements start with a vowel.
daripada digit-digit di bawah jika digit 5 dan digit 7 sentiasa
Cari bilangan cara semua huruf daripada perkataan
berasingan dan tiada digit yang berulang.
‘FORMULA’ boleh disusun dengan syarat susunan bermula
dengan huruf vokal. 4 5 6 7 8
Solution Number of ways when digits 5 and 7 are always
The vowels are O, U, A together
Huruf vokal ialah O, U, A = 4! × 2
O = 48
6 × 5 × 4 × 3 × 2 × 1 = 6! Number of ways when digits 5 and 7 are always
U separated
= 5! – 48
6 × 5 × 4 × 3 × 2 × 1 = 6! = 120 – 48
A = 72
6 × 5 × 4 × 3 × 2 × 1 = 6!
Alternative Method
Number of ways
Bilangan cara
3 6
= P1 × P6
= 2 160
2 There are 6 books on a bookshelf. What is the 3 Find the number of ways 4-digit even numbers
number of ways to arrange the books if two of the can be formed from the digits 3, 4, 5 and 7 without
bestselling books must be arranged side by side? repetition.
Terdapat 6 buah buku di atas rak buku. Berapakah bilangan Cari bilangan cara nombor genap 4 digit yang boleh dibentuk
cara untuk menyusun buku-buku itu jika dua buah buku yang daripada digit-digit 3, 4, 5 dan 7 tanpa ulangan.
terlaris mesti disusun bersebelahan?
Even number must end with digit 4.
5! × 2! = 120 × 2 Number of ways for three digits before digit 4
= 240
4
3×2×1=6
83
5 × 4 × 3 × 2 × 1 = 120 7
3×2×1=6
Number of ways for 5 characters with the first letter
‘Q’ Number of ways for three digits with the last
Q digit ‘9’
5 × 4 × 3 × 2 × 1 = 120 9
3×2×1=6
Number of ways
= 120 + 120
Number of ways
= 240
=6+6+6
= 18
Example 7 Example 8
The diagram shows eight letter cards. Find the number of ways to arrange eight
Rajah menunjukkan lapan kad huruf. shareholders to sit at a round table in a meeting.
Cari bilangan cara menyusun lapan orang pemegang
F E S T I V A L saham untuk duduk di sebuah meja bulat dalam suatu
Find/ Cari mesyuarat.
(a) the number of ways to arrange four letter cards
from all the cards, Solution
bilangan cara menyusun 4 kad huruf daripada semua kad itu, Number of ways
(b) the number of arrangements if the arrangement Bilangan cara
of the 4 letter cards begins with a vowel. = (8 – 1)!
bilangan susunan jika susunan 4 kad huruf itu bermula = 7!
dengan huruf vokal. = 5 040
Solution
8
(a) P4 = 1 680
(b) Vowel/Huruf vokal: E, I, A
1 7
E = P1 × P3 = 210
1 7
Smart Tip
I = P1 × P3 = 210
1 7 Number of permutations of n different
A = P1 × P3 = 210 objects in a circle arrangement = (n – 1)!
Number of arrangements/Bilangan susunan Bilangan pilih atur bagi n objek yang berbeza
dalam susunan bulatan = (n – 1)!
= 210 + 210 + 210
= 630
84
G G B B B B B
Number of ways = 2! × 5! × 6 F F F M M M M
= 1 440 Number of ways = 3! × 4! × 5
= 720
3 Ali has 4 watermelons, 5 papayas and 2 apples. Find 4 4 rattan chairs and 5 wooden chairs are required
the number of ways if he wants to arrange all the to be arranged in a row. Find the number of
fruits in a row and the fruits of the same kind must arrangements that can be formed if 3 rattan chairs
be placed together. must be arranged together.
Ali mempunyai 4 biji tembikai, 5 biji betik dan 2 biji epal. Cari 4 buah kerusi rotan dan 5 buah kerusi kayu perlu disusun
bilangan cara jika dia ingin menyusun semua buah itu sebaris sebaris. Cari bilangan susunan jika 3 buah kerusi rotan mesti
dan buah yang sama jenis diletakkan bersebelahan. disusun bersebelahan.
= 17 280
5 Find the number of ways to arrange five students to 6 What is the number of ways to arrange seven
sit at a round table for the group work. teachers to sit at a round table in a meeting?
Cari bilangan cara menyusun lima orang murid untuk duduk Berapakah bilangan cara menyusun tujuh orang guru untuk
di sebuah meja bulat untuk kerja berkumpulan. duduk di sebuah meja bulat dalam suatu mesyuarat?
Smart Tip
1 A combination is a selection of several objects from a set such that the arrangement of the objects is not taken into
consideration. The number of combinations of r objects chosen from n different objects at a time is given by
Gabungan ialah pemilihan beberapa objek daripada suatu set dengan keadaan susunan objek itu tidak diambil kira. Bilangan gabungan bagi r objek
yang dipilih daripada n objek yang berbeza pada satu masa diberi oleh
n n!
Cr =
r!(n – r)!
n n
2 The relationship between permutation, Pr and combination, Cr is
n n
Hubungan antara pilih atur, Pr dan gabungan, Cr ialah
n n
Cr × r! = Pr
85
8 SHIFT 3 =
Answer/ Jawapan: 56
Situation Answer
Situasi Jawapan
86
2 There are 10 deer in a park. If 4 deer are to be given 3 There are 13 people in a room. If each person is
away to a zoo, how many ways are there to select asked to shake hand with each other exactly once,
the deer? how many handshakes would there be?
Terdapat 10 ekor rusa di sebuah taman. Jika 4 ekor rusa akan Terdapat 13 orang di dalam sebuah bilik. Jika setiap orang
diberikan ke sebuah zoo, berapakah bilangan cara pemilihan diminta untuk bersalam antara satu sama lain sekali sahaja,
rusa itu boleh dilakukan? berapakah bilangan cara bersalaman yang dapat dilakukan?
87
Number of ways
7
= C3
7!
= 3!(7 – 3)!
7!
= 3! 4!
= 35
2 There are 10 scouts participating in a camp. 3 Johan wants to mix 5 different watercolours to
Determine the number of ways produce new colours. Find the number of new
Terdapat 10 orang pengakap yang menyertai satu perkhemahan. colours that can be produced.
Tentukan bilangan cara Johan ingin mencampurkan 5 warna air berbeza untuk
(a) 5 scouts are chosen to cook, menghasilkan warna baharu. Cari bilangan warna baharu yang
5 orang pengakap dipilih untuk memasak, mungkin yang boleh dihasilkan.
(b) not more than 4 scouts are chosen to set up a
tent. The colours that can be mixed:
tidak lebih daripada 4 orang pengakap dipilih untuk 2 from 5 or 3 from 5 or 4 from 5 or 5 from 5
memasang khemah.
The total number of new colours
10 5 5 5 5
(a) C5 = 252 ways = C2 + C3 + C4 + C5
(b)
10 10 10
C1 + C2 + C3 + C4
10 = 10 + 10 + 5 + 1
= 26
= 10 + 45 + 120 + 210
= 385 ways
88
Review 4
Paper 1
9 8 7 6 4 M N P C O M B I N E
Diagram 2/ Rajah 2
Diagram 1/ Rajah 1
A code of 5 characters is to be formed using Find the number of ways to arrange
Cari bilangan cara menyusun
all the numbers and letters without repetition.
(a) four letters from all the cards,
The code must consists of 3 numbers followed
empat huruf daripada semua kad itu,
by 2 letters. How many different codes can be
(b) if the arrangements in (a) begin with a consonant.
formed? jika susunan di (a) bermula dengan huruf konsonan.
[5 marks/markah]
89
Section B
5 (a) Saiful has seven marbles of different colours. He wants to arrange all the marbles in a row. Find the number of
ways the marbles can be arranged if the white marble is not placed next to the red marble.
Saiful mempunyai tujuh biji guli yang berlainan warna. Dia ingin menyusun semua guli itu sebaris. Cari bilangan cara guli itu boleh
disusun jika guli putih tidak diletak bersebelahan dengan guli merah.
[4 marks/markah]
(b) Diagram 4 shows seven cards.
Rajah 4 menunjukkan tujuh keping kad.
3 4 5 6 7 X Y
Diagram 4/ Rajah 4
A code that consists of numbers and letters is to be formed using five cards. How many different codes can be
formed if each code must contain at least 3 numbers that are arranged side by side.
Satu kod yang terdiri daripada angka dan huruf akan dibentuk menggunakan lima keping kad. Berapakah bilangan kod berlainan yang
dapat dibentuk jika setiap kod mesti mengandungi sekurang-kurangnya 3 angka yang disusun bersebelahan.
[4 marks/markah]
H O TS Zo n e
1 Find the number of arrangements of all the letters from the word ‘REMAINS’ if the vowels are always placed in
the odd places. Applying
Cari bilangan susunan bagi semua huruf daripada perkataan ‘REMAINS’ jika huruf vokal sentiasa berada pada kedudukan ganjil.
2 Diagram 1 shows eight points on the straight line AB and four points on the straight line CD.
Rajah 1 menunjukkan lapan titik pada garis lurus AB dan empat titik pada garis lurus CD.
A B
C D
Diagram 1/ Rajah 1
If a triangle can be formed by joining three points from the two straight lines, calculate the number of triangles
that can be formed. Applying
Jika sebuah segi tiga boleh dibentuk dengan menyambungkan tiga titik daripada dua garis lurus itu, hitung bilangan segi tiga yang dapat
dibentuk.
90
5 Probability Distribution
Taburan Kebarangkalian
5.1 Random Variable/ Pemboleh Ubah Rawak
Smart Tip
A random variable refers to a variable with numerical values as a result of a random phenomenon. The random variable
is denoted by X.
Pemboleh ubah rawak merujuk kepada suatu pemboleh ubah dengan nilai berangka hasil daripada suatu fenomena rawak. Pemboleh ubah rawak
diwakili oleh X.
Exercise 1 List all the values of the random variable for each of the following situations.
Senaraikan semua nilai pemboleh ubah rawak bagi setiap situasi yang berikut.
PL 2 Demonstrate the understanding of probability distribution.
2 A chess player plays five games. X represents the 3 A sample of eight bulbs are selected from a bulb
number of times he wins. factory. X represents the number of defected bulbs.
Seorang pemain catur bermain sebanyak lima permainan. Satu sampel 8 biji mentol dipilih daripada sebuah kilang mentol.
X mewakili bilangan kali dia menang. X mewakili bilangan mentol yang rosak.
X = {0, 1, 2, 3, 4, 5} X = {0, 1, 2, 3, 4, 5, 6, 7, 8}
Smart Tip
1 A discrete random variable has countable values of the random variable.
Pemboleh ubah rawak diskret mempunyai nilai pemboleh ubah yang boleh dibilang.
2 A continous random variable has values of the random variable in a certain interval.
Pemboleh ubah rawak selanjar mempunyai nilai pemboleh ubah yang berada dalam suatu selang tertentu.
91
Event Answer
Peristiwa Jawapan
Smart Tip
1 If X represents a discrete random variable with the values of r1, r2, ..., rn then, the probability for each value is
P(X = r1), P(X = r2),… , P(X = rn ).
Jika X mewakili pemboleh ubah rawak diskret dengan nilai r1, r2, ..., rn maka, kebarangkalian bagi setiap nilai ialah P(X = r1), P(X = r2),…,
P(X = rn).
n
2 P(X = ri) = 1 where each P(X = ri) > 0.
i=1
n
i=1
P(X = ri) = 1 dengan keadaan setiap P(X = ri) > 0.
92
Example 2
1 There are three blue balls and two green balls in a
bag. Two balls are drawn at random from the bag
A coins is tossed three times. X represents the
and X represents the number of times of getting
number of times of getting tails.
Sekeping duit syiling dilambungkan sebanyak tiga kali. X
blue ball.
mewakili bilangan kali mendapat angka. Terdapat tiga biji bola biru dan dua biji bola hijau di dalam
sebuah beg. Dua biji bola diambil secara rawak dari beg itu dan
Solution X mewakili bilangan kali mengambil bola biru.
X = {0, 1, 2, 3 }
Let H is getting heads and T is getting tails. X = {0, 1, 2}
Katakan H ialah mendapat gambar dan T ialah mendapat Let B is getting blue ball and G is getting green ball.
angka.
First Second Outcome X=r
drawn drawn
First Second Third Outcome X=r
toss toss toss Kesudahan B BB 2
Lambungan Lambungan Lambungan B BB 2
pertama kedua ketiga
B
H HHH 0 G BG 1
H G BG 1
T HHT 1
H G GG 0
H HTH 1
T B GB 1
T HTT 2 G
H THH 1 B GB 1
H
T THT 2 B GB 1
T
H TTH 2
T
T TTT 3 X = {BB, BB, BG, BG, GG, GB, GB, GB}
2 A fair dice is tossed three times. X represents the number of times of getting number 4.
Sebiji dadu adil dilambungkan sebanyak tiga kali. X mewakili bilangan kali mendapat nombor 4.
X = {0, 1, 2, 3}
Let 4 is getting the number ‘4’ and 4’ is not getting the number ‘4’.
93
Example 3
The diagram shows a spinning wheel that is divided into four sectors of equal size. One sector is in yellow and
three sectors are in green.
Rajah menunjukkan sebuah roda putar yang dibahagikan kepada empat sektor yang sama besar. Satu sektor berwarna kuning dan tiga
sektor berwarna hijau.
Green
Hijau
Green Yellow
Hijau Kuning
Green
Hijau
The wheel is spun three times. Given X is the number of times of getting yellow sector. Complete the table below.
Hence, draw the graph of probability distribution of X.
Roda itu diputarkan sebanyak tiga kali. Diberi X ialah bilangan kali mendapat sektor berwarna kuning. Lengkapkan jadual di bawah.
Seterusnya, lukis graf taburan kebarangkalian bagi X.
X=r 0 1 2 3
P(X = r)
Solution
Let Y is getting yellow sector and G is getting green sector.
Katakan Y ialah mendapat sektor kuning dan G ialah mendapat sektor hijau.
P(X = r)
First spin Second spin Third spin Outcome Probability X=r
Putaran Putaran Putaran Kesudahan Kebarangkalian
pertama kedua ketiga
1 1
0.4
4 Y YYY 64 3
1 Y
4 3 3
G YYG 64 2
4
Y 0.3
1 3
3 4 Y YGY 64 2
1
4
4 G
3 9
G YGG 64 1
4
0.2
1 3
4 Y GYY 64 2
3 1 Y
4 4 3 9
G GYG 64 1
4 0.1
G
1 9
3 4 Y GGY 64 1
4 G
3 27
G GGG 64 0
4 0 r
0 1 2 3
P(X = 0) = P(GGG)
27
=
64
= 0.4219
P(X = 1) = P(YGG) + P(GYG) + P(GGY) P(X = 3) = P(YYY)
9 9 9 1
= + + =
64 64 64 64
27 = 0.0156
=
64
= 0.4219 X=r 0 1 2 3
P(X = 2) = P(YYG) + P(YGY) + P(GYY) P(X = r) 0.4219 0.4219 0.1406 0.0156
3 3 3
= + +
64 64 64
9
=
64
= 0.1406
94
Red
Merah
The wheel is spun three times and X represents the number of times of obtaining red sector. Complete the table
below. Hence, draw the graph of probability distribution of X.
Roda itu diputarkan sebanyak tiga kali dan X mewakili bilangan kali mendapat sektor berwarna merah. Lengkapkan jadual di bawah.
Seterusnya, lukis graf taburan kebarangkalian bagi X.
X=r 0 1 2 3
P(X = r)
1 0.1
P(X = 0) = P(BBB) = 27
= 0.0370
2 2 2
P(X = 1) = P(RBB) + P(BRB) + P(BBR) = 27 + 27 + 27 r
0
6 0 1 2 3
= 27
= 0.2222
4 4 4
P(X = 2) = P(RRB) + P(RBR) + P(BRR) =
27 + 27 + 27
12
= 27
= 0.4444
8
P(X = 3) = P(RRR) = 27
= 0.2963
X=r 0 1 2 3
95
X=r 0 1 2
P(X = r)
1
P(X = 0) = P(TT) = 4 0.50
= 0.25
1 1
P(X = 1) = P(HT) + P(TH) = 4 + 4
1 0.25
=2
= 0.5
1
P(X = 2) = P(HH) = 4
= 0.25 0 r
0 1 2
X=r 0 1 2
Smart Tip
1 An experiment that only has two outcomes, either ‘success’ or ‘failure’ is known as a Bernoulli trial. If a Bernoulli trial is
repeated for n times, it is known as the binomial experiment.
Suatu eksperimen yang mempunyai dua kesudahan sahaja, iaitu ‘kejayaan’ atau ‘kegagalan’ dikenali sebagai percubaan Bernoulli. Jika percubaan
Bernoulli dilakukan sebanyak n kali, ia dikenali sebagai ekperimen binomial.
2 A binomial random variable is the number of success, r from n Bernoulli trials. The probability distribution for binomial
random variable is known as binomial distribution.
Pemboleh ubah rawak binomial ialah bilangan kejayaan, r daripada n percubaan Bernoulli. Taburan kebarangkalian bagi pemboleh ubah rawak
binomial dikenali sebagai taburan binomial.
3 A binomial random variable X in n trials, where the probability of ‘success’ is p can be written as:
Pemboleh ubah rawak binomial X dalam n percubaan, dengan keadaan kebarangkalian ‘kejayaan’ ialah p boleh ditulis sebagai:
X ~ B(n, p)
4 The probability of obtaining r successes in a binomial distribution is given by
Kebarangkalian memperoleh r kejayaan bagi suatu taburan binomial diberi oleh
X = r) = nCrpr qn – r, r = 1, 2, 3,…, n
P(X
where/dengan keadaan
n = number of trials/bilangan percubaan
p = probability of success/kebarangkalian kejayaan
q = probability of failure/kebarangkalian kegagalan
96
2 It is found that the probability Intan wins in a story 3 A coin is tossed 7 times, find the probability that
2 Sekeping duit syiling dilambungkan sebanyak 7 kali, cari
telling competition is . If she takes part in
5 kebarangkalian bahawa
7 competitions, find the probability that (a) not obtaining tails,
Didapati bahawa kebarangkalian Intan memenangi dalam tidak mendapat angka,
2
suatu pertandingan bercerita ialah . Jika dia mengambil
5
(b) obtaining tails three times.
bahagian dalam 7 pertandingan, cari kebarangkalian bahawa tiga kali mendapat angka.
(a) she wins exactly 4 times,
dia menang tepat 4 kali. n = 7, p =0.5, q = 0.5
(b) she loses less than twice. X = Number of times obtaining tails
dia kalah kurang daripada dua kali. X ~ B(7, 0.5)
( 25 ) ( 35 ) = 0.1935
4 3
(a) P(X = 4) = 7C4
( )( )
2 6 3 1 7 2
( ) ( 35 )
7 0
= 7C6 + C7
5 5 5
= 0.0172 + 0.001638
= 0.01884
97
(a) X = {0, 1, 2, 3, 4, 5} 2 2 4 16
(b) P(X = 0) = 5C0(0.2)0(0.8)5 = 0.3277 (b)
P(X = 1) = 5C1(0.2)1(0.8)4 = 0.4096
P(X = 2) = 5C2(0.2)2(0.8)3 = 0.2048 X=r 0 1 2 3 4
P(X = 3) = 5C3(0.2)3(0.8)2 = 0.0512
1 1 3 1 1
P(X = 4) = 5C4(0.2)4(0.8)1 = 0.0064 P(X = r)
16 4 8 4 16
P(X = 5) = 5C5(0.2)5(0.8)0 = 0.0003
(c)
P(X = r) P(X = r)
0.5 3
8
5
16
0.4 1
4
3
16
0.3 1
8
1
16
0.2
0 r
0 1 2 3 4
0.1
0 r
0 1 2 3 4 5
98
Find/ Cari
( 107 ) ( 103 ) = 0.0102
1 5
6
P(X = 1) = C1 (a) P(X 2),
(b) the value of k./ nilai k.
P(X = 2) = C ( ) ( ) = 0.0595
76 3 2 4
10 10 2
(a) P(X 2) = P(X = 0) + P(X = 1) + P(X = 2)
P(X = 3) = C ( ) ( ) = 0.1852
76 3 3 3
1 7 10
10 10 3 = 20 + 20 + 20
P(X = 4) = C ( ) ( ) = 0.3241
76 3 4 2
9
10 10 4 = 10
P(X = 5) = C ( ) ( ) = 0.3025
76 3 5 1
10 10 5 10 7 1
(b) k = 1 − 20 − 20 − 20
P(X = 6) = C ( ) ( ) = 0.1176
76 3 6 0
2
10 10 6
= 20
1
= 10
P(X = r)
0.4
0.3
0.2
0.1
0 r
0 1 2 3 4 5 6
Smart Tip
Mean, = np Variance, s2 = npq Standard deviation, s = npq
Min = np Varians s 2 = npq Sisihan piawai, s = npq
99
Example 6
1 X is a discrete random variable such that
X is a discrete random variable such that X ~ B(n, p). X ~ B(300, 0.25). Find the mean and the standard
If the mean and the standard deviation of X are 120 deviation of X.
X ialah pemboleh ubah rawak diskret dengan keadaan
and 2 5 respectively, find the value of n.
X ialah pemboleh ubah rawak diskret dengan keadaan X ~ B(300, 0.25). Cari min dan sisihan piawai bagi X.
X ~ B(n, p). Jika min dan sisihan piawai bagi X masing-masing
ialah 120 dan 2 5 , cari nilai n. n = 300, p = 0.25, q = 1 – 0.25 = 0.75
Mean = np
Solution = (300)(0.25)
Mean/Min = np = 120 = 75
Standard deviation = npq
Standard deviation/ Sisihan piawai = npq
= (300)(0.25)(0.75)
2 5 = 120q = 7.5
20 = 120q
1
q=
6
1 5
Thus/ Maka, p =1− 6 = 6
5
( )
n 6 = 120
n = 144
2 A discrete random variable X has a binomial 3 A discrete random variable X ~ B(n, p) has a mean
distribution, that is X ~ B(600, p). It is given that the of 60 and a standard deviation of 3 5 . Find the
standard deviation is 12 and p , 0.5. Determine the value of n.
value of p. Suatu pemboleh ubah rawak diskret X ~ B(n, p) mempunyai
Suatu pemboleh ubah rawak diskret X bertaburan binomial, min 60 dan sisihan piawai 3 5 . Cari nilai n.
iaitu X ~ B(600, p). Diberi bahawa sisihan piawai ialah 12 dan
p , 0.5. Tentukan nilai p. Mean = np = 60
Standard deviation = npq
n = 600, Given p and q = 1 – p
3 5 = 60q
Standard deviation = npq
(3 5 )2 = 60q
12 = npq
45 = 60q
12 = (600)(p)(1 – p) 3
600p(1 − p) = 122 q=
4
600p(1 − p) = 144 ( 24) 3 1
25p(1 − p) = 6 Therefore, p = 1 − =
4 4
25p – 25p2 = 6
25p2 – 25p + 6 = 0
n ( )
1
4
= 60
n = 240
(5p − 2)(5p – 3) = 0
2 3
p = or p =
5 5 2
Given p < 0.5, [ p =
5
100
Example 7
Solution
(a) Let X = Number of participants who failed to proceed to the final
Katakan X = Bilangan peserta yang gagal ke peringkat akhir
X ~B 4, ( 37 )
P(X 2) = P(X = 0) + P(X = 1) + P(X = 2)
( )( )
3 0 4 4 4 3 1 4 3 4 3
( )( ) ( ) ( 47 )
2 2
= 4C0 + C1 + C2
7 7 7 7 7
= 0.1066 + 0.3199 + 0.3599
= 0.7864
(b) X ~B 15, ( 3
7 ) 3
Mean/Min = np =15 × = 6.429
7
Standard deviation/ Sisihan piawai = npq
3 4
= 15 × ×
7 7
= 1.917
1 It is found that 80% of the candidates who took the culinary test will pass the test.
Didapati bahawa 80% daripada calon yang mengambil ujian memasak akan lulus ujian itu.
(a) In a sample that consists of 8 candidates, find the probability that at least 3 candidates passed the test.
Dalam suatu sampel yang terdiri daripada 8 orang calon, cari kebarangkalian bahawa sekurang-kurangnya 3 orang calon lulus
ujian itu.
(b) If there are 200 candidates who sat for the test on a certain day, determine the mean and the standard
deviation of the number of candidates who passed the test.
Jika terdapat 200 orang calon yang menduduki ujian itu pada suatu hari tertentu, tentukan min dan sisihan piawai bagi bilangan
calon yang lulus ujian itu.
(b) μ = np
= (200)(0.8)
= 160
s = npq
= 200(0.8)(0.2)
= 32
= 5.657
101
2
(a) n = 8, p =
= 0.4, q = 1 – 0.4 = 0.6
5
Let X = Number of students who are in boarding school
P(X > 2) = 1− [P(X = 0) + P(X = 1)]
= 1 − [8C0(0.4)0(0.6)8 + 8C1(0.4)1(0.6)7]
= 1 − [0.01680 + 0.08958 ]
= 0.8936
(b) μ = np
= (3 600)(0.4)
= 1 440
s = npq
= (3 600)(0.4)(0.6)
= 864
= 29.39
3 In a basketball game, it is found that Johan attains 40% of success of the total trials he did.
Dalam suatu permainan bola keranjang, didapati bahawa Johan mendapat 40% kejayaan daripada jumlah percubaan yang dia lakukan.
Calculate/ Hitung
(a) the probability that Johan attains exactly 5 successes from 6 trials,
kebarangkalian bahawa Johan mendapat tepat 5 kejayaan daripada 6 percubaan,
(b) the number of trials he has to make so that the probability of success is more than 0.99.
bilangan percubaan yang perlu dia lakukan supaya kebarangkalian kejayaan lebih daripada 0.99.
102
(a) Mean = np = 35
Standard deviation = 2 7
np(1 – p) = 2 7
np(1 – p) = (2 7 )2
35(1 – p) = 28
28
1 – p = 35
4
1 – p = 5
1
p = 5 or 0.2
np = 35
n(0.2) = 35
n = 175
103
Smart Tip
1 A normal distribution has continuous random variables. The normal distribution, X can be written as X ~ N(, s2),
where is mean and s2 is variance.
Suatu taburan normal mempunyai pemboleh ubah rawak selanjar. Taburan normal, X boleh ditulis sebagai X ~ N(, s2), dengan keadaan ialah
min and s2 ialah varians.
2 The probability of X from X = a to X = b is written as P(a , X , b). The area under a graph represents the probability of X.
Kebarangkalian bagi X dari X = a hingga X = b ditulis sebagai P(a , X , b). Luas di bawah graf mewakili kebarangkalian bagi X.
ff(
f(x)
x)
x =
P(a , X , b)
0 x
a b
3 The total area under a graph which is equal to the total probability of all the values of X is 1.
Jumlah luas di bawah graf yang bersamaan dengan jumlah kebarangkalian bagi semua nilai X ialah 1.
X–
4 A continous random variable X of a normal distribution can be converted to the standard score, Z, which is Z = .
X–. s
Suatu pemboleh ubah rawak selanjar X bagi taburan nomal boleh ditukarkan kepada skor piawai, Z, iaitu Z =
s
Exercise 9 Find the area of each of the shaded regions based on the given diagram.
Cari luas bagi setiap rantau berlorek berdasarkan rajah yang diberikan.
PL 2 Demonstrate the understanding of probability distribution.
Example 8
1 f(x)
f(x)
0.132
0.526
0 x
u
0 x
−c c (a) f(x)
(a) f(x)
0 x
u
0 x
c (b) f(x)
(b) f(x)
0 x
u
0 x
c
(a) P( , X , u) = 0.5 – 0.132 = 0.368
Solution (b) P(X , u) = 1 – 0.132 = 0.868
1 – 0.526
(a) P(X . c) = = 0.237
2
(b) P(X , c) = 1 – 0.237 = 0.763
104
0.864
0.158
0 x 0 x
a −m m
0 x 0 x
−a m
0 x 0 x
−a −m
Exercise 10 Determine the probability of each of the following by using the standard normal distribution
table.
Tentukan kebarangkalian bagi setiap yang berikut dengan menggunakan sifir taburan normal piawai.
PL 2 Demonstrate the understanding of probability distribution.
Example 9
1 P(Z . 0.812)
P(–0.7 , Z , 1.7) f(z)
Solution
f(z)
Area = 0.2085
Area A Area B z
Luas A Luas B 0 0.812
= 0.2420 = 0.0446
P(Z . 0.812) = 0.2085
z
−0.7 0 1.7
P(–0.7 , Z , 1.7)
= 1−Area A – Area B
1 – Luas A – Luas B
= 1 − 0.2420 − 0.0446
= 0.7134
105
Area Area
= 0.1131 = = 0.1131
Area = 0.2236
z z
1.21 0 0 1.21 z
0 0.76
Area
= 0.00539 Area = 0.1151 Area = 0.2119
z z
0 2.55 1.2 0 0.8
P(0 , Z , 2.55) = 0.5 − P(Z . 2.55) P(–1.2 , Z , 0.8) = 1 – P(Z , −1.2) – P(Z . 0.8)
= 0.5 − 0.00539 = 1 – P(Z . 1.2) – P(Z . 0.8)
= 0.4946 = 1 – 0.1151 – 0.2119
= 0.6730
=
z z z
0 0.3 2.2 1.4 0.6 0 0 0.6 1.4
f(z) f(z) f(z) f(z)
Area A Area B
= = 0.3821 Area A Area B
= 0.0139
= 0.2743 = 0.0808
z z =
z z
0 0.3 0 2.2 0 0.6 0 1.4
106
Example 10
1 P(Z . k) = 0.0548 2 P(Z . k) = 0.3156
P(k , Z , 1.07) = 0.6281
P(Z . k) = 0.0548 P(Z . k) = 0.3156
k = 1.6 k = 0.48
Solution
P(k , Z , 1.07) = 0.6281
P(Z . k) – P(Z . 1.07) = 0.6281
P(Z . k) – 0.1423 = 0.6281
P(Z . k) = 0.7704
For/ Untuk k , 0,
1 – P(Z , k) = 0.7704
P(Z , k) = 0.2296
k = − 0.74
Smart Tip
(
P(X . k)= P Z . k –
s )
where = mean and s = standard deviation.
dengan keadaan = min dan s = sisihan piawai.
107
2 The masses of durians in an orchard are normally 3 Given the lengths of chillies in a farm are normally
distributed with a mean of 2.8 kg and a variance of distributed with a mean of 8 cm and a standard
2
0.8 kg . Find the probability that a durian chosen deviation of 2.5 cm. If a chilli is randomly selected,
randomly will have a mass more than 3 kg. determine the probability that the length of chilli is
Jisim durian di sebuah dusun bertaburan normal dengan more than 7 cm.
min 2.8 kg dan varians 0.8 kg2. Cari kebarangkalian bahawa Diberi panjang cili di sebuah kebun bertaburan normal dengan
sebiji durian yang dipilih secara rawak mempunyai jisim lebih min 8 cm dan sisihan piawai 2.5 cm. Jika sebiji cili dipilih
daripada 3 kg. secara rawak, tentukan kebarangkalian bahawa panjang cili itu
lebih daripada 7 cm.
= 2.8, s2 = 0.8, s = 0.8 = 0.8944
X = The masses of the durians = 8, s = 2.5
X ~ N(2.8, 0.89442) X = The lengths of the chillies
P(X . 3) = P Z . ( 3 – 2.8
) X ~ N(8, 2.52)
0.8944
= P(Z . 0.2236) P(X . 7) = P Z . (
7–8
2.5 )
= 0.4115 = P(Z . –0.4)
= 1 – P(Z . 0.4)
= 1 – 0.3446
= 0.6554
108
= P(0.16 , Z , 2.66)
= P(Z . 0.16) – P(Z . 2.66)
(b) P(25 , X , 35) = P ( 25 –8 30 , Z , 35 –8 30 )
= 0.4364 – 0.00391 = P(–0.625 , Z , 0.625)
= 0.4325 = 1 – P(Z . 0.625) – P(Z , –0.625)
= 1 – 0.2660 – 0.2660
= 0.468
n(A)
P(A) =
n(S)
x
0.468 =
500
x = 234
[ 234 grade B rambutans
109
= 60, s = 10
P(45 , X , 75)
=P (
45 – 60
10
,Z,
75 – 60
10 ) −1.5 1.5
z
= P(–1.5 , Z , 1.5)
= 1 − P(Z , −1.5) − P(Z . 1.5)
= 1 − 0.0668 − 0.0668
= 0.8664
Review 5
Paper 1
u
0 x
0 1 2 3 4 G
0 x
Diagram 1/ Rajah 1 14 19
Diagram 2/ Rajah 2
110
Section B
5 (a) A discrete random variable X has a binomial distribution, which is X ~ (12, p). If the mean is 3, find
Suatu pemboleh ubah rawak diskret X mempunyai taburan binomial, iaitu X ~ (12, p). Jika min ialah 3, cari
(i) the value of p,
nilai p,
(ii) P(X 3).
[4 marks/markah]
(b) The diameters of oranges in an orchard are normally distributed with a mean of 6.4 cm and a standard
deviation of 2 cm. Calculate
Diameter buah oren di sebuah kebun bertaburan normal dengan min 6.4 cm dan sisihan piawai 2 cm. Hitung
(i) the probability that an orange chosen at random from the orchard has a diameter more than 7.5 cm,
kebarangkalian bahawa sebiji buah oren yang dipilih secara rawak dari kebun itu mempunyai diameter lebih daripada 7.5 cm,
(ii) the value of h if 40% of the oranges have diameters more than h cm.
nilai h jika 40% daripada buah oren itu mempunyai diameter lebih daripada h cm.
[4 marks/markah]
Paper 2
Section A
1 The diameters of concrete cylindrical pillars produced in a factory are normally distributed with a mean of 40 cm and a
standard deviation of s cm. It is given that the probability of a pillar selected at random from the factory has diameter
more than 43.6 cm is 0.0548.
Diameter tiang silinder konkrit yang dihasilkan di sebuah kilang bertaburan normal dengan min 40 cm dan sisihan piawai s cm. Diberi
bahawa kebarangkalian sebatang tiang yang dipilih secara rawak dari kilang itu mempunyai diameter lebih daripada 43.6 cm ialah 0.0548.
Find/ Cari
(a) the value of s,
nilai s,
[2 marks/markah]
(b) P(40 , X , 44),
[3 marks/markah]
(c) the value of k such that P(X . k) = 0.8085.
nilai k dengan keadaan P(X . k) = 0.8085.
[3 marks/markah]
111
Section B
3 (a) In a survey conducted in a state, it is found that 60% of the graduates are unemployed. If 12 graduates from the
state is selected at random, find the probability that
Dalam satu kajian yang dijalankan di sebuah negeri, didapati bahawa 60% daripada siswazah menganggur. Jika 12 orang siswazah
dari negeri itu dipilih secara rawak, cari kebarangkalian bahawa
(i) exactly 8 graduates are unemployed,
tepat 8 orang siswazah menganggur,
(ii) at least 10 graduates are unemployed.
sekurang-kurangnya 10 orang siswazah menganggur.
[5 marks/markah]
(b) There are 400 students who participate in a cross-country running. The time taken to complete the run is
normally distributed with a mean of 50 minutes and a standard deviation of 15 minutes. The participants that
complete the run more than 60 minutes will not be given a medal.
Seramai 400 orang murid menyertai larian merentas desa. Masa yang diambil untuk menamatkan larian itu bertaburan normal
dengan min 50 minit dan sisihan piawai 15 minit. Peserta yang menamatkan larian lebih daripada 60 minit tidak akan diberikan
pingat.
(i) If a participant is chosen at random, find the probability that the participant does not get the medal.
Jika seorang peserta dipilih secara rawak, cari kebarangkalian bahawa peserta itu tidak mendapat pingat.
(ii) The medals are only awarded to the first 30 participants that complete the run. If Johan completed his run
in 27 minutes, is he qualified for the medal?
Pingat hanya diberikan kepada 30 orang peserta terawal yang menamatkan larian. Jika Johan menamatkan lariannya dalam masa
27 minit, adalah dia layak mendapat pingat itu?
[5 marks/markah]
H O TS Zo n e
1 The lecture hours in a college start at 0830 hours. On a certain day, it is found that the arrival of students in the
college are normally distributed with a mean of 0815 hours and a standard deviation of 12 minutes.
Waktu kuliah di sebuah kolej bermula pada jam 0830. Pada suatu hari tertentu, didapati bahawa waktu ketibaan pelajar di kolej itu
bertaburan normal dengan min jam 0815 dan sisihan piawai 12 minit.
(a) Find the percentage of the students who are late to the college.
Cari peratus pelajar yang lewat ke kolej.
(b) Determine the time if 60% of the students have arrived in the college. Applying
112
6 Trigonometric Functions
Fungsi Trigonometri
6.1 Positive Angles and Negative Angles / Sudut Positif dan Sudut Negatif
Smart Tip
1 Positive angle is an angle measured in the anticlockwise direction from the positive x-axis.
Sudut positif ialah suatu sudut yang diukur mengikut lawan arah jam dari paksi-x yang positif.
2 Negative angle is an angle measured in the clockwise direction from the positive x-axis.
Sudut negatif ialah suatu sudut yang diukur mengikut arah jam dari paksi-x yang positif.
y y
x
O θ°
θ°
x
O
Exercise 1 Represent each of the following angles in Cartesian plane. Hence, determine which quadrant
each of the angles lies in the Cartesian plane.
Wakilkan setiap sudut yang berikut dalam satah Cartes. Seterusnya, tentukan sukuan yang manakah setiap sudut
itu berada dalam satah Cartes.
PL 1 Demonstrate the basic knowledge of trigonometric functions.
Example 1
p
(a) 600° (b) − 4 rad
Solution
(a) y (b)
y
x
60°
π x
7π
4 4
113
11π
5
π
5 80°
x x
11 p
p − 2p = 720° – 640° = 80°
5 5
First quadrant First quadrant
3 840° 4 – 7 p rad
y 6
y
5π
120° 6
60° 7
π
x 6
x
7 5
840° – 720° = 120° 2p − p= p
6 6
Second quadrant Second quadrant
6.2 Trigonometric Ratios of Any Angle/ Nisbah Trigonometri bagi Sebarang Sudut
Smart Tip
y Adjacent side
Sisi bersebelahan x 1
cot θ/ kot θ = =
r
P(x, y) Opposite side y = tan θ
r Sisi bertentangan
y
θ°
r x
−r O x
−r
Hypotenus
Opposite side Hipotenus Opposite to θ
Sisi bertentangan y Bertentangan dengan θ
sin θ = =
Hypotenuse r θ°
Hipotenus
Adjacent to θ
Adjacent side Bersebelahan dengan θ
Sisi bersebelahan x
cos θ/ kos θ = =
Hypotenuse r Quadrant II/ Sukuan II Quadrant I/ Sukuan I
Hipotenus sin θ = sin(180° − θ) sin θ
Opposite side cos θ = − cos(180° − θ) cos θ/ kos θ
kos θ = − kos(180° − θ) tan θ
Sisi bertentangan y
tan θ = = tan θ = − tan(180° − θ)
Adjacent side x
Sisi bersebelahan
sin (+) All (+)/ Semua (+)
Hypotenuse
Hipotenus r 1 tan (+) cos (+)/ kos (+)
cosec θ/ kosek θ = =
Opposite side y = sin θ
Sisi bertentangan Quadrant III/ Sukuan III Quadrant IV/ Sukuan IV
sin θ = − sin (180° + θ) sin θ = − sin (360° − θ)
Hypotenuse cos θ = − cos (180° + θ) cos θ = cos (360° − θ)
Hipotenus r 1 kos θ = − kos (180° + θ) kos θ = kos (360° − θ)
sec θ/ sek θ = =
Adjacent side x = cos θ/kos θ tan θ = tan (180° + θ) tan θ = − tan (360° − θ)
Sisi bersebelahan
114
Example 2 24
1 Given cos θ = , where 180° θ 360°, find the
25
3 value of
If sin θ = 5 , where 90° θ 180°, find the value of 24
3 Diberi kos θ = , dengan keadaan 180° θ 360°, cari
Jika sin θ = , dengan keadaan 90° θ 180°, cari nilai 25
5 nilai
(a) tan θ, (b) sec θ/sek θ. (a) sin θ, (b) cot θ/ kot θ.
Solution y
y
24 x
5 O θ
3
θ 25
x
−4 O −7
5 5
Given tan x =
12 and x is not in the first quadrant, Given tan x = − 7 , where 90° x 270°, find the
2 3
find the value of value of
5 5
Diberi tan x = dan x tidak berada dalam sukuan pertama, Diberi tan x = – , dengan keadaan 90° x 270°, cari nilai
12 7
cari nilai (a) cosec x/ kosek x, (b) sec x/sek x.
(a) sin x, (b) cosec x/kosek x. y
y
5
74
5
−12 x
x x
x O −7 O
13
−5 5
(a) sin x =
74
1
–5 cosec x =
(a) sin x = 13 5
74
5
= − 13 74
=
5
1
(b) cosec x = –7 7
5 (b) cos x = =–
– 13 74 74
1
13 sec x =
=− 7
5 –
74
74
=–
7
115
Exercise 3 Given sin 50° = 0.766 and cos 50° = 0.6428, find the value of
Diberi sin 50° = 0.766 dan kos 50° = 0.6428, cari nilai
PL 2 Demonstrate the understanding of trigonometric functions.
Example 3
1 sin 40° 2 tan 40°
cos 40°/ kos 40°
sin 40° = cos (90° − 40°) tan 40° = cot (90° − 40°)
Solution = cos 50° = cot 50°
cos 40°/ kos 40° = 0.6428 cos 50°
= sin(90° − 40°) = sin 50°
= sin 50° 0.6428
= 0.766 = 0.766
= 0.8392
3 sec 40°/ sek 40° 4 cosec 40°/ kosek 40° 5 cot 40°/ kot 40°
sec 40° = cosec (90° − 40°) cosec 40˚ = sec (90˚ − 40˚) cot 40° = tan (90° − 40°)
= cosec 50° = sec 50˚ = tan 50°
1 1 sin 50°
= sin 50° = cos 50° = cos 50°
1 1 0.766
= 0.766 = 0.6428 = 0.6428
= 1.305 = 1.556 = 1.192
Smart Tip
The values of the trigonometric ratios of the special angles 30°, 45° and 60° are:
Nilai nisbah trigonometri bagi sudut-sudut khas 30°, 45° dan 60° ialah:
30° Ratio
2 2 Nisbah cos θ cosec θ sec θ cot θ
sin θ tan θ
3 Angle kos θ kosek θ sek θ kot θ
Sudut
60° 1 2
1 3
1 1 30° 2 3
2 2 3 3
1 1
45° 45° 1 2 2 1
2 2
2 3 1 2 1
1 60° 2
2 2 3 3 3
45°
1
116
6
4 4
cos 3 p/ kos 3 p 7 ( 5
) ( )
5
sec − 4 p / sek − 4 p
8 cosec (−300°)/ kosek (−300°)
cosec (−300°)
4
cos p
3 ( 5
sec − 4 p ) = cosec (–360 – (–300°))
= cosec (–60°)
= cos 240° = sec (–225°) = – cosec (60°)
= – cos (240° – 180°) = sec (–225° – (–180°)) 2
= – cos 60° = sec (–45°) =−
1 3
=− = – sec (45°)
2
=– 2
6.3 Graphs of Sine, Cosine and Tangent Functions / Graf Fungsi Sinus, Kosinus dan Tangen
Smart Tip
1 y = sin x 2 y = cos x/ kos x 3 y = tan x
y y y
1 1
x x x
0 0 90° 180° 270° 360° 0 90° 180° 270° 360°
90° 180° 270° 360°
1 1
x 0° 90° 180° 270° 360° x 0° 90° 180° 270° 360° x 0° 90° 180° 270° 360°
y 0 1 0 –1 0 y 1 0 –1 0 1 y 1 ∞ 0 ∞ 0
117
p 3p y 0 –1 –2 –1 0
x 0 p 2p
2 2
y
y 0 –4 0 4 0
y
x
O 3 2
4
2 2
1
x
O π π 3π 2π
2 2
2
−4
2 y = tan x + 1 3 y = 3 sin x
p 3p p 3p
x 0 p 2p x 0 p 2p
2 2 2 2
y 1 ∞ 1 ∞ 1 y 0 3 0 –3 0
y y
1
x
x O π π 3π 2π
O π π 3π 2π 2 2
2 2
−3
p 3p p 3p
x 0 p 2p x 0 p 2p
2 2 2 2
y 0 ∞ 0 ∞ 0 y –4 0 4 0 –4
y y
x x
O 3 2 O π π 3π 2π
2 2 2 2
−4
118
Example 6
1 y = sin 2x
y = |cos 2x|/ y = |kos 2x| 2p
Period = 2 = p
Solution Number of cycles = 2
2p Amplitude = 1
Period/ Kala = 2 = p
Number of cycles/ Bilangan kitaran = 2
Amplitude/ Amplitud = 1 p p 3p 5p 3p 7p
x 0 p 2p
4 2 4 4 2 4
p p 3p 5p 3p 7p y 0 1 0 –1 0 1 0 –1 0
x 0 p 2p
4 2 4 4 2 4
y
y 1 0 1 0 1 0 1 0 1
1
y
x
1
O π π 3π π 5π 3π 7π 2π
4 2 4 4 2 4
−1
x
O 3 5 3 7 2
4 2 4 4 2 4
p 3p p 3p
x 0 p 2p x 0 p 2p
2 2 2 2
y 1 0 1 0 1 y 0 ∞ 0 ∞ 0
y y
x
O π π 3π 2π
2 2
−1 x
O 3 2
2 2
p p 3p 5p 3p 7p p p 3p 5p 3p 7p
x 0 p 2p x 0 p 2p
4 2 4 4 2 4 4 2 4 4 2 4
y 0 ∞ 0 ∞ 0 ∞ 0 ∞ 0 y –1 0 –1 0 –1 0 –1 0 –1
y y
x
O π π 3π π 5π 3π 7π 2π
4 2 4 4 2 4
O x −1
3 5 3 7 2
4 2 4 4 2 4
119
Example 7
1 Sketch the graph y = sin x for 0 < x < p. Then, on
the same axes, draw a suitable straight line to find
Sketch the graph y = cos x for 0 < x < 2p. Then, on the number of solutions for the equation
the same axes, draw a suitable straight line to find x
the number of solutions for the equation 2 sin x − p = 0.
x Lakarkan graf y = sin x untuk 0 < x < p. Seterusnya, pada
2 cos x = 1 − p .
paksi yang sama, lukis garis lurus yang bersesuaian untuk
Lakarkan graf y = kos x untuk 0 < x < 2p. Seterusnya, pada
paksi yang sama, lukis garis lurus yang bersesuaian untuk mencari bilangan penyelesaian bagi persamaan
x x
mencari bilangan penyelesaian bagi persamaan 2 kos x = 1 – p . 2 sin x − p = 0.
Solution y = sin x
y = cos x p
x 0 p
2
p 3p
x 0 p 2p y 0 1 0
2 2
y 1 0 –1 0 1 x
2 sin x − p = 0
x x
2 cos x = 1 − p sin x – 2p = 0
x 0 p x 0 p
1 x x
cos x = 2 − 2p sin x = 2p
1 1
1 x y 0 x y 0
2 y = 2p 2
y = 2 − 2p
y
y
1
1 y = cos x y = sin x x
1 y=
2 2
1
O x
π π 3π 2π 2
2 2 1 x
y= − x
2 2π
−1 O
2
Number of solutions/ Bilangan penyelesaian = 2 Number of solutions = 2
2 Sketch the graph y = −tan x for 0 < x < 2p. Then, on p tan x + 2x = 0
the same axes, draw a suitable straight line to find 2x
tan x + p = 0
the number of solutions for the equation
p tan x + 2x = 0. 2x
p = – tan x x 0 p
Lakarkan graf y = −tan x untuk 0 < x < 2p. Seterusnya, pada 2x
paksi yang sama, lukis garis lurus yang bersesuaian untuk y= p
mencari bilangan penyelesaian bagi persamaan y 0 2
p tan x + 2x = 0.
y
2x
y = – tan x y=
p 3p
x 0 p 2p 2
2 2
y 0 ∞ 0 ∞ 0 x
O 3 2
2 2
Number of solutions = 3
120
x 1
cos 2x = 3p –
3 x 0 p 2p
2x
2|tan 2x| = p – 4 (× 12 )
x x
3 cos 2x = p – 1 |tan 2x| = p – 2
x 0 p 2p
x y –1 0 1 x
y = p – 1 y = p – 2
y –2 –1 0
y
y
3 y = 3 cos 2x
y =|tan 2x|
2
x
y= 1
1
2
x
O 3 2 y= x –2
1 p
2 2 x
O 3 2
2 2 2
2
3
3
5 Sketch the graph y = 2 cos 2 x for 0 < x < p. Then,
3 1 x
on the same axes, draw a suitable straight line to cos 2 x = – 2p
find the number of solutions for the equation 2 x 0 p 2p
3 1 x 3 x
cos 2 x = 2 − 2p . 2 cos 2 x = 1 – p
y 1 0 –1
3 x
Lakarkan graf y = 2 kos x untuk 0 < x < p. Seterusnya, y = 1 – p
2
pada paksi yang sama, lukis garis lurus yang bersesuaian y
untuk mencari bilangan penyelesaian bagi persamaan
3 1 x 2
kos x = − .
2 2 2p 3
1 y = 2 cos x
2
3
y = 2 cos 2 x
O x
π 2π π 4π 5π 2π
p 2p 4p 5p −1 3 3 3
x 0 p 2p 3
3 3 3 3 y=1− x
−2 π
y 2 0 –2 0 2 0 –2
Number of solutions = 3
121
Smart Tip
1 sin2 θ + cos2 θ = 1 2 1 + tan2 θ = sec2 θ 3 1 + cot2 θ = cosec2 θ
sin2 θ + kos2 θ = 1 1 + tan2 θ = sek2 θ 1 + kot2 θ = kosek2 θ
Exercise 8 Solve the following trigonometric equations for 0° < x < 360°.
Selesaikan persamaan trigonometri yang berikut untuk 0° < x < 360°.
PL 3 Apply the understanding of trigonometric functions to perform simple tasks.
Example 8
1 2 cos2 x = 1 – sin x
2 kos2 x = 1 – sin x
tan2 x – 3 sec x = 3
tan2 x – 3 sek x = 3
2 cos2 x = 1 – sin x
2(1 – sin2 x) = 1 – sin x
Solution
2 – 2 sin2 x = 1 – sin x
tan2 x – 3 sec x = 3
2 sin x – sin x – 1 = 0
2
(sec x – 1) – 3 sec x – 3 = 0
2
(2 sin x + 1)( sin x – 1) = 0
sec2 x – 3 sec x – 4 = 0
2 sin x + 1 = 0 , sin x – 1 = 0
(sec x + 1)( sec x – 4) = 0
2 sin x = −1 , sin x = 1
sec x + 1 = 0 , sec x – 4 = 0 1
sec x = −1 , sec x = 4 sin x = − 2 ,
1 1 1
cos x = −1 , cos x = 41 When sin x = − 2
cos x = −1 , cos x = 4 x = 210°, 330°
When/ Apabila cos x = −1 When sin x = 1
x = 180° x = 90°
1 [ x = 90°, 210°, 330°
When/ Apabila cos x = ,
4
x = 75°31’, 284°29’
[ x = 75°31’, 180°, 284°29’
122
Example 9
1 7 – 6 sin2 θ = 1 + 6 cos2 θ
7 – 6 sin2 θ = 1 + 6 kos2 θ
sec2 x(cos2 x − cot2 x) − 1 = − cosec2 x
sek2 x(kos2 x − kot2 x) − 1 = − kosek2 x LHS:
7 – 6 sin2 θ
Solution = 7 – 6(1 – cos2 θ)
LHS/ Sebelah kiri: = 7 – 6 + 6 cos2 θ
sec2 x(cos2 x − cot2 x) − 1 = 1 + 6 cos2 θ (RHS)
1
( cos2 x
= cos2 x cos2 x − sin2 x − 1 )
1
=1– −1
sin2 x
= − cosec x (RHS/ Sebelah kanan)
2
LHS: LHS:
(sec θ + 1)(sec θ − 1) 3 + 2 cosec2 θ
= sec2 θ − 1 = 3 + 2 (cot2 θ + 1)
= (1 + tan2 θ ) − 1 = 3 + 2 cot2 θ + 2
= tan2 θ (RHS) = 5 + 2 cot2 θ (RHS)
123
6.5 Addition Formulae and Double Angle Formulae / Rumus Sudut Majmuk dan Rumus Sudut Berganda
Smart Tip
Addition formulae/ Rumus sudut majmuk:
1 sin ((A ± B) = sin A cos B ± cos A sin B/ sin (A ± B) = sin A kos B ± kos A sin B
2 cos ((A ± B) = cos A cos B sin A sin B/ kos (A ± B) = kos A kos B sin A sin B
tan A ± tan B tan A ± tan B
( ± B) =
3 tan (A / tan (A ± B) =
1 tan A tan B 1 tan A tan B
124
125
LHS: sin (A + B)
= tan A + tan B
kos A kos B
(
sin 4θ −
p
)
3 + sin 4θ + 3
p
( ) LHS:
( p
= sin 4θ cos − cos 4θ sin
3
p
3 )
+ sin (A + B)
cos A cos B
( p
sin 4θ cos 3 + cos 4θ sin
p
3 ) =
sin A cos B + cos A sin B
cos A cos B
p sin A cos B cos A sin B
= 2 sin 4θ cos =
3 cos A cos B + cos A cos B
= 2 sin 4θ
1
2 () =
sin A sin B
cos A + cos B
= sin 4θ (RHS) = tan A + tan B (RHS)
2 sin 50° cos 40° − cos 50° sin 40° 3 cos 55° cos 15° − sin 55° sin 15°
sin 50° kos 40° − kos 50° sin 40° kos 55° kos 15° − sin 55° sin 15°
sin 50° cos 40° − cos 50° sin 40° cos 55° cos 15° − sin 55° sin 15°
= sin (50° − 40°) = cos (55° + 15°)
= sin 10° = cos 70°
126
Example 12
3 12
Given sin A = and tan B = − 5 , where A is an acute angle and B is an obtuse angle. Find the value of
5
3 12
Diberi sin A = dan tan B = − , dengan keadaan A ialah sudut tirus dan B ialah sudut cakah. Cari nilai
5 5
(a) cos (A + B)/ kos (A + B),
(b) tan (A – B).
Solution (a) cos(A + B)
y = cos A cos B − sin A sin B
sin A =
3
5
= ( )( ) ( )( )
4
5
−
5
13
−
3 12
5 13
5 4 20 36
3 cos A = =− −
5 65 65
A 56
x 3 =−
O 4 tan A = 65
4
y
12
sin B = (b) tan(A – B)
13
13 5 tan A – tan B
12 cos B = − =
13 1 + tan A tan B
() ( )
B x 12 3 12
−5 O tan B = − – −
5 = 4 5
1+ ( )( )
3
4
−
12
5
63
=−
16
4 5
1 Given sin A = and tan B = 12 , where A and B are acute angles. Find the value of
5
4 5
Diberi sin A = dan tan B = , dengan keadaan A dan B ialah sudut tirus. Cari nilai
5 12
(a) tan (A + B),
(b) sin (B − A).
y (a) tan (A + B)
4 tan A + tan B
sin A = =
5 1 – tan A tan B
5 3 4 5
4 cos A = +
A
5 3 12
x 4 =
O 3 tan A =
3 1– ( )( )
4 5
3 12
7
= 4
y 4
5 9
sin B = 63
13 =
13 12 16
5 cos B =
B 13 (b) sin (B – A)
x
O 12 5 = sin B cos A − cos B sin A
tan B =
12
= ( )
5 3
13 5
− ( )
12 4
13 5
15 48
= −
65 65
33
=−
65
127
y (a) sin (R + Q)
= sin R cos Q + cos R sin Q
3
sin R = –
4
5
= − ( )( ) ( )( )
4 12
5 13
+ −
3
5
−
5
13
x
R O 3 48 15
cos R = – =− +
4
5 5 65 65
33
=−
65
(b) cos (R + Q)
y
= cos R cos Q – sin R sin Q
12
= − ( )( ) ( )( )
3 12
5 13
– −
4
5
−
5
13
x 5 36 20
O Q sin Q = – =− –
5
13 65 65
13 12 56
cos Q =
13 =−
65
3 Given cos θ = m for 0° < θ < 90°, express in terms (a) cos (90° − θ)
of m, = cos 90°cos θ + sin 90° sin θ
Diberi kos θ = m bagi 0° < θ < 90°, ungkapkan dalam sebutan = sin θ
m, = 1 – m2
(a) cos(90°− θ)/ kos(90°− θ),
(b) cot θ/ kot θ. (b) cot θ
1
y =
tan θ
cos θ = m 1
sin θ = 1 – m2 =
1 1 m2
tan θ =
1 – m2 1 – m2
m
θ° m
x m
O m =
1 – m2
Smart Tip
Double-angle formulae: Half-angle formulae:
Rumus sudut berganda: Rumus sudut separuh:
1 sin 2A = 2 sin A cos A A A
1 sin A = 2 sin cos
sin 2A = 2 sin A kos A 2 2
A A
2 cos 2A = cos2 A – sin2 A sin A = 2 sin kos
2 2
kos 2A = kos A – sin A
2 2
cos 2A = 2 cos2 A – 1 A A A A
2 cos A = cos2 − sin2 / kos A = kos2 2 − sin2 2
kos 2A = 2 kos A – 12 2 2
A A
cos 2A = 1 – 2 sin2 A = 2 cos2 − 1 = 2 kos2 − 1
2 2
kos 2A = 1 – 2 sin2 A A 2A
= 1 − 2 sin2 = 1 − 2 sin
2 tan A 2 2
3 tan 2A =
1 – tan2 A A
2 tan
2
3 tan A =
A
1 – tan 2
2
128
129
RHS: LHS:
sin θ
cos4 θ – sin4 θ = (cos2 θ + sin2 θ)(cos2 θ – sin2 θ) 2 tan θ 2 cos θ
= (1)(cos 2θ) =
1 + tan2 θ sec2 θ
= cos 2θ (LHS) sin θ
2 cos θ
=
1
cos 2
θ
sin θ
=2 cos θ
× cos2 θ
= 2 sin θ cos θ
= sin 2θ (RHS)
130
Example 15
3
Given cos θ = − , where θ is an obtuse angle. Find the value of
5
3
Diberi kos θ = − , dengan keadaan θ ialah sudut cakah. Cari
5
1 1
(a) sin 4θ, (b) cos θ./ kos θ.
2 2
Solution θ
(b) 2 cos2 − 1 = cos θ
y 2
θ cos θ + 1
4 cos2 =
sin θ = 2 2
5 θ cos θ + 1
5 3 cos =
4 cos θ = − 2 2
θ 5
(− 35 ) + 1
x
−3 O
=
2
(a) sin 4θ
= 2 sin 2θ cos 2θ ( 25 )
= 2 (2 sin θ cos θ)(2 cos2 θ − 1) =
2
=2 2
4
5[ ( )( )][ ( ) ]
−
3
5
2−
3 2
5
−1
=
1
5
=2 −
24
25( )( )
−
7
25
336
=
625
12 35
1 Given tan x = for 0° < x < 90°, find 2 Given cos A = where A is an acute angle. Find
5 37 ,
12 the value of
Diberi tan x = bagi 0° < x < 90°, cari 35
5 Diberi kos A = , dengan keadaan A ialah sudut tirus.
(a) sin 2x, (b) tan 2x. 37
Cari nilai
y A A A
12 (a) cos / kos , (b) sin .
sin x = 2 2 2
13
5 A
13 cos x =
12 13 (a) cos A = 2 cos2 −1
2 y
x 12
O
x tan x = A cos A + 1
5 5 cos =
2 2
37
( )
12
35
(a) sin 2x = 2 sin x cos x +1 A
= 37
( )( )
12 5 x
O 35
=2 2
13 13 6
= 12
120 37 sin A =
= 37
169
A A 35
2 tan x (b) sin A = 2 sin cos cos A =
2 2 37
(b) tan 2x =
1 – tan2 x
( )
12 A 6
12 = 2 sin
2
5
37 2 37
= A 12 1 37
12 2
1–
5
sin
2
=
37 2
× ×
6
24 37
5
=
37
= 1
119 =
–
25 37
120
=–
119
131
(a) cosec θ
1 (a) cos 2x = 2 cos2 x – 1
=
sin θ
=
1 = 2 – ( )
4 2
5
– 1
θ θ
2 sin cos
2 2 = 32 – 1
25
1
=
2(t)( 1 – t2 ) = 7
25
1
= (b) cos 4x = 2 cos2 (2x) – 1
2t( 1 – t2 )
( )
= 2 7 – 1
2
25
(b) tan θ
θ = 98 – 1
2 tan 625
2
=
θ = – 527
1 – tan2 625
2
2( t
1 – t2 ) x
− 1 = cos x
(c) 2 cos2
2
=
( t
)
2
1– x cos x + 1
1–t 2 cos2 =
2 2
( 2t
1 – t2 ) x
cos =
2
cos x+1
2
=
t2
1–
1 – t2
=
–
4
5
+1
( )
( 2t
1 – t2 ) 2
=
1 – 2t2
=
( 15 )
1 – t2 2
2t 1 – t2
= × 1
1–t 2
1 – 2t2 =
10
2t(1 – t2)
=
(1 – 2t2) 1 – t2
2t(1 – t2) 1 – t2
= ×
(1 – 2t2) 1 – t2 1 – t2
2t(1 – t2) 1 – t2
=
(1 – 2t2)(1 – t2)
2t 1 – t2
=
1 – 2t2
132
Exercise 16 Solve each of the following equations for 0 < x < 360°.
Selesaikan setiap persamaan yang berikut bagi 0 < x < 360°.
PL 3 Apply the understanding of trigonometric functions to perform simple tasks.
Example 16
1 2 sin 2x = cos x
2 sin 2x = kos x
2 cos 2x – 4 sin x + 1 = 0/ 2 kos 2x – 4 sin x + 1 = 0
2 sin 2x = cos x
Solution
2 sin 2x – cos x = 0
2 cos 2x – 4 sin x + 1 = 0
2(2 sin x cos x) – cos x = 0
2(1 – 2 sin² x) – 4 sin x + 1 = 0
4 sin x cos x – cos x = 0
2 – 4 sin² x – 4 sin x + 1 = 0
cos x (4 sin x – 1) = 0
4 sin² x + 4 sin x − 3 = 0
cos x = 0 or 4 sin x – 1 = 0
(2 sin x − 1)(2 sin x + 3) = 0 1
2 sin x – 1 = 0 or/atau 2 sin x + 3 = 0 sin x = 4
1 3 When cos x = 0
sin x = 2 sin x = − 2
1 x = 90°, 270°
When/ Apabila sin x = 2 1
When sin x = 4
x = 30°, 150°
3 x = 14°29’, 165°31’
When sin x = − 2 , x has no value.
[ x = 14°29’, 90°, 165°31’, 270°
3
Apabila sin x = − , x tidak mempunyai nilai.
2
[ x = 30°, 150°
133
Example 17
y
u
The diagram shows a particle is projected with a speed u m s at an acute angle −1
θ° from the horizontal ground. The range of the displacement of the particle, Sx,
is given by Sx = u2 sin θ cos θ.
Rajah menunjukkan satu zarah dilancarkan dengan kelajuan u m s−1 pada sudut tirus θ° dari
permukaan tanah mengufuk. Julat bagi sesaran zarah, Sx, diberi oleh Sx = u2 sin θ kos θ. θ
x
O
If Sx = 170 and u = 20, determine the values of θ.
Jika Sx = 170 dan u = 20, tentukan nilai-nilai θ.
Solution
Sx = u2 sin θ cos θ
1
(
= u2 sin 2θ
2 )
u2
= sin 2θ
2
When/Apabila Sx = 170, u = 20
202
170 = sin 2θ
2
170 × 2
sin 2θ = Video
202
= 0.85 Scan QR code or visit https://youtu.
2θ = 58.21°, 121.79° be/7Eo-fuy0f7g to watch an example
of using double angle identities to
θ = 29.11°, 60.90° solve equations.
For educational purposes only
134
O pada permukaan tanah mengufuk. Sudut pelancaran dari paksi-x ialah θ, dengan keadaan θ
ialah sudut tirus. Selepas t saat, zarah itu berada di titik P(20, 140). Diberi gerakan zarah itu
gx2
memenuhi persamaan y = x tan θ + sek² θ, dengan keadaan g m s−2 ialah pecutan graviti.
2v2
Jika v = 10 dan g = 10, tentukan nilai θ.
gx2
y = x tan θ + sec² θ
2v2
(10)(20)²
140 = 20 tan θ + (1 + tan2 θ)
2(10)²
140 = 20 tan θ + 20 (1 + tan2 θ)
140 = 20 tan θ + 20 + 20 tan2 θ
0 = 20 tan2 θ + 20 tan θ − 120
= tan2 θ + tan θ − 6 = 0
(tan θ − 2)(tan θ + 3) = 0
tan θ − 2 = 0 or tan θ + 3 = 0
tan θ = 2 tan θ = −3
Since 0° < θ < 90°
tan θ = 2
θ = 63°26’
2 The diagram shows a water wheel with centre P. P is h m above the water level.
The position of point Q on the water wheel is represented by the equation
p Q
y = − 6 sin t + 5, where t is the time taken for the wheel to complete one cycle,
8
in seconds and y is the distance from the water level, in metre. P Pusat
Centre
Rajah menunjukkan sebuah roda air berpusat P. P ialah h m di atas paras air. Kedudukan hm
p
titik Q pada roda air itu diwakili oleh persamaan y = − 6 sin 8 t + 5, dengan keadaan t ialah
masa yang diambil oleh roda air untuk membuat satu pusingan lengkap, dalam saat dan Paras air
Water level
y ialah jarak dari paras air, dalam meter.
Determine/ Tentukan
(a) the value of h,
nilai h,
(b) the diameter, in m, of the water wheel,
diameter, dalam m, bagi roda air,
(c) the maximum distance, in m, of point Q from the water level,
jarak maksimum, dalam m, bagi titik Q dari paras air,
(d) the time taken, in seconds, for the wheel to make a complete cycle.
masa yang diambil, dalam saat, bagi roda itu untuk membuat satu putaran lengkap.
(a) h = 5 m
(b) Amplitude = 6 m
[ Diameter = 2 × 6 m = 12 m
(c) 6 m + 5 m = 11 m
p
t = 2p
(d)
8
8
t = 2p × p
= 16 seconds
135
Paper 2
Section A
3
1 (a) Sketch the graph of y = 4 sin x for 0 < x < 2p.
2
3
Lakarkan graf y = 4 sin x bagi 0 < x < 2p.
2
[4 marks/markah]
(b) Hence, on the same axes, draw a suitable straight line to find the number of solutions for the equation
3
12p sin x = 6x − 3p for 0 < x < 2p.
2
Seterusnya, menggunakan paksi yang sama, lukis satu garis lurus yang bersesuaian untuk mencari bilangan penyelesaian bagi persamaan
3
12p sin x = 6x − 3p bagi 0 < x < 2p.
2
[3 marks/markah]
136
H O TS Zo n e
1 Express cos 3x in terms of cos x. Hence, solve the equation cos 3x + cos x = 0 for 0° < x < 360°. Applying
Ungkapkan kos 3x dalam sebutan kos x. Seterusnya, selesaikan persamaan kos 3x + kos x = 0 bagi 0° < x < 360°.
Seterusnya, cari nilai sin 75° sin 15° tanpa menggunakan kalkulator.
137
7 Linear Programming
Pengaturcaraan Linear
7.1 Linear Programming Model / Model Pengaturcaraan Linear
Smart Tip
Constraint Inequality
Kekangan Ketaksamaan
Exercise 1 Write down a mathematical model for each of the following situations.
Tuliskan satu model matematik bagi setiap situasi yang berikut.
PL 2 Demonstrate the understanding of linear programming model.
Solution Perimeter = 2x + 2y
Let x = number of bags with the price RM37.90 sold [ 2x + 2y , 64
in a day
bilangan beg yang berharga RM37.90 yang dijual
dalam sehari
y = number of bags with the price RM60 sold in
a day
bilangan beg yang berharga RM60 yang dijual dalam
sehari
[ 37.90x + 60y > 1 000
138
(i) x + y < 25
1
(ii) y . x
2
Smart Tip
1 If the inequalities are ax + by . c and ax + by > c, shade the region above the straight line ax + by = c.
Jika ketaksamaan ialah ax + by . c dan ax + by > cc, lorekkan rantau di bahagian atas garis lurus ax + by = c.
2 If the inequalities are ax + by , c and ax + by < c, shade the region below the straight line ax + by = c.
Jika ketaksamaan ialah ax + by , c dan ax + by < cc, lorekkan rantau di bahagian bawah garis lurus ax + by = c.
3 The solid line ( ----------------- ) is used for ‘>’ and ‘<’, while the dotted line ( ------- ) is used for ‘.’ and ‘,’.
Garis padu ( ----------------- ) digunakan untuk ‘>’ dan ‘<’,
’, manakala garis sempang ( ------- ) digunakan untuk ‘.’ dan ‘,’.
Example 2
1 x – 3y , 6
x + 2y , 6
Solution
y
4
2
2
x
4 2 0 2 4 6
x 2
0 2 4 6
4
139
y y
4
6
x
O 2 4 6 8
2
2
2
x
4 2 0 2 4 4
Exercise 3 Write a mathematical model related to the following situations. Hence, draw a graph to
represent each mathematical model obtained.
Tuliskan model matematik yang berkaitan dengan situasi berikut. Seterusnya, lukiskan graf untuk mewakili setiap
model matematik yang diperoleh.
PL 3 Apply the understanding of linear programming model to perform simple tasks.
Example 3
A restaurant allocates RM6 000 to buy more than 60 tables and chairs. The price of a table and a chair are
RM240 and RM80 respectively.
Sebuah restoran memperuntukkan RM6 000 untuk membeli lebih daripada 60 buah meja dan kerusi. Harga bagi sebuah meja
dan sebuah kerusi masing-masing ialah RM240 dan RM80.
Solution
Let/ Katakan
x = number of tables/ bilangan meja
y = number of chairs/ bilangan kerusi
Constraint II/Kekangan II
Constraint I/ Kekangan I: x + y . 60
y
Constraint II/ Kekangan II: 240x + 80y < 6 000
80
Constraint I/ Kekangan I
y
70
60
60
50
50
40
x + y 60 40
30
30
20
20
10 10
140
40
40
30
30
20
x + y 40
20
10 x 2y
10
x
0
10 20 30 40
x
0 10 20 30 40
2 A shop sells two types of shoes, A and B with the price RM120 and RM80 respectively. The total sales of the
shop is at least RM24 000 in a week. The total sales of shoes B is more than twice the total sales of shoes A.
Sebuah kedai menjual dua jenis kasut, A dan B masing-masing dengan harga RM120 dan RM80. Jumlah jualan bagi kedai itu dalam
seminggu adalah sekurang-kurangnya RM24 000. Jumlah jualan kasut B adalah lebih daripada dua kali jumlah jualan kasut A.
400 300
300 200
y 2x
200 100
120x + 80y 24 000
100 x
0 100 200 300
x
0
100 200 300
141
Let x= number of single storey houses and y = number of double storey houses
Constraint I: x – y > 150
Constraint II: x + y < 400
Constraint I: Constraint II:
y y
400 400
300 300
200 200
x x
O 0
100 200 300 400 100 200 300 400
Exercise 4 Write the optimal function for each of the following situations.
Tuliskan fungsi optimum bagi setiap situasi yang berikut.
PL 3 Apply the understanding of linear programming model to perform simple tasks.
Example 4
The table shows the selling price and the cost price of two types of foods sold in a stall.
Jadual menunjukkan harga jual dan harga kos bagi dua jenis makanan yang dijual di sebuah gerai.
Solution
Let/Katakan
x = number of packets of nasi lemak sold
bilangan bungkus nasi lemak yang dijual Smart Tip
y = number of packets of fried noodles sold
Optimal function/ Fungsi optimum, k = ax + by
bilangan bungkus mi goreng yang dijual
k = (1.60 – 1.20)x + (2.00 – 1.45)y
k = 0.4x + 0.55y
142
3 The table shows the selling price and the cost price of two types of calculators sold in a stationery shop.
Jadual menunjukkan harga jual dan harga kos bagi dua jenis kalkulator yang dijual di sebuah kedai alat tulis.
ACTIVITY
Write the optimal function for the total profit obtained by the shop.
Tuliskan fungsi optimum bagi jumlah keuntungan yang diperoleh kedai itu.
Steps/Langkah-langkah:
1 Teacher provides a set of questions involving linear programming model on the coloured cards.
Guru menyediakan beberapa set soalan melibatkan model pengaturcaraan linear pada kad berwarna.
Example/Contoh:
(a) Identify the constraints in the problem.
Kenal pasti kekangan yang terdapat dalam masalah.
(b) Form a mathematical model related to the problem.
Tuliskan model matematik yang berkaitan dengan masalah.
(c) Represent each mathematical model obtained in (b) graphically.
Wakilkan setiap model matematik yang diperoleh di (b) secara grafik.
2 Students perform this activity in groups of three students. A coloured card is randomly selected for each group.
Murid melakukan aktiviti ini secara berkumpulan yang terdiri daripada tiga orang murid. Satu kad berwarna dipilih secara rawak
bagi setiap kumpulan.
3 Each group is required to answer all questions on the selected card. Write each answer on a mahjung paper.
Setiap kumpulan dikehendaki menjawab semua soalan yang terdapat pada kad yang dipilih. Tulis setiap jawapan pada kertas
mahjung.
4 The group work of each group is posted on the class’s notice board. Students are required to stand next to their
group work.
Hasil kerja setiap kumpulan ditampal pada papan kenyataan kelas. Murid-murid dikehendaki berdiri di sebelah hasil kerja masing-
masing.
5 A group is required to move to each group to evaluate the work of other groups. Once completed, other groups
need to do the same.
Satu kumpulan dikehendaki bergerak ke setiap kumpulan bagi menilai hasil kerja kumpulan yang lain. Setelah selesai, kumpulan
lain perlu melakukan langkah yang sama.
6 Teacher holds a discussion with students to enhance their understanding.
Guru mengadakan perbincangan dengan murid untuk menambahkan kefahaman mereka.
143
Example 5
situation.
Rajah menunjukkan rantau berlorek yang 60
memenuhi beberapa kekangan daripada suatu
(20, 54)
situasi.
(a) By using a suitable value of k, draw a 50
straight line k = 3x + 2y on the graph.
On the same graph, draw a straight
line parallel to the line k = 3x + 2y 40
that passes through each point of the
vertices of the region.
Menggunakan satu nilai k yang sesuai, lukis 30
garis lurus k = 3x + 2y pada graf tersebut.
Pada graf yang sama, lukis garis lurus yang
selari dengan garis lurus k = 3x + 2y dan 20
melalui setiap bucu rantau tersebut. (60, 15)
(b) Hence, find the maximum value and (10, 15)
the minimum value of k. 10
Seterusnya, cari nilai maksimum dan nilai
minimum bagi k.
x
O
10 20 30 40 50 60 70
Solution
y
Given/ Diberi k = 3x + 2y
(a) Let/ Katakan k = 12,
[ 3x + 2y = 12 60
(20, 54)
x 0 4
y 6 0 50
144
600
(300, 500)
500
300
200
(100, 150)
100
2x +y = 200
x
0
100 200 300 400 500 600
(a) By using a suitable value of k, draw a straight line k = 2x + y on the graph. On the same graph, draw a
straight line parallel to the line k = 2x + y that passes through each point of the vertices of the region.
Menggunakan satu nilai k yang sesuai, lukis garis lurus k = 2x + y pada graf tersebut. Pada graf yang sama, lukis garis lurus
yang selari dengan garis lurus k = 2x + y dan melalui setiap bucu rantau tersebut.
(b) Hence, find the maximum value and the minimum value of k.
Seterusnya, cari nilai maksimum dan nilai minimum bagi k.
Given k = 2x + y
(a) Let k = 200, [ 2x + y = 200
x 0 100
y 200 0
145
700
(600, 680)
600
400
300
200
(600, 150)
100
5x + 2y = 1 000
x
0
100 200 300 400 500 600
(a) By using a suitable value of k, draw a straight line k = 5x + 2y on the graph. On the same graph, draw a
straight line parallel to the line k = 5x + 2y that passes through each point of the vertices of the region.
Menggunakan satu nilai k yang sesuai, lukis garis lurus k = 5x + 2y pada graf tersebut. Pada graf yang sama, lukis garis lurus
yang selari dengan garis lurus k = 5x + 2y dan melalui setiap bucu rantau tersebut.
(b) Hence, find the maximum value and the minimum value of k.
Seterusnya, cari nilai maksimum dan nilai minimum bagi k.
Given k = 5x + 2y
(a) Let k = 1 000, [ 5x + 2y = 1 000
x 0 200
y 500 0
(b) Maximum point of the shaded region = (600, 680)
k = 5(600) + 2(680) = 14 360
[ The maximum value of k is 4 360.
Minimum point of the shaded region = (100, 450)
k = 5(100) + 2(450) = 1 400
[ The minimum value of k is 1 400.
146
1 400
1 200
(400, 1 080)
800
(200, 700)
600
400
x
0
200 400 600 800 1 000 1 200
x + 2y = 400
(a) By using a suitable value of k, draw a straight line k = x + 2y on the graph. On the same graph, draw a
straight line parallel to the line k = x + 2y that passes through each point of the vertices of the region.
Menggunakan satu nilai k yang sesuai, lukis garis lurus k = x + 2y pada graf tersebut. Pada graf yang sama, lukis garis lurus
yang selari dengan garis lurus k = x + 2y dan melalui setiap bucu rantau tersebut.
(b) Hence, find the maximum value and the minimum value of k.
Seterusnya, cari nilai maksimum dan nilai minimum bagi k.
Given k = x + 2y
(a) Let k = 400, [ x + 2y = 400
x 0 400
y 200 0
(b) Maximum point of the shaded region = (900, 960)
k = 900 + 2(960) = 2 82
0
[ The maximum value of k is 2 820.
Minimum point of the shaded region = (400, 200)
k = 400 + 2(200) = 800
[ The minimum value of k is 800.
147
Example 6
A tuition centre offers two subjects, namely Biology and Chemistry for Form Five students. x is the number of
students taking Biology and y is the number of students taking Chemistry. The admission of the students to the
tuition centre must be based on the following constraints.
Sebuah pusat tusyen menawarkan dua mata pelajaran, iaitu Biologi dan Kimia untuk murid-murid Tingkatan Lima. x ialah bilangan
murid yang mengambil Biologi dan y ialah bilangan murid yang mengambil Kimia. Kemasukan murid ke pusat tusyen itu mestilah
berdasarkan kekangan yang berikut.
I The ratio of the number of students taking Biology to the number of students taking Chemistry is not more
than 70 : 30.
Nisbah bilangan murid yang mengambil Biologi kepada bilangan murid yang mengambil Kimia adalah tidak lebih daripada 70 : 30.
II The total number of students for both subjects is less than or equal to 70.
Jumlah bilangan murid untuk kedua-dua subjek adalah kurang atau sama dengan 70.
III The number of students taking Chemistry is at least 8.
Bilangan murid yang mengambil Kimia adalah sekurang-kurangnya 8 orang.
IV The number of students taking Biology is at least 15.
Bilangan murid yang mengambil Biologi adalah sekurang- kurangnya 15 orang.
(a) Write four inequalities, other than x > 0 and y > 0 that satisfy all the above constraints.
Tulis empat ketaksamaan, selain daripada x > 0 dan y > 0 yang memuaskan semua kekangan di atas.
(b) Using a scale of 2 cm to 10 students on both axes, construct and shade the region R which satisfies all the
above constraints.
Dengan menggunakan skala 2 cm kepada 10 orang murid pada kedua-dua paksi, bina dan lorekkan rantau R yang memenuhi semua
kekangan di atas.
(c) Based on the graph constructed in (b), find
Berdasarkan graf yang dibina di (b), cari
(i) the range of the number of students taking Biology if the number of students taking Chemistry is 15,
julat bilangan murid yang mengambil Biologi jika bilangan murid yang mengambil Kimia ialah 15 orang,
(ii) the maximum total fees obtained by the tuition centre if the fees for Biology and Chemistry are RM100
and RM300 respectively.
jumlah yuran maksimum yang diperoleh pusat tusyen itu jika yuran Biologi dan Kimia masing-masing ialah RM100 dan
RM300.
Solution
x 70 (b) y
(a) I: y < 30
3 70
y> 7 x
x = 15
II: x + y < 70
III: y > 8 60
IV: x > 15
50
(c) (i) 15 < x < 35
(ii) Maximum total fees
Jumlah yuran maksimum 40
= 100x + 300y
= 100(15) + 300(55)
3
= RM18 000 30 y =7 x
R
20
x + y = 70
10
y=8
x
O 10 20 30 40 50 60 70
148
150
R
100
50
x + y = 300
x
0 50 100 150 200 250 300 350
149
250
200
R
150
x = 3y
100
x + y = 400
50
20x + 5y = 2 000
x
0 50 100 150 200 250 300 350 400
150
151
152
H O TS Zo n e
1 An oil company has two oil refineries, A and B. The operating costs per day of oil refinery A and B are RM25 000 and
RM24 000 respectively. The table below shows the amount of oil produced in both oil refineries in a day.
Sebuah syarikat minyak mempunyai dua kilang penapis minyak, A dan B. Kos operasi sehari bagi kilang penapis minyak A dan B masing-
masing ialah RM25 000 dan RM24 000. Jadual di bawah menunjukkan jumlah minyak yang dihasilkan oleh dua buah kilang penapis
minyak itu dalam sehari.
A 50 100 150
B 100 50 100
The company receives a demand to supply 800 barrels of high grade oil, 1 000 barrels of medium grade oil and 1 800
barrels of low grade oil. How many days are needed for each oil refinery to meet the demand with the minimum
operating cost? Evaluating
Syarikat itu menerima permintaan untuk membekalkan 800 tong minyak gred tinggi, 1 000 tong minyak gred sederhana dan 1 800 tong
minyak gred rendah. Berapa harikah yang diperlukan oleh setiap kilang penapis minyak untuk memenuhi permintaan itu dengan kos
operasi yang minimum?
153
Smart Tip
1 A distance of a particle from a fixed point, O, measured in a specific direction is known as instantaneous displacement, s.
Jarak suatu zarah dari satu titik tetap, O, yang diukur pada arah tertentu dikenali sebagai sesaran seketika, s.
s = negative s = positive
negatif s=0 positi
positif
2 The rate of change of displacement with respect of time is known as instantaneous velocity, v.
Kadar perubahan sesaran terhadap masa dikenali sebagai halaju seketika, v.
v zero/ sifar v positive/ positif v negative/ negatif
negati
Example 1
A particle moves along a straight line and passes through a fixed point O. Its displacement, s m, t seconds after
passing through O is given by s = 5t 2 − t. Find
Suatu zarah bergerak di sepanjang satu garis lurus dan melalui satu titik tetap O. Sesaran, s m, zarah itu pada masa t saat selepas melalui
O diberi oleh s = 5t2 − t. Cari
(a) the instantaneous displacement, in m, of the particle when t = 4,
sesaran seketika, dalam m, zarah itu apabila t = 4,
(a) the distance, in m, travelled by the particle in the fourth second.
jarak, dalam m, yang dilalui oleh zarah itu dalam saat keempat.
O 42 76 Smart Tip
in the fourth second The distance travelled in the
dalam saat keempat nth second
Jarak yang dilalui pada saat ke-n
= |Sn − Sn −1|
154
t
3 6
155
4m 1m
2 A particle moves along a straight line and passes 3 A particle moves along a straight line and passes
through a fixed point O. Its displacement, s m, through a fixed point O. Its displacement, s m, is
t seconds after passing through O is given by given by s = 8t – t 2 – 5, where t is the time in seconds
s = t 2 – 4t + 2. Calculate the total distance, in m, after passing through O. Calculate the total distance,
travelled by the particle in the first 5 seconds. in m, travelled by the particle in the first 7 seconds.
Suatu zarah bergerak di sepanjang satu garis lurus dan melalui Suatu zarah bergerak di sepanjang satu garis lurus dan melalui
satu titik tetap O. Sesaran, s m, zarah itu pada masa t saat satu titik tetap O. Sesarannya, s m, diberi oleh s = 8t – t2 – 5,
selepas melalui O diberi oleh s = t2 – 4t + 2. Hitung jumlah dengan keadaan t ialah masa dalam saat selepas melalui O.
jarak, dalam m, yang dilalui oleh zarah itu dalam 5 saat yang Hitung jumlah jarak, dalam m, yang dilalui oleh zarah itu dalam
pertama. 7 saat yang pertama.
t (s) 0 1 2 3 4 5 t (s) 0 1 2 3 4 5 6 7
s (m) 2 –1 –2 –1 2 7 s (m) –5 2 7 10 11 10 7 2
4m
16 m
2 1 2 7 5 2 7 10 11
9m
9m
The total distance travelled in the first 5 seconds The total distance travelled in the first 7 seconds
=4+9 = 16 + 9
= 13 m = 25 m
156
Smart Tip
ds dv d2s
v = dt a= =
dt dt2
Example 3
1 A particle moves along a straight line so that its
displacement, s m, from a fixed point O is given
A particle moves along a straight line and passes
by s = t3 − 12t2, where t is the time in seconds after
through a fixed point O. Its displacement, s m,
movement starts. Find
is given by s = t2 − 4t + 1, where t is the time in
Satu zarah bergerak di sepanjang satu garis lurus supaya
seconds after passing through O. Find
sesarannya, s m, dari titik tetap O diberi oleh s = t3 − 12t2,
Suatu zarah bergerak di sepanjang satu garis lurus dan melalui
dengan keadaan t ialah masa dalam saat selepas gerakan
satu titik tetap O. Sesarannya, s m, diberi oleh s = t2 − 4t + 1,
bermula. Cari
dengan keadaan t ialah masa dalam saat selepas melalui O. Cari
(a) the displacement, in m, when the particle is
(a) the time, in seconds, when the particle stops
instantaneously at rest,
instantaneously,
sesaran, dalam m, apabila zarah itu berehat seketika.
masa, dalam saat, apabila zarah itu berhenti seketika,
(b) the range of values of t, in seconds, when the
(b) the instantaneous velocity, in m s–1, of the
particle moves to the left.
particle when t = 5,
julat nilai t, dalam saat, apabila zarah itu bergerak ke kiri.
halaju seketika, dalam m s–1, zarah itu apabila t = 5,
(c) the range of values of t, in seconds, when the
particle moves to the right, s = t3 − 12t2
ds
julat nilai t, dalam saat, apabila zarah itu bergerak ke v= = 3t2 − 24t
kanan, dt
(d) the initial velocity, in m s–1, of the particle. (a) When v = 0
halaju awal, dalam m s–1, zarah itu. 3t2 − 24t = 0
3t(t − 8) = 0
Solution t = 0 or t = 8
s = t2 − 4t + 1 t = 0 is not accepted. Hence, t = 8 s.
ds
v= = 2t – 4 When t = 8, s = 83 − 12(8)2
dt
= −256 m
(a) When the particle stops instantaneously, v = 0 The particle is 256 m to the left of O.
Apabila zarah berhenti seketika, v = 0
2t − 4 = 0 (b) v0
2t = 4 3t2 − 24t 0
t=2s 3t(t − 8) 0
(d) When/Apabila t = 0,
v = 2(0) – 4
= −4 m s–1
157
Example 4
A particle moves along a straight line and passes through a fixed point O. Its displacement, s m, t seconds after
passing through O is given by s = −2t3 + 5t2 + 4t. Find
Suatu zarah bergerak di sepanjang satu garis lurus dan melalui satu titik tetap O. Sesaran, s m, zarah itu pada masa t saat selepas melalui
O diberi oleh s = −2t3 + 5t2 + 4t. Cari
(a) the value of t, in seconds, when the particle is instantaneously at rest,
nilai t, dalam saat, apabila zarah itu berehat seketika, Smart Tip
(b) the range of values of t, in seconds, when the particle moves to the left, The velocity is maximum or
julat nilai t, dalam saat, apabila zarah itu bergerak ke kiri, dv
minimum when = a = 0.
(c) the maximum velocity, in m s–1, of the particle. dt
Halaju maksimum atau minimum
halaju maksimum, dalam m s–1, zarah itu. dv
apabila = a = 0.
dt
Solution
dv
(a) s = −2t3 + 5t2 + 4t (b) v0 (c) =0
ds dt
v= = −6t2 + 10t + 4 −6t2 + 10t + 4 0 4 (−2) −12t + 10 = 0
dt 3t2 – 5t – 2 . 0 5
When/Apabila v = 0, (3t + 1)(t – 2) . 0 t=
6
−6t2 + 10t + 4 = 0 4 (−2)
3t2 – 5t – 2 = 0 Vmaximum/maksimum
(3t + 1)(t – 2) = 0
( 56 ) + 10( 56 ) + 4
2
= −6
1
t = − 3 or t = 2 1
t . 0, thus, t = 2 s =8 m s−1
1 2
t 6
3
t.2s
158
t
4 6
t
4 6 4st6s
t . 0, thus 0 s < t 6 s dv
(c) = 6t − 30
dv dt
(c) Uniform velocity, =0 dv
dt When =0
−2t + 2 = 0 dt
−2t = 2 6t − 30 = 0
t = 1 s 6t = 30
t = 5
vminimum = 3(5)2 − 30(5) + 72
= –3 m s–1
159
Solution 1
s = − t3 + 3t2 + 27t
s = −t3 + 2t2 + 7t 3
ds
ds v= = −t2 + 6t + 27
v= = −3t2 + 4t + 7 dt
dt
d2s
d2s a = 2 = −2t + 6
a = 2 = −6t + 4 dt
dt
(a) When/Apabila t = 0, a = −6(0) + 4 (a) When t = 0, a = −2(0) + 6 = 6 m s−2
= 4 m s−2
(b) When t = 4, a = −2(4) + 6 = −2 m s−2
(b) When/Apabila t = 3, a = −6(3) + 4
= −14 m s−2
2 A particle moves along a straight line from a fixed 3 A particle moves along a straight line from a fixed
point O. Its velocity, v m s–1, t seconds after passing point O. Its velocity, v m s–1, t seconds after passing
through O is given by v = 12 – (t – 4)2. Find through O is given by v = −10t + 2t2 − 6. Find
Suatu zarah bergerak di sepanjang satu garis lurus dari satu Suatu zarah bergerak di sepanjang satu garis lurus dari satu
titik tetap O. Halajunya, v m s–1, pada masa t saat selepas titik tetap O. Halajunya, v m s–1, pada masa t saat selepas
melalui O diberi oleh v = 12 – (t – 4)2. Cari melalui O diberi oleh v = −10t + 2t2 − 6. Cari
(a) the initial acceleration, in m s–2, of the particle, (a) the initial acceleration, in m s–2, of the particle,
pecutan awal, dalam m s , zarah itu,
–2
pecutan awal, dalam m s–2, zarah itu,
(b) the instantaneous acceleration, in m s , of the –2
(b) the instantaneous acceleration, in m s–2, of the
particle when t = 5. particle when t = 4.
pecutan seketika, dalam m s–2, zarah itu apabila t = 5. pecutan seketika, dalam m s–2, zarah itu apabila t = 4.
160
Smart Tip
Displacement, s Velocity, v Acceleration, a
Sesaran, s Halaju, v Pecutan, a
s = v dt v = a dt
Example 6
A particle moves along a straight line and passes through a fixed point O. Its velocity, v m s–1, is given by
v = t2 − 9t + 18, where t is the time in seconds after leaving point O.
Suatu zarah bergerak di sepanjang satu garis lurus dan melalui satu titik tetap O. Halajunya, v m s–1, diberi oleh v = t2 − 9t + 18,
dengan keadaan t ialah masa dalam saat selepas meninggalkan titik O.
(a) Find the initial velocity, in m s–1, of the particle.
Cari halaju awal, dalam m s–1, zarah itu.
(b) Find the maximum displacement, in m, of the particle.
Cari sesaran maksimum, dalam m, zarah itu.
(c) Sketch a velocity-time graph for a time period 0 < t < 6.
Lakarkan graf halaju-masa untuk tempoh masa 0 < t < 6.
(d) Find the total distance, in m, travelled by the particle in the first 5 seconds.
Cari jumlah jarak, dalam m, yang dilalui oleh zarah itu dalam 5 saat yang pertama.
Solution
(a) When/Apabila t = 0, (c) v
v = (0)2 − 9(0) + 18 = 18 m s–1
(b) v=0 18
t − 9t + 18 = 0
2
(t − 6)(t − 3) = 0 t
t = 6 or/ atau t = 3 0 3 6
s = ∫ v dt
(d) Area above the x-axis/ Luas di bahagian atas paksi-x
= ∫ t2 − 9t + 18 dt 3
t3 9t2 = (t2 − 9t + 18) dt
0
= 3 − 2 + 18t + c
[
t3 9t2
]
3
= – + 18t
When/Apabila t = 0, s = 0, 3 2 0
(0)3 9(3)3
0 = 3 − 2 + 18(0) + c
= [
(3)3 9(3)2
3
–
2
+ 18(3) –
3 ] [
(0)3 9(0)2
–
2
+ 18(0) ]
c=0 = 22.5 − 0
t3 9t2 = 22.5 m
s = 3 − 2 + 18t
Smart Tip Area below the x-axis/ Luas di bahagian bawah paksi-x
When/Apabila t = 3, The displacement is 5
= (t2 − 9t + 18) dt
(3)3 9(3)2 3
[ ]
s = 3 − 2 + 18(3) maximum or minimum t3 9t2 5
ds = – + 18t
= 22.5 m when = v = 0. 3 2 3
dt
When/Apabila t = 6,
Sesaran maksimum atau
minimum apabila
ds
= v = 0.
= [
(5)3 9(5)2
3
–
2
+ 18(5) –
3 ] [
(3)3 9(3)2
–
2
+ 18(3) ]
dt 1
(6)3 9(6)2 = 19 6 − 22.5
s = 3 − 2 + 18(6)
= 18 m 1
= −3 3 m
Maximum displacement = 22.5 m
Sesaran maksimum = 22.5 m |
1
Total distance/Jumlah jarak = 22.5 + −3 3 |
5
= 25 6 m
161
(a) s = ∫ v dt (b) v = 0
= ∫ (4 − 11t − 3t2) dt 4 − 11t − 3t2 = 0
11t 2 3t 3 3t2 + 11t − 4 = 0
= 4t − 2 − 3 + c
(3t − 1)(t + 4) = 0
11t 2 3t – 1 = 0 or t + 4 = 0
= 4t − 2 − t3 + c
3t = 1 t = −4
1
When t = 0, s = 0, t =3
11(0)2 1
0 = 4(0) − 2 − (0)3 + c t . 0, thus t = 3
c=0
11 ()
1 2
11t 2
Thus, s = 4t − 2 − t3 s = 4 ()
1
3
−
2
3
−()1 3
3
37
= m
54
11(2)2
(c) s = 4(2) − − (2)3
2
= −22 m
The particle is located 22 m to the left from
the fixed point O when t = 2.
2 The acceleration of a particle that moves along a straight line, t seconds after passing through a fixed point
O is given by a = t − 6. Given the initial velocity of the particle is 5 m s−1. Find
Pecutan suatu zarah yang bergerak di sepanjang satu garis lurus, pada masa t saat selepas melalui titik tetap O diberi oleh
a = t − 6. Diberi halaju awal zarah itu ialah 5 m s−1. Cari
(a) the minimum velocity, in m s–1, of the particle,
halaju minimum, dalam m s–1, zarah itu,
(b) the range of values of t, in seconds, when the particle is decelerating.
julat nilai t, dalam saat, apabila zarah itu mengalami nyahpecutan.
162
ACTIVITY
= −t2 + 7t + c
(c) s = ∫ v dt
When t = 0, v = 18 9
18 = −(0)2 + 7(0) + c = (−t2 + 7t + 18) dt
0
c = 18
[
t3 7t2
]
9
= – + + 18t
3 2 0
Thus, v =−t2 + 7t + 18
vmax = −
7 2
() ()
+7
7
+ 18
= – [
(9)3 7(9)2
3
+
2 ] [
+ 18(9) – –
(0)3 7(0)2
3
+
2
+ 18(0) ]
2 2
= 202.5 – 0
1
= 30 m s−1 = 202.5 m
4
(ii) When the particles stops at t, v = 0
−t2 + 7t + 18 = 0
t2 − 7t − 18 = 0
(t − 9)(t + 2) = 0
t = 9 or t = −2
t > 0, thus t = 9 s
PAK-21 Think-Pair-Share
Steps/Langkah-langkah:
1 Students choose their partner.
Murid memilih pasangan masing-masing.
2 Teacher gives a set of question related to the kinematics of linear motion to each pair.
Guru memberikan satu set soalan berkaitan kinematik gerakan linear kepada setiap pasangan murid.
Example/ Contoh:
A particle moves along a straight line. Its displacement, in m, is s = t3 – 3t2 – 4t at t seconds after passing
through a fixed point O. Find the initial velocity, in m s –1, of the particle.
Suatu zarah bergerak di sepanjang satu garis lurus. Sesarannya, dalam m, adalah s = t3 – 3t2 – 4t pada masa t saat
selepas melalui satu titik tetap O. Cari halaju awal, dalam m s –1, zarah itu.
3 In pairs, students are required to discuss and solve the question given.
Secara berpasangan, murid dikehendaki berbincang dan menyelesaikan soalan yang diberi.
4 Students share their solution and explain the answers obtained to all the students in the class.
Murid berkongsi penyelesaian dan menjelaskan jawapan yang diperoleh kepada semua murid di dalam kelas.
5 Teacher evaluates the answer and the other students compare their answers.
Guru menilai jawapan dan murid lain membandingkan jawapan mereka.
163
Example 7
A particle moves along a straight line and passes through a fixed point O. Its acceleration, a m s−2, t seconds
after passing through O is given by a = −6t + 12. Given the initial velocity of the particle is 36 m s−1. Find
Suatu zarah bergerak di sepanjang satu garis lurus dan melalui satu titik tetap O. Pecutannya, a m s−2, pada masa t saat selepas melalui
O diberi oleh a = −6t + 12. Diberi halaju awal zarah ialah 36 m s−1. Cari
(a) the range of values of t, in seconds, when the particle moves to the left,
julat nilai t, dalam saat, apabila zarah itu bergerak ke kiri,
(b) the instantaneous displacement of the particle, in m, when t = 2.
sesaran seketika zarah itu, dalam m, apabila t = 2.
Solution
(a) v = ∫ a dt (b) s = ∫ v dt
= ∫ (−6t + 12) dt s = ∫ (−3t2 + 12t + 36) dt
= −3t2 + 12t + c = −t3 + 6t2 + 36t + c
When/Apabila t = 0, v = 36 When/Apabila t = 0, s = 0
36 = −3(0)2 + 12(0) + c 0 = −(0)3 + 6(0)2 + 36(0) + c
c = 36 c=0
Thus/ Maka, v = −3t2 + 12t + 36
Thus/maka, s = −t3 + 6t2 + 36t
When the particle moves to the left, v 0
When/Apabila t = 2,
Apabila zarah bergerak ke kiri, v 0
s = −(2)3 + 6(2)2 + 36(2)
−3t2 + 12t + 36 0
= 88 m
t2 − 4t – 12 . 0
(t + 2)(t – 6) . 0
[ The instantaneous displacement is 88 m
when t = 2.
Sesaran seketika ialah 88 m apabila t = 2.
t
2 6
t . 0, thus/maka t . 6 s
1 A particle moves along a straight line so that its acceleration, a m s−2, t seconds after passing through a fixed
point O is given by a = − 2t + 3. Given the initial velocity of the particle is 4 m s−1. Find the maximum velocity,
in m s–1, of the particle.
Suatu zarah bergerak di sepanjang satu garis lurus supaya pecutannya, a m s−2, t saat selepas melalui titik tetap O diberi oleh
a = − 2t + 3. Diberi halaju awal zarah ialah 4 m s−1. Cari halaju maksimum, dalam m s–1, zarah itu.
v = ∫ a dt d2v
= ∫ (−2t + 3) dt dt2 =–2, ( 0), v is maximum
3
= −t2 + 3t + c When t = 2
When t = 0, v = 4
4 = −(0)2 + 3(0) + c () ()
3 2 3
vmax = – 2 + 3 2 + 4
c=4 = 6.25 m s–1
Thus, v = −t2 + 3t + 4
Maximum velocity
–2t + 3 = 0
2t = 3
3
t=2
164
(a) v = ∫ a dt (b) s = ∫ v dt
= ∫ (−6) dt s = ∫ (−6t + 12) dt
= −6t + c = −3t² + 12t + c
When t = 0, v = 12 When t = 0, s = 0
v = −6t + c 0 = − 3(0)² + 12(0) + c
12 = − 6(0) + c c = 0
c = 12
Thus, s = −3t² + 12t
Thus, v = −6t + 12
When s = 0,
When the particle moves to −3t² + 12t = 0
the right, v . 0. 3t2 – 12t = 0
− 6t + 12 . 0 3t(t – 4) = 0
−6t . −12 t = 0 or t = 4
t , 2
The particle passes through O for the
second time when t = 4.
3 A particle moves along a straight line from a fixed point O with an initial velocity of 9 m s−1. Its acceleration,
a m s−2, t seconds after passing through O is given by a = −4t + 3. Find
Suatu zarah bergerak di sepanjang satu garis lurus dari satu titik tetap O dengan halaju awal 9 m s−1. Pecutannya, a m s−2, pada
masa t saat selepas melalui O diberi oleh a = −4t + 3. Cari
(a) the range of values of t, in seconds, when the particle moves to the left,
julat nilai t, dalam saat, apabila zarah itu bergerak ke kiri,
(b) the instantaneous displacement, in m, of the particle when t = 4.
sesaran seketika, dalam m, zarah itu apabila t = 4.
(a) v = ∫ a dt (b) s = ∫ v dt
= ∫ (−4t + 3) dt s = ∫ (−2t2 + 3t + 9) dt
= −2t2 + 3t + c 2 3
x = − 3 t³ + 2 t2 + 9t + c
When t = 0, v = 9 3 3
2
9 = −2(0)2 + 3(0) + c When t = 0, s = 0
c = 9 2 3
t . 0, thus, t . 3 s 0 = − 3 (0)³ + 2 (0)2 + 9(0) + c
Thus, v = −2t2 + 3t + 9 c=0
When the particle moves to 2 3
the left, v 0 s = − 3 t³ + 2 t2 + 9t
−2t2 + 3t + 9 0
When t = 4,
2t2 − 3t – 9 . 0
2 3
(2t + 3)(t – 3) . 0 s = − 3 (4)³ + 2 (4)2 + 9(4)
1
= 17 3 m
165
Example 8
1 An object moves along a straight line and passes
An object moves along a straight line from a fixed through a fixed point O with an initial velocity of
point O. Its velocity, v m s–1, at t seconds after 14 m s−1. Its acceleration, a m s−2, at t seconds after
passing through O is given by v = 13t – 3t2 + 10. passing through O is given by a = 12 – 4t.
Find Suatu objek bergerak di sepanjang satu garis lurus dari satu
titik tetap O dengan halaju awal 14 m s−1. Pecutannya, a m s−2,
Suatu objek bergerak di sepanjang satu garis lurus dari satu
pada masa t saat selepas melalui O diberi oleh a = 12 – 4t.
titik tetap O. Halaju, v m s−1, objek itu pada masa t saat
(a) Find the instantaneous displacement, in m, of
selepas melalui O diberi oleh v = 13t – 3t2 + 10. Cari
the object when its velocity is uniform.
(a) the acceleration, in m s–2, of the object when Cari sesaran seketika, dalam m, objek itu apabila halajunya
t = 1, adalah seragam.
pecutan, dalam m s−2, bagi objek itu apabila t = 1,
(b) Sketch a velocity-time graph for 0 < t < 7.
(b) the total distance, in m, travelled by the object Lakarkan graf halaju-masa untuk 0 < t < 7.
in the first 6 seconds.
jumlah jarak, dalam m, yang dilalui oleh objek itu dalam (a) Given a = 12 – 4t
6 saat yang pertama. v = ∫ a dt
= ∫ (12 − 4t) dt
Solution
= 12t − 2t2 + c
(a) v = 13t – 3t2 + 10
dv When t = 0, v = 14
a= = 13 – 6t
dt 14 = 12(0) – 2(0)2 + c
When/ Apabila t = 1, c = 14
a = 13 – 6(1) = 7 m s−2
Thus, v = 12t − 2t2 + 14
(b) When the object stops, v = 0 s = ∫ v dt
Apabila objek berhenti, v = 0 = ∫ (12t − 2t2 + 14) dt
2
13t – 3t2 + 10 = 0 = 6t2 – t3 + 14t + c
3
3t2 – 13t – 10 = 0 When t = 0, s = 0,
(3t + 2)(t – 5) = 0 2
3t + 2 = 0 or t – 5 = 0 0 = 6(0)2 – (0)3 + 14(0) + c
3
2 c=0
t = −3 t=5 2
Thus, s = 6t2 – t3 + 14t
t . 0, thus/ maka t = 5 v 3
v = 13t 3t2 + 10 For uniform velocity, a = 0
12 – 4t = 0
t=3
10 A 2
s = 6(3)2 – (3)3 + 14(3)
3
= 78 m
t
Area A/ Luas A 0 5 B6
5 (b) v = – 2t2 + 12t +14
= (13t – 3t2 + 10) dt
0 = −2(t2 – 6t – 7)
[
13 2 3
]
5
= t – t + 10t = −2(t + 1)(t – 7)
2 0
= [
13 2
2
(5) – (5) + 10(5) – 0
3
] v
Area B/ Luas B
6
= (13t – 3t2 + 10) dt
5
14
[
13 2 3
]
6
= t – t + 10t 0
t
2 5 7
= [
13 2
2
(6) – (6)3 + 10(6) –
13 2
2 ] [
(5) – (5)3 + 10(5) ]
= − 9.5 m
Total distance = Area A + Area B
Jumlah jarak = Luas A + Luas B
= 87.5 +|–9.5|
= 97 m
166
(a) v = 9t – 4t2 + 9
dv
a= = 9 – 8t
dt
1
When t = 2 ,
a=9–8 2
1
()
= 5 m s–2
(b) When the object stops,
v = 0
9t – 4t2 + 9 = 0
4t2 – 9t – 9 = 0
(4t + 3)(t – 3) = 0
4t + 3 = 0 or t – 3 = 0
3 t = 3
t = − 4
t . 0, thus, t = 3
v
v = 9t 4t2 + 9
9 A
5
t
0 3 B
B
46
Area A
3
= 0 (9t – 4t2 + 9) dt
9
[ 4
]
3
= t2 – t3 + 9t
2 3 0
[
9 2 4 3 9 4
= (3) – (3) + 9(3) – (0)2 – (0)3 + 9(0)
2 3 2 3 ] [ ]
= 31.5
Area B
5
= 3 (9t – 4t2 + 9) dt
[9 4
]
5
= t2 – t3 + 9t
2 3 3
[9 2 4 3 9 4
] [
= (5) – (5) + 9(5) – (3)2 – (3)3 + 9(3)
2 3 2 3 ]
1
= –9 – 31.5
6
2
= –40
3
Total distance
= Area A + Area B
= 31.5 + –40
2
3| |
1
= 72 m
6
167
ds 2
(a) v = = 3mt2 + 2pt + 10 (b) s = − 3 t3 + 4t2 + 10t
dt
dv ds
a = = 6mt + 2p v = = −2t2 + 8t + 10
dt dt
At maximum velocity, a = 0, t = 2 When v = 0,
0 = 6m(2) + 2p −2t2 + 8t + 10 = 0
12m + 2p = 0 t2 – 4t – 5 = 0
6m + p = 0 (t + 1)(t – 5) = 0
p = − 6m 1 t = –1 or t = 5
t . 0, thus t = 5
Maximum velocity = 18, t = 2
v
vmaximum = 3m(2)2 + 2p(2) + 10 v = 2t2 + 8t + 10
18 = 12m + 4p + 10
12m + 4p = 8
3m + p = 2 2 10 A
Substitute 1 into 2 , t
0 5 B8
3m + (−6m) = 2
−3m = 2
2 54
B
m = − 3
2 Area A
Substitute m = − 3 into 1 , 5
= 0 (–2t2 + 8t + 10) dt
p = −6 − 3
2
( ) 2
[
= – t3 + 4t2 + 10t
5
]
= 4 3 0
2
[ m = – and p = 4
3
2 3
[ 2
] [
= – (5) + 4(5) + 10(5) – – (0)3 + 4(0)2 + 10(0)
3
2
3 ]
2
= 66
3
Area B
8
= 5 (–2t2 + 8t + 10) dt
2
[ ]
8
= – t3 + 4t2 + 10t
3 5
2 3
3 [ 2 2
] [
= – (8) + 4(8) + 10(8) – – (5)3 + 4(5)2 + 10(5)
3 ]
1 2
= –5 – 66
3 3
= −72
2
Total distance = 66 + |–72|
3
2
= 138 m
3
168
Section C
1 A particle moves along a straight line from a fixed point O with an initial velocity of –6 m s−1. Its acceleration, a m s−2,
at t seconds after passing through O is given by a = 5 – 2t.
[Assume the movement to the right is positive]
Suatu zarah bergerak di sepanjang satu garis lurus dari satu titik tetap O dengan halaju awal –6 m s−1. Pecutan, a m s−2, zarah itu pada
masa t saat selepas melalui O diberi oleh a = 5 – 2t.
[Anggapkan gerakan ke arah kanan adalah positif]
Find/Cari
(a) the range of time, in seconds, when the particle is decelerating,
julat masa, dalam saat, apabila zarah itu mengalami nyahpecutan,
[2 marks/markah]
(b) the time, in seconds, when the particle is instantaneously at rest,
masa, dalam saat, apabila zarah itu berehat seketika,
[4 marks/markah]
(c) the total distance, in m, travelled by the particle in the first 4 seconds.
jumlah jarak, dalam m, yang dilalui oleh zarah itu dalam 4 saat yang pertama.
[4 marks/markah]
2 A particle moves along a straight line from fixed point O. Its velocity, v m s−1, t seconds after passing through O is given
by v = kt2 – 8t, where k is a constant. The acceleration of the particle is 16 m s−2 when t = 2 s.
[Assume the movement to the right is positive]
Suatu zarah bergerak di sepanjang satu garis lurus dari satu titik tetap O. Halajunya, v m s−1, pada masa t saat selepas melalui O diberi
oleh v = kt2 – 8t, dengan keadaan k ialah pemalar. Pecutan zarah itu ialah 16 m s−2 apabila t = 2 s.
[Anggapkan gerakan ke arah kanan adalah positif]
Find/Cari
(a) the value of k,
nilai k,
[3 marks/markah]
(b) the range of values of t, in seconds, when the particle moves to the left,
julat nilai t, dalam saat, apabila zarah itu bergerak ke kiri,
[2 marks/markah]
(c) the value of t, in seconds, when the particle is instantaneously at rest,
nilai t, dalam saat, apabila zarah itu berehat seketika,
[2 marks/markah]
(d) the total distance, in m, travelled by the particle in the first 3 seconds.
jumlah jarak, dalam m, yang dilalui oleh zarah itu dalam 3 saat yang pertama.
[3 marks/markah]
3 A particle starts from a fixed point O and moves along a straight line. After t seconds, its velocity, v m s−1, is given
by v = 16 + 6t − t 2. Calculate
[Assume the movement to the right is positive]
Suatu zarah bermula dari satu titik tetap O dan bergerak di sepanjang satu garis lurus. Selepas t saat, halajunya, v m s−1, diberi oleh
v = 16 + 6t − t2. Hitung
[Anggapkan gerakan ke arah kanan adalah positif]
(a) the instantaneous velocity, in m s−1, of the particle when t = 5,
halaju seketika, dalam m s−1, zarah itu apabila t = 5,
[2 marks/markah]
(b) the range of values of t, in seconds, when the particle moves to the right,
julat nilai t, dalam saat, apabila zarah itu bergerak ke kanan,
[2 marks/markah]
(c) the maximum velocity, in m s−1, of the particle,
halaju maksimum, dalam m s−1, bagi zarah itu,
[3 marks/markah]
(d) the total distance, in m, travelled by the particle in the first 9 seconds.
jumlah jarak, dalam m, yang dilalui oleh zarah itu dalam 9 saat pertama.
[3 marks/markah]
169
H O TS Zo n e
1 An object moves along a straight line from a fixed point O. Its acceleration, a m s–2, at t seconds after passing
through O is given by a = 18 – ht, where h is a constant. When t = 2, the particle has a deceleration of 6 m s−2 and a
velocity of 72 m s−1. When t = 3, the particle is at P and when t = 4.5, the particle is at Q.
Suatu zarah bergerak di sepanjang satu garis lurus dari satu titik tetap O. Pecutannya, a m s–2, pada masa t saat selepas melalui O diberi
oleh a = 18 – ht, dengan keadaan h ialah pemalar. Apabila t = 2, zarah itu mengalami nyahpecutan 6 m s–2 dan mempunyai halaju
72 m s–1. Apabila t = 3, zarah itu berada di P dan apabila t = 4.5, zarah itu berada di Q.
(a) Find/ Cari
(i) the value of h,
nilai h,
(ii) the velocity at P.
halaju di P.
(b) Sketch a velocity-time graph for the particle. Analysing
170